Microbiology Exam 4, microbiology exam 4, Microbiology Exam 4, microbiology exam 4, Microbiology Exam 4 - Kakirde, Microbiology Exam 4, Microbiology Exam 4, Microbiology Exam 4, Microbiology Exam 4, Microbiology Exam 4, Exam 4 Microbiology, Microbiol...
Rhabdovirus: rabies what is the reservoir
Order Carnivora (foxes, skunks, etc,) Raccoons in mid-Atlantic, Northeast Skunks, bats in Midwest
How do pathogens passively penetrate host tissue?
Organism penetrates tissue due to wound, burn, insect bite, animal bite, etc.
faintly staining, thin gram-negative bacilli.
Organisms belonging to the genus Legionella are:
What is Septicemia?
Organisms multiplying/colonizing the bloodstream Sepsis, blood poisoning
RNA viruses that includes seven genera: Influenza virus A, Influenza virus B, Influenza virus C, Influenza virus D, Isavirus, Thogotovirus and Quaranjavirus.
Orthomyxovirus
What are the pros and cons of each test ?
Pheno: are very simple and not very time consuming, but are too generalized Geno: are specific and viral, but very time consuming and expensive Immun: are specific and quick, but too specific negative results are common
What are some examples of bacterial PAMPs?
Peptidoglycan Teichoic acid Lipopolysaccharide fMet
Helicobacter pylori
Performing the Urea breath test will cause a color change when this organism is detected:
What occurs in a lysogenic cycle of a bacteriophage?
Phage DNA incorporates into host chromosome.
What occurs in the lytic cycle of a bacteriophage?
Phage DNA takes over cell, replicates and ultimately lyses (kills) host - releases DNA.
*WHAT* destroy viruses trapped in complexes with antibodies
Phagocytes
What encompasses the second line of defense?
Phagocytes Inflammation Fever Antimicrobials
What is the agent that causes Chagas disease?
Protozoan - Trypanosoma cruzi (vector-bug)
L-cysteine and iron
To cultivate Legionella in the laboratory, a medium supplemented with what ingredient(s) is necessary?
aerotolerance test
To determine if an organism is definitely an obligate anaerobe the following procedure must be performed:
Disease: GAS GANGRENE /ANAEROBIC CELLULITIS & MYONECROSIS Causative organism: CLOSTRIDIUM
Type of Organism: bacterial Part of Body Affected: Cardiovascular & lymphatic system Mode of Transmission: INTERRUPTED BLOOD SUPPLY to TISSUE - ISCHEMIA
What are the 2 physical barriers of mucous membranes and their secretions? How do they work?
Mucus - contains lysozymes Cilia - sweeps microbes from lower respiratory tract
What is the portal of entry for Tularemia?
Multiple points of entry (eyes , respiratory tract) Lesions Lymph nodes Conjuctivitis *Zoonotic (fleas and ticks)
Inflammation of parotid glands More serious complications possible
Mumps
•
Mumps (paramyxovirus)
What are the symptoms for Botulism ?
Muscle weakness, cranial nerves (children susceptible - opportunistic disease)
Leprosy (disease of nervous system)
Mycobacterium leprae. acid fast; development of lesions on skin, bodys cell mediated response destroys nerve endings, loss of sensory perception in areas of skin that have been infected, insensitive areas easily become infected w secondary infections - bacteria goes to nerve cells so theyll travel to appendages , endemic in texas and louisiana bc amardillos.
What is the #1 single infectious disease killer?
Mycobacterium tuberculosis
What is the causative agent of tuberculosis?
Mycobacterium tuberculosis
•
Mycobacterium tuberculosis (Tuberculosis)
Tuberculosis (disease of respiratory)
Mycobacterium tuberculosis; chronic infection of lower respiratory tract, low grade fever, chronic cough, weight loss, green bloody sputum, night sweat
Allows new virions to escape from cell without getting stuck
N antigen: neuraminidase
What is produced by the urease enzyme?
NH3 and CO2
Equation of nitrogen assimilation?
NH3→ R-NH2 (amino group)
Detect absence of MHC proteins due to viral infection, kill cells
NK cells
natural killer cells
NK cells are lymphocytes that bind to and attack foreign body cells NK cells bind to receptors on any cell lacking class 1 MHC proteins-> NK cells releases pore forming proteins called perforins and granzymes-> foreign cell lyses
Denitrification equation?
NO3 → NO2 → N2 (in some bacteria can proceed to ammonia (NH3)
•
Nucleic acids methods for identification of infectious disease agents
•
Nucleoside reverse transcriptase inhibitors (NRTI)
What is an infectious dose?
Number of organisms required to cause an infection.
Gastrointestinal Microbiota
Numbers of bacteria in the monogastric human gastrointestinal tract = 10^13 to 10^14 microbial cells *Microbial populations* in different areas of the GI tract are *influenced by diet and the physical conditions in the area*
Prebiotics
Nutrients that encourage the growth of beneficial microbes *Nutrients*
specific immuunity
-must be aquired -includes 3rd line of defense -there is a lag time between exposure and maximal response -antigen specific -exposure results in immunologic memory
CLOSTRIDIUM (bacterial)
1. TETANUS (NS) 2. BOTULISM(NS) 3. GAS GANGRENE/ANAEROBIC CELLULITIS & MYONECROSIS (Cadio /Lymph) 4. PSEUDOMEMBRANOUS COLITIS (GASTROENTERITIS)
CHLAMYDIA (bacterial)
1. TRACHOMA/INCLUSION CONJUNCTIVITIS (eye) 2. PSITTACOSIS/ORNITHOSIS (lower respiratory) 3. CHLAMYDIAL URETHRITIS, NGU, NONGONOCOCCAL URETHRITIS (genital and urinary)
3 types of of Horizontal Gene Transfer
1. Transduction 2. Conjugation 3. Transformation
What is required for bacterial virus replication?
A live, actively growing cell. If the cell isn't growing then the virus can't replicate.
What is an acute infection?
A severe and short infection.
Which TB infection is being described? - Only 10% of infected people develop the infection - Without treatment very few survive
Active tuberculosis infection
•
Activity spectrum
Indirect effects: Damage from virus results in activation of non-specific defenses ---- _________ cells, Natural Killer (NK) cells kill infected host cells ---- Complement, interferon result in ______ ______ _____
Phagocytic host cell death
What do dendritic cells do?
Phagocytosis.
What do neutrophils do?
Phagocytsosis.
What is responsible for digestion in the first 3 compartments of the rumen?
Bacteria and archaea, this is a very anaerobic environment.
Adherence to Epithelial Cells
Bacteria and viruses that initiate infection often adhere specifically to epithelial cells *through interactions between molecules on the surfaces of the pathogen and the host cell*
Bacteria can be toxigenic (produce toxins)
Bacteria may not spread, but release soluble toxins which dissolve in body fluids, damaging cells.
What type of disease is Septicemia?
Bacteria or fungal
What are the symptoms of Cutaneous Anthrax?
Characteristic lesion - black spot on sore, can cause septsis
•
Chicken pox (varicella)
VZV
Chickenpox and shingles Replicates in epithelial cells
Chlamydia (disease of urinary)
Chlamydia traachomatis, obligate intracellular pathogen
What antibiotic(s) target protein synthesis (ribosomes)?
Chloramphenicol Aminoglysocides Macrolides Tetracycline
What is a prevention for Naegleria fowleri?
Chlorination
28.4
Choosing the Right Treatment
Risk Factors of Infection
Compromised host --One or more resistance mechanisms are inactive --The probability of infection is increased --Age ----Young and old individuals are more susceptible --Stress can predispose a healthy individual to disease --Diet plays a role in host susceptibility to infection --Certain genetic conditions can compromise a host
Urea is?
Common form of nitrogen found in animal and human waste. NH2CONH2
Normal Microbiota aids host defenses by
Competing w/ pathogens for nutrients Occupying host surfaces Killing invaders with bacteriocins Changing conditions, e.g. pH Stimulating host defenses ---Leaking into body, keeping defenses on alert Producing vitamins benefiting overall host health
What is cytolysis?
Complement proteins assemble to form a membrane attack complex (MAC) and bind to pathogens. They create holes in the membranes and kill bacteria. (this works especially well in Gram negative bacteria.)
What is opsonization?
Complement proteins coat pathogens, encourages phagocytic cells to eat the pathogens.
Aerotolerant Anaerobes
Do not require oxygen as they metabolise energy anaerobically. Unlike obligate anaerobes however, they are not poisoned by oxygen. They can be found evenly spread throughout the test tube.
What is the diagnosis for fifths disease?
Doctor doing a visual test with patient and blood testing for parviovirus
Protozoan cyst
Dormant and protective form of protozoa
Pustule
Pus filled
What keeps bacteria under the radar
Quorum sensing
Equation of ammonification?
R-NH2 → NH3
Environmental factor affecting the skin microbiome (e.g. hygiene, temperature, humidity)
Composition is influenced by: ---Environmental factors (e.g., weather) ---Host factors (e.g., age, personal hygiene)
What is the process of the expression of viral genes?
Early genes stop host physiology, start virus replication (host DNA is broken down and the nucleotides are then used to make viral DNA)
Most common infectious diseases: GI and Resp. why?
Easy in, Easy out; most accessible Greatest frequency of exposure
What is the portal of entry for Acanthamoeba?
Epithelial entry (eyes, skin)
M protein
RHEUMATIC FEVER antigen - INHIBITS PHAGOCYTOSIS
infection process of HIV
RNA is copied into cDNA by reverse transcriptase cDNA inserts into host chromosome New RNA made Protein precursor made, then processed; assembly occurs Virions bud through cell membrane
Which type of viruses mutate more rapidly? Why?
RNA viruses do not have proof-reading in RNA-dependent polymerase, so mutate much more rapidly than DNA viruses.
Translation
RNA-> Protein
Negative (-) RNA viruses are complementary to a message, so they must be transcribed into a message by ___________?
RNA-dependent RNA polymerase. (This basically turns it into RNA (+) virus)
Papule
Raised lesion
•
Rapid Tests
What are some additional causes of meningitis
Escherichia coli (Rod shaped) Streptococcus agalactiae (Sphere shaped) Cronobacter sakazakii (Chromogenic differential agar- turn blue)
Common causes of Urinary infections
Escherichia coli, Pseudomonas, Enterococcus, Klebsiella, Proteus, Staphylococcus
Etest
Etest is a non-diffusion-based technique that employs a preformed and predefined gradient of an antimicrobial agent immobilized on a plastic strip
Mold morphology
Eukaryotic, chitin and cellulose cell walls, obligate aerobes, grow by elongation
What is passive humoral immunity?
Received from a donor. Can be artificial (ex: receiving antibodies from another donor) or natural (ex: from mother to offspring)
NK Cell
Recognizes cells displaying stress induced proteins not MHC-1 proteins and kills them
What is the vaccine for Malaria?
Recombinant protein vaccine passed stage 3 clinical trials as of January 2016
recombination/reassortment - how do they happen and what do they cause;
Recombination and reassortment of antigens can result in virus that can jump species, introducing more pathogenic strains into different populations
What is the portal of entry for the Bubonic Plague?
Respiratory tract or bites
What are the symptoms of Bubonic Plague?
Results in massive bacterial growth within the bloodstream (Bubos) Hemorrhage Obstructed blood vessels
HIV Movement
Reverse transcriptase makes DNA copy of RNA genome Integrase binds the DNA copy and inserts into host cell's chromosomes (provirus) Can exist as latent or productive
What are the symptoms of the measles ?
Exanthem bumps, Laryngitis, Secondary infections- sepsis (infection in the blood stream) *Rare: encephalitis (swelling of brain), neuron destruction
Dimorphic fungi
Exhibit two growth forms EX: Outside the body they are mold, but in the body they are yeast Systemic mycoses by mold spore inhalation
3)
Explain how diet plays a role in host susceptibility to infection. Discuss other host factors that can affect our resistance to infections.
3)
Explain the difference between direct agglutination and passive agglutination.
How do complement proteins differentiate between self cells and pathogens?
Self cells are already bound to other serum proteins, so complement proteins bind to pathogens not bound to other serum proteins.
DNA Replication
Semiconservative
Species, individual variations in presence/structure of receptors
Genetic resistance
•
Serological tests to detect pathogen-induced antibodies
What is the diagnosis for Tularemia?
Serology (can be misinterpreted as a number of other diseases) PCR
What are complement proteins?
Serum proteins that "complement" the innate immune system.
What is the agent for Meningitis?
Several causative microorganisms
What is a staph infection?
Staphylococcus aureus bacteria in the dermal layer of skin
What is the agent for Endocarditis ?
Staphylococcus aureus, Streptococcus, Neisseria gonorrhoeae
Toxic shock syndrome (disease of urinary)
Staphylococcus aureus; bacteria enters blood and produces exotoxins; low bp, red rash on trunk, vomiting and fever
Scalded Skin Syndrome (skin disease)
Staphylococcus aureus; common in infants; high fever, septicemia; death can occur; caused by exotoxins called exfoliants. toxins travel through bloodstream to skin causing upper skin layers to seperate and peel.
Opsonins
Stick viruses to phagocytes
Why does polymyxin work well against gram - cells?
Targets the cell membrane, and gram - have the outer membrane sticking out
Chlamydoconidia
Survival spores
What is Meningioencephalitis ?
Swelling of brain and meninges
Herpes viruses
Switch between productivity in epithelial cells and latency in neuronal cells
False
T/F: N. gonorrhoeae is oxidase positive and able to ferment glucose, maltose, and sucrose.
Chemokine receptors
T4 helper lymphocytes Macrophages Monocytes Dendritic cells Determine type of CD4+ cell HIV is able to infect
AIDS
T4 lymphocytes being destroyed and we can no longer replace them Marked decline in cytotoxic lymphocytes
HIV Replication
T4 lymphocytes stimulate antigens or cytokines activate the HIV Molecules of mRNA transcribed off provirus DNA then it goes through nuclear pores to the ER for translation
Chronic HIV Phase
T4 replacement equals T4 dying rate, but eventually dying rate outpaces Reverse transcriptase has high error rate
Nucleocapsid
Genome surrounded by a capsid Lacks an envelope
What is the treatment for Tularemia?
Gentamicin, streptomycin
water borne pathogen from recreational water
Giardia
Giardia lamblia
Giardiasis Flagella movement through oral-fecal route
What makes the large intestines such a great fermentation vessel?
Large intestine most heavily colonized part of the GI Colon is an excellent in vivo fermentation vessel *Facultative aerobes* (e.g. E. coli) deplete all oxygen and create an anoxic environment
What are the symptoms of Smallpox?
Large nutrophil filled bumps
What are the symptoms of Leishmaniasis?
Large open wound that kills the epidermis and dermis
Macroconidia
Large, multinucleate of fungus
What type of infection is Chicken Pox and Shingles ?
Latent
What type of infection is Roseola?
Latent ; sub-clinical
Which TB infection is being described? - People are carriers
Latent tuberculosis infection
What type of infection is fifth disease?
Latent; sub-clinical
•
HIV-1 and HIV-2
What is the treatment for rabies?
HRIG (prevention, or 4 doses within 2 weeks after the infection) Immunotherapy *ONCE REACHES BRAIN NO CURE*
What are the mortality rates of Meningioencephalitis ?
High rates
How does viral replication occur in the cell?
Later genes produce capsids, get viral DNA "packaged" into capsids, start lysis or budding (release of virions).
post-exposure prphylaxis
High doses of anti-retroviral drugs within 1-72 hours of exposure Treatment would last at least 28 days Can decrease viral amounts to undetectable levels
What are symptoms of Nonhemorrhagic Fever?
High fever, no capillary disruption
What does Hfr stand for?
High frequency recombinant (type of conjugation)
C (the diagnosis is cryptosporidiosis, from Cryptosporidium hominis, which causes mild diarrhea unless you don't have an immune system - then you're the patient) (treatment is supportive with rehydration and nitazoxanide or paromomycin)
Ignatius presents with severe watery diarrhea and gives a stool sample to be examined in the lab, which appears as shown under acid-fast staining. Which of the following is most likely true about Ignatius? A. Swims in freshwater streams B. Routinely eats undercooked meat C. Is HIV positive D. Works on a farm E. Owns a dog
What is multiple sclerosis?
Immune system treats a myelin basic protein as foreign.
What is type I diabetes mellitus?
Immune system treats pancreatic beta cell antigen as foreign.
What is rheumatoid arthritis?
Immune system treats synovial joint antigen as foreign.
not being susceptible to disease
Immunity
Yes- this is a suitable source
Indicate whether the following clinical specimens are suitable or unsuitable for anaerobic culture: Bile
Yes- this is a suitable source
Indicate whether the following clinical specimens are suitable or unsuitable for anaerobic culture: Blood
No- this is not a suitable
Indicate whether the following clinical specimens are suitable or unsuitable for anaerobic culture: Coughed (expectorated) sputum
Yes- this is a suitable source
Indicate whether the following clinical specimens are suitable or unsuitable for anaerobic culture: Endometrium
No- this is not a suitable
Indicate whether the following clinical specimens are suitable or unsuitable for anaerobic culture: Feces
No- this is not a suitable
Indicate whether the following clinical specimens are suitable or unsuitable for anaerobic culture: Throat swab
Yes- this is a suitable source
Indicate whether the following clinical specimens are suitable or unsuitable for anaerobic culture: Tissue biopsy
STORCH
ORGANISMS that can INFECT a DEVELOPING FETUS Syphilis (bacterial) Toxoplasmosis (viral) Other Rubella/measles (viral) Cytomegalovirus Herpes Simplex virus (Other): Chickenpox (viral) Zika (viral) Liseriosis (bacterial) Lyme disease (bacterial)
Who is most likely to get Cryptococus neoformans?
Occur more commonly in immune compromised populations (HIV/AIDS )
What temperature does nitrogen fixation occur at? What about in industrial processes?
Occurs at ambient temperature. In industrial processes occurs ~600°C.
How quick is a secondary response to a pathogen?
It is quicker than a primary response. B memory cells are able to divide rapidly.
How quick is a primary response to a pathogen?
It is slow. This occurs the first time you're exposed to a pathogen or when you get vaccinated, only one B cell for a particular antigen.
What type of rash is Chicken Pox and Shingles ?
Large Pustular
What type of rash is Cutaneous Anthrax?
Large Pustular
What type of rash is Leishmaniasis?
Large Pustular
What type of rash is Smallpox?
Large Pustular
What type of rash is Staphylococcal Scalded Skin Syndrome?
Large Pustular
Streptococcal skin infections
M protein Protein G Protease Erysipelas Scarlet Fever
•
M. leprae (Hansen's disease - leprosy)
What is a secondary infection?
Occurs subsequent to primary infection (may involve opportunistic pathogens).
Neisseria gonorrhoeae
Match the organism with the appropriate term, disease or selective media. Choose the best answer: Thayer Martin
Bacteriophage structure
May contain a tail
•
Measles (paramyxovirus)
What is the agent in Measles
Measles virus
Another word for decomposition?
Mineralization.
•
Minimal inhibitory concentration (MIC)
What can Toxoplasmosis cause?
Miscarriage
•
Mode of action/target/symptoms of the following toxins:
•
Modification of known drugs (e.g vancomycin)
Attachment -Binding to host surfaces via adhesins such as
Molecules, structures such as fimbriae, capsules
MHC-1
On host cells and binds viral epitope so it can be recognized by CTL with T-cell receptors of a complementary shape, then apoptosis is induced by killer T cells
What is a virus associated with cancer called?
Oncogenic.
What percentage of cases of Roseola have the characteristic rash
Only about 30% of cases
Which types of hypersensitivity are considered delayed?
Only type IV is delayed. All other types are much more immediate.
What is the portal of entry for Cutaneous Anthrax?
Open skin wound or respiratory tract (worse version)
•
Opportunistic pathogens
bloodstream (as part of immune system)
whole blood consists of plasma and formed elements (RBCs and WBCs) plasma: water, dissolved substances, clotting factors serum: plasma without the clotting factors (fibrinogen has been removed)
A (the true intermediate host is sheep, humans get Echinococcus by accident - the definitive host is dogs)
When humans have cystic hydatid disease, the causative agent and host classification are A. Echinococcus granulosus - accidental intermediate host B. Echinococcus granulosus - definitive host C. Taenia solium - accidental intermediate host D. Taenia solium - definitive host
Microaerophilic
Which atmospheric condition is optimum for the cultivation of Campylobacter jejuni?
Microaerophilic environment
Which growth requirement is not appropriate for the cultivation of Haemophilus influenzae:
C (over 50% in the Southern states, likely linked to livestock) (A; think pinworms) (B; for sexually active people, but women more than men, think trichomonas) (D; think cryptosporidium) (E; immune compromise increases risk to all parasitic infections, not just ascarids)
Which population of the US has the highest prevalence of ascariasis? A. Infants B. Sexually active women C. Southern farmers D. Sewage workers E. HIV positive patients
Protistology
a scientific discipline devoted to the study of protists, a highly diverse group of eukaryotic organisms.
VZV travels along nerves of a single dermatome, so characteristically erupts along
a single stripe on one side of the body.
CHLORHEXIDINE
a surfectant and protein denaturant with broad micobicidal properties -skin degerming agents for preperative scrubs, skin cleaning, and burns
Pseudopodia
a temporary protrusion of the surface of an amoeboid cell for movement and feeding.
major role of cell mediated system
attach to infected cells (infected by bacteria or viruses) and destroy them
What is the vaccine for Yellow Fever?
attenuated, given to those traveling in areas where virus is common
What are the symptoms for Infectious Mononucleosis?
atypical lymphocytes Direct, indirect contact Extreme fatigue Rare complication: spleen rupture
Why is genotype and phenotype important?
bacteria only have one chromosome, so pheno- and genotype will align much more closely
What type of infection is Keratitis?
bacterial
What type of infection is Haemophilus influenzae?
bacterial meningitis
How does conjugation get plasmids or other genetic material across?
bacterial sex pilus or direct contact
gut typical resident
bacteroides
Solution to problem of VRE
banning use of avoparcin in animal feed
0.1% stain blue, *release histamine* granulocytes named according to microscopic appearance, presence of granules, type of stain etc.
basophils
after binding to their appropriate MHC, cells __
become activated
mutualism
benefits both organisms
viral size
between 20 nm to 14,000nm -0.02 um to 14um
Nonnucleoside Reverse Transeciptase Inhibitor
bind directly to RT and inhibit reverse transcription Both NRTI and NNRTI induce host level toxicity
antibody structure
binding site: top of purple area
Fc end:
binds to host cells
pathogen goes through part of life cycle in vector
biological
UTI-Cystitis (disease of urinary)
bladder infections; pain in pubic area, frequent urges to urinate when bladder is empty, burning pain with urinating, cloudy, colored urine, fever, nausea, back pain-indicates kidneys may be involved. treated w antibiotics
Indirect effects: T cells of immune system kill infected viral cells to _______ replication of viruses
block
what fluids should be sterile
blood and cerebral spinal fluid
platelets respond to physical roughness, release factors that lead to what
blood clotting
monocytes
blood phagocytes that rapidly leave the circulation; mature into macrophages and dendritic cells
What is the diagnosis for Infectious Mononucleosis?
blood sample ELISA
Modes of action- The cell wall
cell wall becomes fragile (cross walls are broken) and cell lyses
Activation of non-specific and specific defenses
cells killed by host
2nd line of defense
cellular and chemical system that comes immediately into play if infectious agents make it past the surface defense nonspecific (inflammation, fever, interferons, phagocytossis, complement, NK cells)
blood acts as a
cellular defense
portals of entry
characterist route a microbe follows to enter the tissue of the body
antiseptic
chemicals applied to body surfaces to destroy or inhibit vegatative pathogens
What is chemotherapy?
chemicals therapies used to treat a disease
Shingles is recurrence of previous _________
chickenpox
Haemophilus influenzae meningitis (disease of nervous)
children under 5; spreads from respiratory tract to bloodstream to meninges; gram - bacilli
Inhibition of protein synthesis
chloramphenicol,Aminoglycosides,tetracyclines, erythromycin, streptomycin, macrolids
Which disease is an acute diarrheal illness?
cholera
Which disease is an enterotoxin?
cholera
T cells continually replaced, continually destroyed Eventually, host loses is an example of
chronic infection
Foraminifera
class of amoeboid protists that produces a CaCo3 shell. The shells contribute to make landmarks such as the Pink Sands of Bermuda, the While Cliffs of Dover, and the stones that make up the Great Pyramids of Egypt.
molecule undergoes shape change upon binding to antigen
classic lock and key like an enzyme
sanitization
cleansing technique that removes microorganisms and debris from inanimate objects
degermination
cleansing technique that removes microorganisms and debris from living tissues
What is the recommended treatment for Rubella in adults?
clears on its own
platelets involved in what
clotting
serum contains all blood proteins EXCEPT for
clotting factors
What is the shape of Haemophilus influenzae?
coccobacilli (sphere/ rod)
arrangement of the nucleotides; three nucleotides determine the amino acid
codons
What is the diagnosis for ringworm?
culture & microscopy; KOH staining
How do you diagnose Septicemia?
culture microbes in aerobic vs anaerobic environments, antibiotic resistance, etc.
What is the diagnosis Cutaneous Anthrax?
culture on blood agar (Rod Shaped bacteria), clinical
T cells release *what* (signal proteins) that call and activate macrophages which aid attack.
cytokines
*Disease*
damage or injury to the host that impairs host function
resident flora
flora = microbiota natural communites of microbes that become establish in and on us includew mostly bacteria, archae, and fungi, some are pathogenic harmonius relationships w healthy people
histoplasmosis
fungus that affects the respiratory system, found in the Ohio and Mississippi river valley. called "cave disease"
microbial control methods- chemical agents
gases and liquids- antisepsis, disinfection, and sterilization
Porphyromonas
genera of bactoidetes that causes periodontal disease
Borrelia
genera of spirochetes, gram-negative, known for swimming, causes lyme disease.
Trepenoma
genera of spirochetes, gram-negative, known for swimming, causes syphilis
What are the two types of transduction?
general specialized
Fertility plasmids have all the _____
genes
segment of DNA that code for functional proteins/products
genes
Genome
genetic information of cell -Eukaryotes—all the chromosomes -Bacteria—chromosome and plasmids
Genotype
genetic makeup
science of heredity
genetics
genetic information of cell
genome
In Hfr fertility plasmid (F+) integrates into the _____ . It will only conjugate with _____.
genome; F-
genetic makeup
genotype
Rhizobium
genus of alphaproteobacteria that can fix nitrogen
Neisseria
genus of betaproteobacteria, known for it's pilli it uses to attach to cells
Myxoccus xanthus
genus of delta proteobacteria, can aggregate to form colonies covered in polysaccharides. They do not cause infection. Known as the social bacterium.
Campylobacter
genus of epsilonproteobacteria, lives in chickens and causes GI tract infections
Helicobacter
genus of epsilonproteobacteria, lives in stomach, acidophile
vibrio cholerae
genus of gammaproteobacteria, causes chlorea, plague, food poisoning
Pseudomonas aeruginosa
genus of gammaproteobacteria, causes pneuomnia in humans with cystic fibrosis, colonizes in burn wounds
E. coli
genus of gammaproteobacteria, found in the environment, foods, and intestines of people and animals
Yersinia
genus of gammaproteobacteria, gram-negative, causes food poisining (yersiniosis) after eating pork contaminated with versinia
Salmonella
genus of gammaproteobacteria, infection from this bacteria is usually caused by eating raw or undercooked meat, poultry, eggs or egg products
newborn indigenous flora
germ free prior to birth in uterus first exposure after breaking of fetal membranes initial exposure is primarily from mother
reservoirs are where ________ live
germs live
common vehicle transmission
getting food poisoning from contaminated food
mannoproteins
glucose proteins covered in sugar mannose on the outer edge of the fungal cell wall
M-tropic
gp120 adsorbs to CD4 and CCR5 of CD4+ cells Spreads infection, slower replicating
HIV Adsorption
gp120 adsorbs to CD4 and chemokine receptor -Binds to CD4 and induces a conformational change -Interacts with chemokine receptor and another conf. change occurs -Exposes fusion peptide
HIV Entry
gp41 is exposed so viral envelope fuses with host membrane Genome containing viral core enters host cell's cytoplasm
desiccation (drying)
gradual removal of water from cells, leads to metabolic inhibitation -generally not effective for microbial control
Bacterial Meningitis (disease of nervous system)
gram +, high fever; inflammation of the meninges (outer layer of brain ), sharp headache, vomiting , neck pain, loss of brain functions; progression to encephalitis (infection of brain)
Bacteroides
gram-negative bacteria, make up most of our gut bacteria, anaerobic
broad spectrum
greatest range of activity
Reservoir indicates where population of microbes is normally ________, not where it is acquired from (e.g. doorknob)
growing
attach to short chains of sugars
haemagglutinin
microbial death
hard to detect, microbes often reveal no conspicuous vital signs to begin with unlike the human body. Permanent loss of reproductive capability
The more immune individuals, the (*harder/easier*) it is for the disease to be spread among many people; the cycle of transmission is broken.
harder
know what is biofilm and why is it beneficial for the microbes to grow in the biofilm
layer of slime and bacteria that grows on a solid surface -beneficial because antibiotics won't penetrate
Droplet transmission:
less than 1 meter through air
What are the symptoms for Conjunctivits?
pain, pus production, redness, sensitivity to light
epidemic spreads worldwide
pandemic
Disease-causing bacteria; to them, we're dinner. is considered as
parasitism
one organism benefits at the other's expense; Classically, a "parasite" lives in or on host.
parasitism
indirect contact
passes from infected host to intermediate conveyor (fomites, food, aerosols, droplet nuclei) and then to another host; can occur in the absence of the infected individual/animal
Lyme Disease (disease of circulatory)
nonfatal, Borrelia burgdorferi, spirochete, G-; can be lifelong if not treated in time, transmitted by ticks, fever, headache and bulls eye rash
flora of GU tract
normal flora in external genitalia, vagina, and anterior urethra sterile in urine in bladder, posterior urethra, uterus
microbes normally found on the body
normal microbiota
main cause of viral diarrhea Outbreaks on cruiseships make news
norovirus
Limitations of antibiotics
not acid resistant doesn't work well against gram - allergenic location of infection resistance
Portals that do not provide access to susceptible tissue will
not lead to infection
mechanical vector
not necessary to the life cycle of an infectious agent and merely transports it without being infected
Infectious disease can be Non-communicable:
not spread from one host to another. -----Examples: your infected appendix bursts -----You get tetanus from "rusty nail"
potentially harmful diseases which must be reported to the CDC or other Health Unit by physicians
notifiable disease
morbidity
number of people afflicted with a certain disease
Requires a host cell to reproduce
obligate intracellular parasite
The major common cold virus, Rhinovirus, is specific to humans and like all viruses is an __________ __________ ________
obligate intracellular parasite
viral replication - lysogenic convergence
occurs when the infecting phage changes something in the host bacterium's life cycle ex: botulinum toxin
'Swine flu': strain; why do older people have some immunity against it while young people do not
older people have been in contact with it before. -seasonal flu is worse because immune system is weakened
parasitism
one org benefits, one is harmed
Antiobiotic-associated diarrhea
passing loose, watery stools three or more times a day after taking medications used to treat bacterial infections (antibiotics).
causes disease
pathogen
Non-infectious bacteria have low *what*
pathogenicity
What is Clavulanic acid and what does it inhibit? What kind of inhibitor is it?
penicillinase inhibitor - ** inhibitor
Inhibition of cell wall synthesis
penicillins, cephalosporins, bacitracin, vancomycin, Monobactams
Tuberculosis: Signs/symptoms of acute respiratory infections
persistent cough with blood constant fatigue loss of appetite and weight loss fever and night sweats
neutrophils
phagocytes active engulfers and killers of bacteria
chemotaxis and ingestion
phagocytes migrate and recognize PAMPs binding can be enhanced/mediated by opsonization of bacteria
action of cilia
propel mucus and microbes toward GI tract or to where they can be coughed out
skin typical resident
staphylococcus (oily areas) corynebacterium (moist areas)
true or false Mouth is full, fewer in esophagus and stomach; toward end of small intestine, numbers increase greatly.
true
true or false The portal of exit is often the same or connected to a portal of entry
true
true or false you swallow saliva which goes to the stomach and thats not a happy place fro microbes to be
true
true or false: relatively few bacteria cause disease
true
for shingles most cases occur along the ________
trunk
Basidiomycota
typically filamentous fungi composed of hyphae. Most species reproduce sexually with a club-shaped spore-bearing organ (basidium) that usually produces four sexual spores (basidiospores).
naive t helper cell
undifferentiated t cell with specificity to antigen
New influenza antigenic type leaves population *what*
unprotected
What is the portal of entry for Neisseria meningitidis?
upper respiratory tract, penetrates meninges
Urinary tract infections (disease of Urinary)
urine is good growth for many microorganisms
secondary role of cell mediated system
use cytokines to summon macrophages to destroy cells.
prophylaxis
use of a drug to prevent potential for infection of a person at risk
virulence factors that help an organism survive attack from phagocytes *antiphagocytic factors*
used to avoid death by phagocytosis
cold
used to preserve food, media, and cultures -several food poisoning traced back to refridgerated foods (lysteria moncytogenes)
bioremidiation
using bacteria to clean contaminants (oil spills)
Dry heat
using higher temperature than moist heat -incineration; flame or electric heating coil
bacterial endospores most resistant
usually require temperature above boiling
How are emerging infectious diseases different from 'regular' diseases?
vaccines, antibiotics, regularly available, common for things like strep, flu, chicken pox -emerging infectious diseases are unknown, don't know how to treat them
exit from sex by
vaginal fluid, semen
predormal stage
vague feelings of discomfort; nonspecific complaints
What is the treatment for Streptococcus pneumoniae?
vancomycin (antibiotic)
What does VRE stand for?
vancomycin-resistant enterococci
Chicken Pox/ Shingles (skin disease)
varicella-zoster virus; rash crusts over and heals; nerve pain, slight fever; virus travels down nerve ganglia
microbes colonize on the skin but what else happens
various factors keep population down
secondary immunodeficiency diseases
various immune deficiencies acquired after birth and caused by natural processes or artificial agents
primary immunodeficiency diseases
various immune deficiencies present at birth and usually stemming from genetic errors
If people get anthrax from inhaling B. anthracis endospores wafted into the air from a drone, that would be *what type of transmission?*
vehicle
Rabies (disease of nervous system)
very fatal; headache, fever, partial paralysis near bite site. throat muscles undergo painful spasms, hallucinations. virus attacks nervous system, once it hits the brain theres nothing you can do. virus spreads through saliva and is gets helped being spread by aggressiveness (hints: biting, drooling)
spirochetes
very hard to treat since flagellum is INSIDE the periplasm. Known as stealth pathogens.
6)
What is the similarity between the flora found in a female genitals at the menopause stage and a female at the pre-puberty stage? How different is it from a female between the stages of puberty and menopause?
The act of converting inorganic carbon to organic carbon.
Carbon fixation.
What are the two forms of TB?
1. Latent tuberculosis infection 2. Active tuberculosis infectioin
HAEMOPHILUS
1. MENINGITIS 2. PNEUMONIA
HIV
2 molecules of single stranded RNA
Rubella
"German Measles" Virus
Whats the nickname for Impetigo?
"School sores"
mother to offspring, e.g. vectors, Hep B
"vertical"
Cat-scratch fever
(Bartonella sp.) Transmitted by cats (kittens), children most susceptible Gram negative
Brucellosis
(Brucella sp.). Muscle aches, fever Gram negative, aerobic Unpasteurized milk
Certain T cells activated, become T-cytotoxic cells referred to as
(CD8 cells)
What is HIV?
(Human immunodeficiency virus) Retrovirus Causes destruction of white blood cells
Rocky Mountain Spotted Fever
(Rickettsia) Transmitted by ticks Gram negative, rods
how do you get Hep B
(sex/blood transmission)
RINGWORM - TRICHOPHYTON, MICROSPORUM, EPIDERMIPHYTON
*1. Corporis- of the body *2. Cruris- groin *3. Capitis- head *4. Pedis- foot (athletes foot) *5. Manuum-hand *6. Barbae- beard *7. Unguium- nails
The use of Chemical Controls
*Alcohols* ----Denature proteins and dissolve membrane lipids *Heavy metals* ----e.g., ions of mercury, silver, arsenic, zinc, and copper ----Effective but usually toxic (i.e., copper sulfate) *Halogens* ----E.g. iodine, chlorine ----Oxidizes cell constituents
The use of Chemical Controls
*Alcohols* ----Denature proteins and dissolve membrane lipids *Heavy metals* ----e.g., ions of mercury, silver, arsenic, zinc, and copper ----Effective but usually toxic (i.e., copper sulfate) *Halogens* E.g. iodine, chlorine Oxidizes cell constituents
Antibiotic Treatment
*Antibiotics treatment* can alternate the normal microbial communities ---> *Antibiotics-resistant microorganisms:* Staphylococcus aureus, Proteus, Clostridium difficile or Candida albicans *can opportunistically take over without normal competition*.
Clotting mechanism
*Clotting mechanism are often used by host to limit the spread of a pathogenic infection* ------*Fibrinolytic enzymes* - (e.g. streptokinase Streptococcus pyogenes) - dissolve the fibrin clots -----*Fibrin promoting enzymes* - (coagulase - Stapylococcus aureus) - *promote the formation of fibrin clots* and protects the pathogens by host
Antifungal Drugs
*Ergosterol inhibitors* target the unique fungal plasma membrane component ergosterol -----*Polyene antibiotics* - binds specifically to ergosterol causing membrane permeability and cell death ------*Azoles and allylamines *- inhibit ergosterol biosynthesis *Echinocandins* - cell wall inhibitors ----*Inhibit 1,3 β-D glucan synthase* (forms β- glucan polymers) -----Used to treat *Candida* infections -Other drugs target chitin biosynthesis, target folate biosynthesis, or disrupt microtubule aggregation -Antifungal-resistant fungi are emerging
Difference between exotoxins and endotoxins
*Exotoxins* - proteins released from the pathogen ----> It can be divided into 3 categories: 1) Cytolytic toxins (Cytotoxins) 2) AB toxins 3) Superantigen toxins *Endotoxins* - toxic component of gram-negative cell walls (not secreted) ---> affects the small intestine
Adherence by Extracellular Macromolecules
*Extracellular macromolecules* that are not covalently attached to the bacterial cell surface (*glycocalyx*) ---E.g. slime layer, capsule function in attachment, but also for protection against host defense ---*Opacity associated protein* in Neisseria gonorrhoeae - attach to epithelial cells ---*Hemagglutinin* helps influenza virus attach to lung epithelial cells *Fimbriae and pili*
Antifungal Drugs
*Fungi pose special problems for chemotherapy because they are eukaryotic* --Much of the cellular machinery is the same as that of animals and humans --As a result, many antifungals are topical --A few drugs target unique fungal metabolic processes
The use of Physical Controls
*Heat sterilization* ---Most widely used method of controlling microbes ---Endospores *Autoclave* -- >100ºC *Radiation* ---Microwaves, UV, X-rays, gamma rays, and electrons ---Sterilization in the medical field and food industry ---Approved by the WHO *Filtration*
Different stages of pathogenic infections, from colonization to disease
*Infection* - situation in which a microorganism is established and growing in a host, whether or not the host is harmed *Disease* - damage or injury to the host that impairs host function
pathogens
*Microbial parasites* *Microbes that cause disease* *Pathogenicity* - the ability of a parasite to inflict damage on the host *Virulence* - measure of pathogenicity *Opportunistic pathogen* Causes disease only in the absence of normal host resistance (e.g cancer or AIDS)
28.2 Healthcare-Associated Infections
*Nosocomial infection* -----Infection acquired at a healthcare facility -----1.7 million patients acquire HAI (1/20 patients) ----HAI are acquired from patients, hospital personnel etc. Table 27.3 risk factors
What are the diagnostic tests? Give an example of each.
*Phenotypic- features of virus/bacteria; (ex: Colony morphology) *Genotypic- looking at genetic markers and whether genes are absent or present (ex: Genotyping) *Immunological- use patient's antibodies to detect pathogen (ex: Rapid Strep Test)
Fusion Inhibitors
*Prevent viruses from successfully fusing with the host cell * E.g. enfuvirtide Blocks gp41 membrane receptors on T lymphocytes and stop the virus from entering the cells Therapy cost ~ $25,000/year
antibody-mediated immunity
*Primary response* -1st exposure to Ag -Long lag time (AMI process): weeks -Slow to protect first Ab class produced is IgM, followed by IgG -Memory B cells produced against Ag *anamnestic response* -reexposure to same Ag4-response in hours: IgG Ab class produced, then IgM -Ab response is stronger with reexposure
process of cell mediated immunity
*Primary response* -1st exposure to Ag -Long lag time for CMI response (days-wks) -Slow build-up of activated T cells -Memory T cells produced (clones) *secondary/enhanced response* -Re-exposure to Ag -Short lag time for CMI response (immediate) -Rapid build-up of activated T cells from memory cells
Probiotics
*Probiotics* - live culture of intestinal bacteria -*Recolonize normal gut flora* Evidence of *short-term affect* by probiotics, but little or *no long-term affect*
Antimicrobial Drug Susceptibility Testing
*Selection of antibiotics for microbial therapy* must be made on case-by-case bases *Minimum inhibitory concentration (MIC)* procedure for antibiotic susceptibility testing employs an antibiotic dilution assay ----Wells containing serial dilutions of antibiotics are inoculated with a standard amount of a test organism ----Look for inhibited growth (turbidity)
•Groups of microbes producing endotoxins
*The lipopolysaccharide (LPS) portion of the cell envelope of gram-negative Bacteria* -Released when the cell lyse Generally less toxic than exotoxins, but large doses can cause septic shock, hemorrhagic shock and tissue necrosis -Present in e.g. E. coli, Shigella, and Salmonella
Microhabitat variations in microbiome
*The skin surface varies greatly in chemical composition and moisture content* *Three microenvironments* (1) Dry skin (2) Moist skin (3) Sebaceous skin Skin microflora examined by molecular ecology methods ---19 phyla detected ---Each microenvironment shows a unique microbiota
Habitats in the GI tract (e.g. variations in pH, O2-levels etc between different parts of GI-tract)
*acidity of the stomach* and the *duodenum of the small intestine (~pH 2)* prevents many organisms from colonizing the GI tract
Effect of Antimicrobial agents
*icides:* agents which kill microbes ---->E.g., germicides: kill pathogens ---->Often effective against only a certain subset of microbes -------->E.g., algicides, fungicides, bactericides, etc... -*statics:* agents which prevent growth -------->If removed, growth continues -------->E.g., bacteriostatic, fungistatic, etc...
•Growth-dependent identification methods
*oxidase test:* colonies turn blue if contain cytochrome c
•Penicillins Vs. Cephalosporins (e.g. β-Lactaases and resistance)
*β-Lactam Antibiotics* -One of the most important groups of antibiotics of all time -*Penicillins* and *cephalosporins* Cell wall synthesis inhibitors Over half of all antibiotics used worldwide Allergies *Penicillin* -Effective primarily against gram-positive bacteria ---gram negative are impermeable -Semi-synthetic forms are effective against some gram-negative bacteria *Cephalosporins* - Produced by fungus Cephalosporium -Same mode of action as the penicillins -*Broader spectrum of activity than penicillins* -Resistant to *β-lactamases* ------Dihydrothiazine ring E.g. Ceftriaxone ----Commonly used to treat gonorrhea (Neisseria gonorrhoeae)
VRE: - Gram -/+ - Communal or HAI? - Where is it found in normal flora? - What infections does it cause?
- + - HAI - colon, female genital tract, skin - UTI, bacteremia, endocarditis
IgA (dimer only
- 4 (dimer); 2 (monomer) binding sites -percentage in serum: 13% -cant cross placenta -cant fix complement -secretory antibody;; on mucous membrane
Isocahedral virus morphology
- A 20-sided polyhedron
What is the difference between an antimicrobial and antibiotic?
- Antimicrobial: inhibits/kill bacteria; simple structures - Antibiotic: antimicrobial that is produced by another microorganisms; complex structure
Where is Polymyxin B used and what does it work against? ** How do you use it?
- Cell Membrane - Gm - ** Triple antibiotic ointment
Why is horizontal gene transfer important?
- Diversity- changes geno- and phenotype - Medically: virulence factor (transformation), can create antibiotic resistance (conjugation), toxin genes (transduction)
What should we do about resistance?
- Education - Improve immune system with other treatments - Decrease use
Natural mechanisms of resistance
- Enzymes break down antibiotics - Efflux pumps - Mutation (permanent, change structure) - Block entry (change surface)
First Line of defense of the Skin
- Epithelial cells
What is monobactams? What does it work against? What kind of antibiotic is it?
- Gm- - low toxicity - affects cell wall
Know the surface antigens for serotyping
- Haemagglutinin (HA) involved in virus entry to the cell - Neuraminidase (NA) invoved in virus release from the cell
b lymphocytes (b cells)
-involved in antibody mediated immunity -activated b cell produces specific antibodies to destroy a specific antigen
What are common mediums for growing viruses?
- Human host - Embryonated eggs (ex: chicken embryo) - Or in eukaryotic cell tissue culture
- Treatment of cholera
- IV fluid, ORT (Oral Rehydration Therapy),Vaccination
What does Isoniazid (INH) inhibit? What would you use it for and what is it good for?
- Inhibits mycolic acid sythesi - use for cell wall - good penetration ability
Helical virus morphology
- Length not fixed -- open structure - Any volume enclosed by varying length - Evolve without constraint of fixed coat
Treatment for TB? How long?
- Multiple drug therapy - 6 mths - 1 year
The speed of DNA replication is 40 minutes to copy the genome in E. coli. How can E. coli have a generation time of 20 minutes? How does this differ in a eukaryote?
- No steps; can have multiple chromosomes replicating at once- every time origin is open - has steps, have to finish first step before moving on to the next
Where is Rifamycins used and what does it inhibit? What does it penetrate and what kinds of side effects does it have?
- Nucleic Acid Synthesis -Inhibits synthesis of mRNA -Penetrates tissue -Enters CNS -Side effect •Red urine, feces, saliva, tears, and sweat
How is resistance being selected for?
- Overuse (one kind too much) - Misuse (not following prescription, using for virus, demand, wrong medication, sub-therapeutic: long term low dose) - Horizontal Gene Transfer - Travel
Where is Chloramphenicol used and what does it inhibit?
- Protein Synthesis - Inhibits peptide bond formation
- DNA -> DNA - DNA -> RNA - RNA -> Protien
- Replication - Transcription - Translation
What two characteristics make antibiotic effective treatments?
- Selectively toxic - Target something essential
Importance of Understanding Heredity
- To know flow of genetic information from genes to proteins - To solve or understand problems with antibiotic resistance - How emerging disease arise
Unknown capsid morphology
- Unknown symmetry - Neither polyhedral nor helical
Mycobacterium tuberculosis is an _____ _____ bacterium which means it contains _____ acid.
- acid fast - mycolic
Where is the genetic information found in eukaryotes? Bacteria?
- all the chromosomes - chromosome and plasmid
Mycobacterium tuberculosis grows in _____ which cause _____ in lungs. What may appear, why?
- alveoli, tubercula - small lumps; body forms granulomas to defend
Treatment of VRE
- antibiotic sensitivity test (see where there is resistance) - Linezolid (protein synthesis inhibitor) - Telavancin (synthetic glycopeptide)
Characteristics of Antibiotic: - ________ compound - Where do they come from? - Activity: nonspecific/specific - May have _____ effects - _____ exists - _____ vs. _____ spectrum
- antimicrobial - other microbes (natural products) - specific - side - resistance - broad, narrow
How would you diagnose Haemophilus influenzae?
- blood agar or chocolate agar (sphere/rod) -Facultative anaerobe -sample CSF
Vibrio cholerae: - Where is it found? - Infectious dose high/low? Is it lower in food or water, why? - Resistant to acid: yes/no?
- brackish water - high, food, food protects it from stomach acid - no
Why would it be important to understand the mutation rate?
- could potentially give rise to something new - mutations can be beneficial, not just bad
How do you die from cholera?
- dehydration, electrolyte imbalance
What does Vancomycin effect and what does it work against? When would you use Vancomycin?
- effective against penicillinase - works for cell wall - toxicity problems-- limit use - LAST LINE ANTIBIOTIC FOR RESISTANT
What kind of antibiotic is bacitracin and what does it work against? What does it affect? and what does it cause? ** how do you use it?
- effects against Gm + - causes nephrotoxicity (toxicity in the kidneys) - polypeptide antibiotic/ works against cell wall ** Triple antibiotic ointment
Who are the people most likely to have an active tuberculosis infection?
- elderly and children - immunocompromised - smokers - people who live in crowded conditions
Order easiest to hardest to treat with antibiotics and why: fungi, gram negative, gram positive, virus
- gram positive: simple; no outer membrane - gram negative: outer membrane - fungi: eukaryotes - virus: use our cells to replicate
What is the problem with targeting the cell membrane with antibiotics? So how can we use it?
- low selective toxicity because our cell membrane is the same as a bacteria - topically
Specialized transduction: - _____ virus - Virus enters host cells and enters into host's _____ (prophage) - What happens if the host cell becomes damaged? - What is the result?
- lysogenic - genome - virus will come back out and will begin to replicate (general transduction/lytic cycle) and kill the host cell (get new deformed virus) - deformed- will have viral and host DNA
t lymphocytes (t cells)
-involved in cell mediated immunity -cells mature in thymus gland -cell to cell defense -also have a role in antibody mediated immunity
Generalized transduction: - _____ virus (what does this mean) - uses _____ off host to get enough nucleotides for virus DNA - How come it makes it a defective virus?
- lytic (get in, replicate, kill cell, move on to the next) - chromosomes - carries host DNA to the new bacterium, NO viral DNA
Eukaryotes: - multiple/single chromosome - diploid+ or haploid - circular/linear chromosomes - DNA gyrase/Histone packaging - Attached to plasma membrane/Found in the the nucleus
- multiple - diploid or greater - linear - histone - nucleus
Which is broad spectrum and which is narrow spectrum? - targets only specific organisms - works on a wide range of bacteria (gram + and gram -)
- narrow - broad
How does DNA know how to begin? When proteins bind to this site it opens up DNA, which allows for _____ _____ to enter. They move around in a circle until they reach the what?
- origin of replication - replication fork - termination of replication
Bacteria: - multiple/single chromosome - diploid+ or haploid - circular/linear chromosomes - DNA gyrase/Histone packaging - Attached to plasma membrane/Found in the the nucleus
- single - haploid - circular - DNA gyrase - Attached to plasma membrane
Codons: - Wobble: _____ nucleotide is least important - How many sense codes for amino acids? Why are there more codes than amino acids? - How many nonsense? What do they code for? - What is the start codon?
- third - 61, allows for mistakes - 3 stop codons - AUG (methionine)
Bacteria can have simultaneous _____ and _____. What is the benefit? Down-side?
- transcription and translation - faster; can't control environment so need to respond quickly to environment - costly (lots of energy
What does cephalosporis work against and how is it taken?
- works against cell wall - injected
Invassiveness
-*Ability of a pathogen to grow in host tissue at densities that inhibit host function* ---->Can cause damage without producing a toxin -Many pathogens use a combination of toxins, invasiveness, and other virulence factors to *enhance pathogenicity*
•Major structural groups (aminoglycosides, macrolides etc.)
-*Aminoglycosides* are antibiotics that contain amino sugars bonded by glycosidic linkage -Target the 30S ribosomal subunit, *inhibiting protein synthesis* ------e.g. kanamycin, neomycin, amikacin, streptomycin -*Not commonly used today* ------Used when other antibiotics fail (e.g. semisynthetic penicillins) *Macrolides* contain *lactone rings bonded to sugars* --Broad-spectrum antibiotic that targets the 50S subunit of ribosome ----Patients with penicillin allergy *Tetracyclines* contain four rings, *napthacene ring system* -*Widespread* use in humans and animals -Common antibiotics resistance -*Broad-spectrum inhibition of protein synthesis* -Inhibits functioning of 30S ribosomal subunit
Degree of Removal of Microorganism
-*Sterilization:* the complete removal or destruction of all microbes ---->Even spores and viruses are removed ---->Often use chemicals referred to as sterilants -*Disinfection:* the killing, inhibition or removal of disease causing microbes ---->Also reduces the total microbial population ---->Employ disinfectants: agents to disinfect objects -*Sanitization:* reduction of microbial population to levels deemed "acceptable" -*Antisepsis:* prevention of infection of living tissue by microorganisms ----->*Antiseptics:* chemical agents that kill or inhibit growth of microorganisms when applied to tissue
non-phospholipases
-*Streptolysin O* - (Streptococci pyogenes) affect the sterols of the cytoplasmic membranes -Lysogenic bacteriophage - phage conversion ---Cause pharyngitis and acute/chronic rheumatic heart disease Leukocidins - lyse white blood cells (leukocytes) α-toxin - (Staphylococcus aureus) kills nucleated cells and lyses erythrocytes Key virulence factors in S. aureus pneumonia Ongoing development of vaccination with attenuated strains of S. aureus
Urinary tract infections
----->Escherichia coli, Proteus mirabilis and Staphylococcus saprophyticus *frequently cause urinary tract infections (UTI)* ----->*Viral and fungal infections* can also cause UTI ----->10% of females develop UTI yearly
Relationship between Lactobacillus acidophilus and estrogen levels
----Lactobacillus acidophilus, a resident organism in the vagina, ferments the glycogen, producing lactic acid ----Lactic acid maintains a local acidic environment ----L. acidophilus is present absent puberty and after menopause *(glycogen- estrogen relationship)*
*Infection*
-->situation in which a microorganism is established and growing in a host, whether or not the host is harmed
IgM
-10 binding sites (pentamer) -percentage in serum: 6% -cant cross placenta -can fix complement -produced at first response to antigen, can serve as a b-cell receptor
IgE
-2 binding sites -percentage in serum: 0.002% -cant cross placenta -cant fix complement -binds to mast cells and basophils -antibody of allergy, worm infection
IgD
-2 binding sites -percentage in serum: 1% -cant cross placenta -cant fix complement -receptor on B cells
IgG
-2 binding sites -percentage in serum: 80% -can cross the placenta -can fix complement -long term immunity; memorizes antibody; neutralizes toxins, opsonizes, fixes comlement -binds to phagocytes
*HIV structure PRINT PICTURE
-2 identical (+) strand ssRNA molecules enclosed in a capsid -Envelope with gp120 binding sites->cd 4 receptor sites -T4 helper cells same in body -Enzymes: Reverse Transcriptase (RT), Integrase, and Protease
categories and types of biological agents - know an example of each
-3 categories-A,B,C -A: high priority and mortality, require special action (Bacillus anthracis) -B: low mortality, more economic than public health impact (ricin) -C: unknown-emerging pathogens, potentially easy to disseminate, potential high mortality rate/public health threat (Zika) -bacterial agents: Bacillus anthracis (anthrax) (A) -viral agents: Variola virus (smallpox) (A) -toxins: ricin (B)
Measurement of virulence (e.g. LD-50)
->Virulence can be estimated from experimental studies of the *LD50 (lethal dose50)* -------->The amount of an agent that kills 50% of the animals in a test group ->*Highly virulent pathogens* show little difference in the number of cells required to kill 100% of the population as compared to 50% of the population
Measuring Virulence
->Virulence can be estimated from experimental studies of the *LD50 (lethal dose50)* -------->The amount of an agent that kills 50% of the animals in a test group ->*Highly virulent pathogens* show little difference in the number of cells required to kill 100% of the population as compared to 50% of the population
Biofilm formation (e.g. dental plaque)
-Attachment and growth have been well studied in the formation of *biofilms on tooth surfaces* -Acidic glycoproteins from saliva form a thin film (provide attachment site) -Streptococci colonize the film -S. sobrinus and S. mutans are common agents in tooth decay -Dextran - polysaccharide -Mixed assemblage
How do you diagnose Acanthamoeba?
-Brain biopsy (Removal of tissue for histological examination; to rule out cancer) -CFS
normally sterile body areas
-Brain; Central nervous system -Blood; Tissues; Organ systems -Bone and joints; muscle -Sinuses -Inner and Middle Ear -Lower Respiratory Tract: Larynx; Trachea; Bronchioles (bronchi); Lungs; Alveoli -Kidneys; Ureters; Urinary Bladder; Posterior Urethra -Cervix; Uterus; Fallopian Tubes
where is aminoglycosides used and what does it effect?
-Cause misreading -Effective against Gm- -Toxic side effects •Kidney damage •Auditory nerve damage - works against protein synthesis
Phenotypic Identification
-Colony morphology: What does the infectious agent look like/ shape? Colony -Biochemical: Does it have the presence of a specific enzyme?
•AB toxins
-Consist of two subunits, A and B -Protoxins -Work by binding to host cell receptor (B subunit) and transferring damaging agent (A subunit) across the cell membrane ---Examples: diphtheria toxin, tetanus toxin, botulinum toxin
Pauteurization
-Controlled heating temperature well below boiling -Reduces total microbial load and thereby increases shelf life of treated material -Keeps taste and nutrient values *Flash pasteurization* (high temperature short-term - HTST) -----72°C for 15 sec *Ultra high-temperature* (UHT) sterilization -----140-150°C for 1-3 sec
What is the diagnosis for Listeria monocytogenes?
-Culture methods are difficult -Grow best at 4 Celsius -ELISA -Immunofluorescence -DNA probe
How would you diagnose Neisseria meningitidis?
-Culture on chocolate agar (diplococcoal) -Oxidase test using CSF (positive)
Dental Plaque
-Extensive growth of oral microorganisms, results in a *thick bacterial layer (dental plaque)* -As plaque continues to develop, anaerobic bacterial species (e.g. Fusobacterium) begin to grow -As dental plaque accumulates, the microorganisms produce high concentrations of acid (especially *lactic acid*), resulting in *decalcification of the tooth enamel (dental caries)*
Where is Macrolides used and what is it good against?
-Gram+ only -Few side effects—children -Block tunnel
Contribution of human gut microorganisms to human health
-Human gut microorganisms synthesize: ---->*Enzymes* (digesting of complex carbohydrates) ---->*Amino acids* (essential) -Human gut microorganisms contribute to "maturing" of GI tract ---->*Induce expression of genes* involved in nutrient uptake and metabolism in gut epithelial cells ---->*Priming immune system* to recognize normal gut flora
Where is Quinolones used and what does it inhibit?
-Inhibits DNA gyrase - Nucleic Acid Synthesis
what does Ethambutol inhibit? What would you use it for? What kind of use is it?
-Inhibits incorporation of mycolic acid into cell wall - Use for cell wall -Used in combination to control resistance
Where is Tetracycline used and what does it interfere with? What are it's side effects?
-Interfere with tRNA - works against protein synthesis -Side effects ---Tooth discoloration in children ---Liver damage in pregnant women
How can you diagnose a Staph Infection?
-Isolation on blood agar (hemolysis- gold color) *Au-gold -Coagulase (positive/ solid)
•Biosafety level (BSL-1-4)
-Laboratories are classified according to their containment potential, or *biosafety level (BSL)*, and are designated as *BSL-1 through BSL-4* (see Table 27.2) -Most university labs have *BSL-1 and BSL-2 for teaching and research* -*BSL-3 and BSL-4 requires pressurized labs*
What is the recommended treatment for a staph infection?
-Methicilin (Use topical at first at localized area) -MRSA (Methicilin resistant staph aureus) use Amoxicillin *Use intravenous antibiotics is pathogen is getting close to blood stream
Antiviral Drugs
-Most antiviral drugs also *target host structures*, -------*Toxicity* (difficulties with selective toxicity) -Recent efforts to treat HIV have resulted in several new antiviral treatments --Most successful and commonly used antivirals are the *nucleoside analogs* (e.g. zidovudine/Azidothymidine - AZT) --Block reverse transcriptase and production of viral DNA (i.e. *nucleoside reverse transcriptase inhibitors)*
nitrogen cycle
-Nitrogen fixation - N2 to NH3 Cyanobacteria, Azotobacter -Nitrification - NH3 to NO3 Nitrosomonas, Nitrobacter -Denitrification - NO3 to N2 Denitrifying bacteria (Pseudomonas)
Toxicity
-Organism causes disease by means of a toxin that inhibits host cell function or kills host cells -Toxins can travel to sites within host not inhabited by pathogen *Exotoxins* - proteins released from the pathogen *Endotoxins* - toxic component of gram-negative cell walls (not secreted)
physical or anatomical barriers
-Outermost layer of skin is composed of epithelial cells compacted, cemented together, and impregnated with keratin; few pathogens can penetrate if intact -Sloughing of epidermis -Flushing effect of sweat glands, urine, feces -Mucous coat impedes attachment and entry of bacteria -Mucus traps microbes -nasal hair traps larger particles -cilia of trachea -ciliated cells move mucus to pharynx
28.1 Safety in the Microbiology Laboratory
-Present significant biological hazards for workers -Standard lab practices for handling clinical samples are in place to protect workers
how is the human benefited with microbes in the gut?
-Production of various nutrients including vitamins -Protection from pathogens -Stimulation and training of the immune system -Prevention of inflammatory diseases -Proper development of lining of GI tract -Digestion of remaining nutrients in colon
detergents and soaps
-Quaternary ammonia compounds (QUATS) -soaps- mechanically remove soil and grease containing microbes
Enzymes in Saliva (category and Name)
-Saliva contains *antimicrobial enzymes*: ---->*Lyzosymes* - cleaves the glycosidic linkages in peptidoglycan ---->*Lactoperoxidase* (secreted by mammary, salivary, and other mucosal glands) - initiate oxidative reactions that result in oxidized intermediates with bactericidal effects
Microbial diversity in the gastroinstestines
-Salmonella enterica and S. bongori -*Flagellated enterobacteria* found in animals and environment -Cause various diseases such as *gastrointestinal illnesses, typhoid fever, paratyphoid fever*
Microbial traits are controlled or influenced by heredity
-Shape -Metabolism -Structure -Ability to cause disease -Everything
physical/chemical barriers
-Sneeze reflex -Skin secretions: sebum and sweat have antimicrobial chemicals. -Lysozyme and Lactoferrin (tears, sweat, saliva, mucous, human milk): lysozyme destroys the cell wall of gram-pos. bacteria; lactoferrin has various antimicrobial affects. -Acidity of stomach (pH 2) due to HCl: ↓ in microbes (gastric juice kills most bacteria) -Vagina: microbiota and acidic pH (acids released by some flora). -Semen: antimicrobial chemicals. -Urine: urine flow in ureters and bladder -Bile and intestinal enzymes: help to destroy microbes
Virulence Factors that interfere with host defense
-Some virulent factors protect the pathogen by interfering with normal host defense mechanisms -Virulent factors can affect fibrin, a blood protein that forms clots -*Clotting mechanism are often used by host to limit the spread of a pathogenic infection* ------*Fibrinolytic enzymes* - (e.g. streptokinase Streptococcus pyogenes) - dissolve the fibrin clots -----*Fibrin promoting enzymes* - (coagulase - Stapylococcus aureus) - *promote the formation of fibrin clots* and protects the pathogens by host immune cells
sulfur cycle
-Sulfur oxidation (can be both chemical and biological process) - S2H to S0 to SO4 2 - -Sulfate reduction (only biological process) - SO4 2 - to S0 to S2H
Where is Fluoroquinolones used?
-Synthetic quinolones -Broad spectrum -Nontoxic -Ciprofloxacin--anthrax - Nucleic Acid Synthesis
Urogenital Tracts and Their Microbes
-The *bladder* is typically *sterile* -Altered conditions (e.g. pH) can cause potential pathogens in the urethra to multiply and become *pathogenic* ----->Escherichia coli, Proteus mirabilis and Staphylococcus saprophyticus *frequently cause urinary tract infections (UTI)* ----->*Viral and fungal infections* can also cause UTI ----->10% of females develop UTI yearly
Attenuation
-The *decrease or loss of virulence* -Attenuation often occurs in *lab strains* (i.e. virulence has no selective advantage); *possible to purposely select for less virulent strains* (e.g. mutagenesis of virulent genes) -Virulence may be *regained or permanently lost* -Attenuated strains can be *useful to produce vaccines*, (e.g. measles, mumps, rubella, and rabies)
The availability of nutrients is most important in affecting pathogen growth
-The availability of nutrients is most important in affecting pathogen growth ---Iron is an important growth-limiting micronutrient for bacteria ---Transferrin and lactoferrin are high-affinity iron-binders, which sequester iron in the host and limit infections -Pathogens can produce iron-chelating compounds (i.e. siderophores)
Oral Cavity: Common microbes/pathogens in lower respiratory tracts
-The lower respiratory tract *lacks microflora in healthy individuals* -*Ciliated epithelial cells* act as defense by removing particles -*Bacteria and virus* can cause infection in lower respiratory tracts and lungs (*pneumonia*) Pneumonia cause >4,000,000 deaths yearly
Microbial diversity in Oral Cavity
-The oral cavity is a *complex, heterogeneous microbial habitat* -High concentrations of nutrients -Saliva contains *antimicrobial enzymes*: -Lyzosymes - cleaves the glycosidic linkages in peptidoglycan -Lactoperoxidase (secreted by mammary, salivary, and other mucosal glands) - initiate oxidative reactions that result in oxidized intermediates with bactericidal effects
- what type of habitats and microbes
-The oral cavity is a *complex, heterogeneous microbial habitat* Most microorganisms are -*facultatively aerobic* ----Obligately anaerobic ----Obligately aerobic *Not all taxa are present and similarly distributed in all individuals*
How do epithelial cells act as the first line of defense of the skin?
-They *provide barriers* *-Secrete Mucus* (water soluble proteins, glycoproteins etc.) that inhibit microbial attachment) *-Physical Removal:* swallowing or sneezing
Virulence factors (enzymes, adherence molecules, toxins)
-Virulence factors are *molecules produced by a pathogens that enhance invasiveness:* ---->Attachment/ Adherence Molecules ---->Enzymes ---->Toxins -Enhance virulence by breaking down or altering host tissue to provide access to nutrients --*Hyaluronidase* break down hyaluronic acid --*Collagenase* - destroy *collagen* -Many other types of proteases, nucleases, lipases to degrade proteins, nucleic acids and lipids
clonal selection theory
-a theory that explains how first-time exposure to antigens are recognized by lymphocytes that leads to adaptive immunity -before birth, stem cells in bone marrow undergo a continuous series of divisions and genetic changes that generate millions of different T and B lymphocytes -each lymphocyte has a particular/unique receptor specificity to a particular epitope or antigen
what are viruses
-acellular -obligate intracellular parasites -no ATP generating system -no ribosomes of protein synthesis
dyes as antimicrobial agents
-aniline dyes (crystal violent/ malachite green) very active agent against gram positive species or bacteria and various fungi
animal viruses: enveloped
-attachment -penetration by fusion (viral envelope and cell membrane fuse together) -uncoating -biosynthesis (needs to make coat proteins and receptors) -assembly -release: by budding
animal viruses: non-enveloped
-attachment: binding sites must match the receptor sites on host cell -penetration: phagocytosis -uncoating: capsid/envelope are taken away -biosynthesis: genome replication, transcription, translation -assembly: virus particles are assembled -release : by lysis
cells involvedd in b cell response
-b cells with igD antibodies displayed on their b cells are also APCs -t helper 2 cells: its tcr/cd4 bind to b cells MCH II -plasma cells: activated b cells differentiate into plasma cells; plasma cells produce antibodies (igM and igG) -memory b cells
mechanisms of drug actions
-cell wall inhibitors; blocks synthesis and repairs -cell membrane;causes loss of selective permeability -DNA/RNA; inhibit replication and transcription, inhibits gyrase -protein synthesis inhibitors acting on ribosomes -metabolic pathways and products; block pathways and inhibit metabolisms
how to protect against it
-cover nose and mouth with tissue when coughing/sneezing, then throw it away -wash hands often w/soap and water -alcohol based cleaners are effected against developed viruses like influenza -avoid touching eyes, nose, mouth -avoid unnecessary traveling to areas w/high rate of infection
how influenza kills;
-deaths from Influenza are often caused by body's own immune system destroying the respiratory epithelium -secondary bacterial infections
How likely are you to get this disease or any emerging infectious disease? (Think about this one in the global context!)
-depends on where you are or what you are doing
growing viruses-phages
-easiest to grow because bacteria are easy to grow -lawn of bacteria on spread plate -add bacteriophages -infection results in 'plaques'-clear zones on the plates where bacteria have been killed by the viruses
what is the advantage of biological agents over chemical and nuclear weapons
-easily concealed -multiply rapidly -easy production
alcohols
-ethyl -isopropyl
gases and aersols
-ethylene oxide -propylene oxide
how to distinguish between natural and intentional outbreaks of disease
-first indicator is always an increased number of pts. showing up in hospitals w/symptoms caused by the particular agent -disease that is unusual for a season/geographical area -unusual disease for an age group
aldehydes
-glutaraldehyde- sterilant for heat sensitive instruments -formaldehyde- disinfectant, preservative, extream toxicity
Baltimore classification-7 groups
-group I: dsDNA -group II: ssDNA -group III: dsRNA -group IV: ssRNA(+) -group V: ssRNA(-)template for mRNA transcription -group VI: Retroviruses (ssRNA(+)RT) -group VII: Pararetroviruses (dsDNA-RT)
growing viruses-plant
-grown in living plants -cell culture (protoplast) -problem: how to inoculate plant cells that have tough cell walls
methods of physical control
-heat; moist and dry -cold temperatures -desiccation -radiation -filtration
What percentage of cases are Viral meningitis?
80%
indications of biological attack
-mysterious illness w/many people sick for unknown reasons -large # of insects or unusual insects -large # of dead wild/domestic animals -mysterious disease of plants over large area -mass casualties w/flu-like symptoms
clonal selection theory
-naive t and b cells continually circulate between blood and peripheral lymphoid tissue until they encounter their specific antigen -first encounter of particulat lymphocyte to matching antigen causes it to expand into a clone of cells that can react to that antigen
what are viroids
-naked pieces of RNA (no capsid) -3D -insect enters the cell -plant pathogens
viral structure
-nucleic acid-DNA or RNA, but never both -capsid (protein coat): made of capsomeres -envelope
non-specific immunity
-present at birth -includes 1 and 2nd lines of defense -there is immediate maximal response -not antigen specific -exposure results in no immunologic memory
What are the symptoms of a staph infection?
-pus-filled bumps -Warm to the touch (increase immune activity) -Surrounding area red and inflammed (edema- diluting pathogen)
why viruses are not alive
-use host cell nutrients -do not have DNA and RNA at the same time -no ribosomes for protein synthesis
vaccines against seasonal Influenza;
-usually a mix of influenza A and B -usually grown in chicken eggs -not good for over year or 2 (virus mutates rapidly)
Bulla
1 cm
Monocistronic
1 mRNA = 1 protein
What is the mortality rates of viral meningitis?
1%
Naked Viral Entry
1) Capsid proteins rearrange and viral nucleic acid passes through cytoplasm 2) Receptor mediated endocytosis
Fungal virulence factors
1) Compromised immune system 2) Adhesins 3) Capsules to resist phagocytic engulfment
Naked Viral Movement
1) Endocytic vesicle lyses 2) Capsid undergoes a conformational change to create pores in the endocytic vesicle
Fungi properties
1) Eukaryotic 2) Rigid cell wall 3) Chemoheterotrophs 4) Absorption of nutrients 5) Parasites or saprophytes Most are beneficial!
Envelope Viral Movement
1) Nucleocapsid is dumped into cytoplasm because the viral envelope fused with endocytic vesicle
Damaging Fungal Virulence Factors
1) PAMPs binding to PRR 2) Secrete enzymes to digest cells 3) Mycotoxins secreted
Reverse transcriptase
1) RNA dependent DNA polymerase 2) Ribonuclease activity 3) DNA dependent DNA polymerase Double stranded DNA intermediate
Enveloped Viral Entry
1) Viral envelope fuses with host membrane and release nucleocapsid into the cytoplasm 2) Virus taken in by endocytosis
Microbial Control Goals
1- Destroy and reduce the transmission of pathogens 2- Reduce or eliminate contaminating microbes from food, water, swimming pools, etc.
ASPERGILLUS
1. AFLATOXIN 2. ASPERGILLOSIS
Importance of the nitrogen cycle?
1. Agriculture - plants require fixed nitrogen 2. Aquaculture (including Sea World) - high ammonia levels toxic for fish and orcas - "biofilters" (promote nitrification), nitrate less toxic. 3. Wastewater - denitrification promoted to remove nitrate from wastewater, reduce eutrophication)
Why is the carbon cycle important?
1. It is the base of the food chain in many environments 2. Wastewater treatment 3. Biofuel production (like making methane from agricultural waste)
*HIV infection - cellular level;
1. Attachment - HIV gp120 binding sites must match CD4 receptor cites -CD receptor sites - only found on Macrophages (WBC) -Some CNS cells, T4 helper cell - this is the main target of HIV -CD4 cells are killed by the virus itself -Apoptosis of the cells -Killing of the infected CD4 cells by CD8 cells (cytotoxic or killer T-cells) 2. Penetration by fusion - not phagocytosis 3. Uncoating - capsid is removed and viral genome is exposed 4. Integration -Viral genome (RNA) is reversely transcribed by viral reverse transcriptase to DNA -Then host cell DNA polymerase will covert the resulting single stranded DNA into dsDNA -'intergase' inserts the dsDNA into host cell genome, the resulting piece is called 'provirus' virus is latent and then, the HIV genome is expressed and the virus enters the biosynthesis stage: -RNA polymerase will synthesize the HIV genome -RNA polymerase will also transcribe mRNA for protein synthesis to make coat proteins, gp120 binding proteins, reverse transcriptase
four types of hypersensitivity rxn
1. B cell issue: immediate response (allergies) 2. B cell issue: antibody mediated (blood incompatabilities, myasthenia gravis) 3. B cell issue: immune complexes (RA, lupus) 4. T cell issue: cell medated, cytotoxic T cells (contact dermatitis, TB test)
HERPES (viruses, skin)
1. CHICKENPOX 2. SHINGLES 3. COLD SORE/FEVER BLISTER-HSV1 4. GENITAL HERPES- HSV2 5. ROSEOLA/6th Disease
Fungal Diseases of the Nervous System
1. Cryptococcus
What are the 3 roles of complement proteins?
1. Cytolysis 2. Opsonization 3. Enhance inflammation (complement proteins bind to mast cells)
Transmitted by FLEAS
1. EPIDEMIC MURINE TYPHUS 2. SEPTICEMIA
RICKETTSIA (bacterial, cardio/lymph)
1. EPIDEMIC TYPHUS 2. EPIDEMIC MURINE TYPHUS 3. ROCKY MOUNTAIN SPOTTED FEVER
Bacterial Infections of the Cardiovascular & Lymphatic System
1. Endocarditis 2. Pericarditis 3. Tularemia 4. Brucellosis 5. Anthrax 6. Gas Gangrene 7. Bubonic Plague 8. Lyme disease 9. Epidemic Typhus 10. Epidemic Murine Typhus 11. Rocky Mountain Spotted Fever
VRE- 2 types of bacteria: 1. most common 2. most resistant
1. Enterococcus faecalis 2. Enterococcus faecium (most VRE)
NEISSERIA
1. GONORRHEA 2. MENINGEALCOCCAL 3.
Fungal Diseases of the Lower Respiratory System
1. Histoplasmosis 2. CoccidioidoMYCosis 3. PneuMOCystis Pneumonia (PCP) 4. BlastoMYCOsis 5. Aspergillosis
STAPHYLOCOCCUS
1. INPEDIGO of the NEWBORN 2. ENDOCARDITIS 3. TSS TOXIN
Diseases of the Eyes
1. Inclusion conjunctivitis 2. Chlamydia
Clinical stages of HIV infection, how are the stages determined;
1. Incubation period (2-4 weeks) - Initial exposure of body fluids to HIV 2. Acute infection stage Viremia - fever, headaches, muscle and joint aches 2-4 weeks 3. Asymptomatic (latent) disease stage Virus is latent inside CD4 helper cells No signs or symptoms of illness HIV-positive: Immune system is making antibodies against it Antibodies can be detected Seroconversion takes 6-8 weeks 4. Symptomatic stage of the disease Viral genome is turned on, symptoms begin to appear Genome is turned on by (or in combination with) other infections Alcohol and drug abuse Shock, sunburn, etc. 5. Advanced disease stage - full-blown AIDS Severe opportunistic infections -Kaposi sarcoma - cancer causing virus that affects skin and blood vessels -Toxoplasmosis (affects brain) - protozoan -Cryptosporidiosis (GI tract) - protozoan Many other fungal and viral infections
Viral diseases of the Cardiovascular & Lymphatic System
1. Infectious Mononucleosis 2. Cytomegalovirus Inclusion Disease 3. Hemorrhagic Fevers 4. Yellow Fever 5. Dengue Fever 6. Ebola 7. Zika 8. Hantavirus Pulmonary Syndrome
Bacterial diseases of the skin
1. Macule 2. Papule 3. Vesicle 4. Bulla 5. Pustule 6. Pyogenic 7. Toxic shock syndrome 8. Scalded skin syndrome 9. Impetigo of the newborn 10. Osteomyelitis 11. Saint Anthony's fire 12. Scarlet fever 13. Propionibacterium 14. Pseudomonas-acne 15. Swimmers ear
Bacterial Diseases of the Nervous System
1. Meningitis 2. Encephalitis 3. Neisseria Meninges 4. Streptococcus pneumoniae Meningitis 5. Listeriosis 6. Tetanus 7. Botulism 8. Leprosy
VIRAL DISEASES OF THE DIGESTIVE SYSTEM
1. Mumps 2. Hepatitis A 3. Hepatitis B 4. Hepatitis C 5. Hepatitis D 6. Hepatitis E
Factors that influence degradation of organic matter?
1. Nutrients present in environment 2. Abiotic conditions (pH, oxygen availability, temperature) 3. Microbial community present 4. Xenobiotic degradation (synthetic chemicals)
Steps in translation
1. Parts needed to begin process come together (ribosomal subunit, codon on mRNA, anticodon on tRNA) 2. On the assembled ribosome, the tRNA with first amino acid pairs with start codon on mRNA. First tRNA sits in P site 3. Second codon of mRNA pairs with tRNA carrying second amino acid at the A site. First amino acid joins the second with peptide bond. This attaches the polypeptide to the tRNA in the P site. 4. Ribosome moves along mRNA until the second tRNA is in the P site. The next codon will be brought to A site, and the first tRNA is in E site 5. second amino acid joins to the third by another peptide bond, and first tRNA is released from E site 6. Process continues until reaches stop codon where the polypeptide (amino acid chain in P site) is released 7. Last tRNA is released, ribosome comes apart, released polypeptide forms new protein
What are Koch's postulates?
1. Pathogen must be present in every case of the disease but absent in healthy individuals. 2. Pathogen must be isolated from the diseased host and grown in pure culture. 3. The pathogen from the pure culture must cause the disease when inoculated into a healthy, susceptible lab animal. 4. The pathogen must be isolated from the inoculated animal and must be shown to be the same as the original organism.
Bacterial Diseases of the Lower Respiratory System
1. Pertussis 2. Tuberculosis 3. Pneumococcal Pneumonia 4. Group A Strep Pneumonia 5. Klebsiella Pneumonia 6. Serratia Pneumonia 7. Legionnaires Disease 8. Haemophilus Pneumonia 9. Atypical Pneumonia 10. Psittacosis 11. Q Fever
Mycotic infections of the skin
1. Piedra 2. Tenia Versicolor or petyriasis versicolor 3. Ringworm 4. Rose Garner's disease 5. Chromoblastomycosis 6. Candidiasis 7. Thrush/oral candidiasis 8. Vaginitis
Viral Diseases of the Nervous System
1. Poliomyelitis 2. Rhabdovir 3. Encephalitis
Steps of Transcription
1. RNA binds to promoter 2. DNA unwinds at the beginning of the gene 3. RNA synthesized with complementary base pairing of free nucleotide (based on nucleotide bases on the template strand of DNA) 4. site of synthesis moves along DNA 5. DNA that has been transcribed rewinds 6. transcription reaches the terminator 7. RNA and RNA polymerases are releases and the DNA helix re-forms
Transmitted by TICKS
1. ROCKY MOUNTAIN SPOTTED FEVER 2. LIME DISEASE 3. ULCERGLANDULAR TULAREMIA 4. Q FEVER
What are the 5 signs of inflammation?
1. Redness 2. Heat 3. Pain 4. Swelling 5. Loss of function or altered function
Viral Diseases of the Lower Respiratory System
1. Respiratory Syncytial Virus 2. Influenza
SALMONELLA (bacterial, lower digestive)
1. Salmonellosis 2. Typhoid fever
Vaccines
1. Shingles 2. Streptococcus pneumonia meningitis 3. Rubeola/Mesales 4. Typhoid 5. Hepatitis A& B 6. Mumps/Parotitis 7. Polio 8. Tetanus 9. Diphtheria 10. Pertussis
Bacterial Diseases of the Lower Digestive System
1. Staphylococcus Food Poisoning/Intoxication 2. BACillary Dysentery 3. Salmonellosis 4. Typhoid Fever 5. Cholera 6. GASTROenteritis 7. CampyloBACTER 8. Vibrio parahaemolyticus
Bacterial Diseases of the Upper Respiratory System
1. Streptococcus 2. Diphtheria 3. Common Cold - Coronavirus - Rhinovirus - Adenovirus
Viral diseases of the skin
1. Warts 2. Smallpox 3. Chickenpox 4. Shingles 5. Measles 6. Rubella 7. Cold sores/fever blisters 8. Fifth disease 9. Roseola
viral replication - lysogenic cycle,
1. attachment 2. penetration 3. integration-genome integrated, virus inserts genes into host cell genome -prophage: virus is latent -bacterial cell replicates its genome, at one point, the viral genome is turned on.. 4. biosynthesis-viral genome turned on (takes over host cell for genome replication, transcription, translation 5. assembly 6. release by lysis
viral replication - phages: lytic cycle
1. attachment-binding sites must match receptor sites 2. penetration-viral DNA is injected into bacterial cell 3. biosynthesis 4. assembly (maturation) 5. release by lysis-bacterial cell is filled up and bursts (20-40 min., 50-200 new viruses)
germicidal categories
1. halogens 2.phenolics 3.chlorhexidine 4.alcohols 5.hydrogen peroxide 6.aldehydes 7.gases 8.detergents and soaps 9.heavy metals 10.dyes 11.acids and alkalis
viral morphology
1. helical-rod made of helical capsomeres 2. spherical-many sided 3. enveloped-envelope can surround the helical or spherical capsid 4. complex structure-everything that doesn't fit in the other categories
6 characteristics of life
1. made of cells 2. grow and maintain structure by metabolizing chemicals from the environment 3. respond to the external environmental stimuli 4. reproduce and pass on their genes to the offspring 5. evolve and adapt to their environment 6. all living organisms possess both DNA and RNA
Steps of DNA replication (8 steps) and the enzyme responsible
1. remove supercoiling: DNA gyrase 2. Break hydrogen bonds to get single strands- helicase 3. keep strands apart with- single stranded protein 4. Add RNA primer (b/c it can be added w/out 3' hydroxyl)- primase 5. Adds DNA, uses old DNA strand as template (semi-conservative)- DNA polymerase III 6. Proofreading (remove RNA primers and fixes errors)- DNA polymerase I and II 7. Fix last gap- DNA ligase 8. Supercoil- DNA gyrase
4 major subdivisions of immune system
1. reticul-endothelial system 2. ECF 3. bloodstream 4. lymphatic system
functions of immune system
1. surveillance of the body 2. recognition of foreign material 3. destructino of entities deemed to be foreign
Incubation for AIDS
10 yrs
microbial cells outnumber body cells by ratio of __
10:1 Body cells: ~10^13 cells Microbes in body: >10^14 cells
Reinfection
10K-50K viruses per animal cell
autoclave-
15 PSI/121*C/15-40mins
Unique genes, ?:?
150:1
When was the last known case of smallpox?
1977
Cell to Cell, from ?:? to ?:?
1:1 to 3:1
If Cutaneous Anthrax is left untreated, what is the mortality rate?
20%
Microbes have been on earth for?
3.5 billion years.
What enviornment does Listeria monocytogenes grow the best in?
4C
What is the mortality rate in adults Staphylococcal Scalded Skin Syndrome?
50%-60% (dermis exposed- pathogen gets into blood stream & dehydration)
what percentage is plasma of your blood
60
components of blood
60% plasma water and salts (electrolytes) Proteins: albumin, immunoglobulins, fibrinogen,
how many different RNA molecules for influenza
8
E (oh yeah, now we gettin' INTEGRATIVE) (the intestinal diagnosis is giardiasis, which is an adherence [NOT INVASION] to the intestinal lumen - if you know that mucosal attachment by pathogens is inhibited by IgA, recognize the presence of sinusitis to support this conclusion, and the lack of viral or fungal infections to rule out the other choices, then you've found your answer)
A 5 year-old-boy with a history of severe allergies and recurrent sinusitis presents with foul-smelling, fatty diarrhea. He is at the 50th percentile for height and weight. The boy's mother reports that he has had several such episodes of diarrhea over the years. He does not have any known history of fungal infections or severe viral infections. Which of the following is the most likely underlying cause of this boy's presentation? A. Thymic aplasia B. Hyper IgE syndrome C. Severe combined immune deficiency D. Wiskott-Aldrich syndrome E. IgA deficiency
What is a naïve B cell? How is one activated?
A B cell that has never been exposed to an antigen. They are activated by an antibody and an epitope binding to an IgM on the cell surface.
Neisseria meningiditis
A CSF (cerebrospinal fluid) specimen from a 21 yo college student grew a pure culture of moist grey colonies. The following lab results were observed: Gram stain = gram negative diplococci BAP and Chocolate agar = aerobic growth in CO2 incubator MacConkey and PEA= no growth Oxidase= positive DNase - neg Carbohydrate fermentation = glucose-pos, maltose-pos, sucrose-neg, lactose-neg What is the most likely organism? ________________________________________
A (these guys are cute - he looks like he has a goofy moustache and big worried eyes!)
A child who lives in a rural community, presents with abdominal pain, anorexia, and watery diarrhea. Fecal smear reveals an abundance of protozoan organisms like the one pictured. The microbe most likely responsible is A. Giardia B. Entamoeba C. Cyclospora D. Cryptosporidium E. Balantidium
What is a nucleocapsid?
A combination of the capsid and nucleic acids is called a nucleocapsid.
asthma
A condition in which a person's airways become inflamed, narrow and swell, and produce extra mucus, which makes it difficult to breathe.
Clostridium perfringens
A diabetic patient presents to the emergency department with excrutiating foot pain. Upon further exam he is found to have a foul smelling wound at the base of his foot. A piece of tissue is submitted for aerobic and anerobic culture. The culture results are as follows. No growth on Blood, Chocolate and Macconkey agar incubated at 35degrees in ambient air. Growth on PRAS-ANA brucella plate inubated in a jar with a Gaspak Growth in the bottom of a thioglycollate broth. Gram stain=large gram positive (and gram variable) rods, no spores observed PRAS-ANA brucella agar shows double zone of beta hemolysis Special potency disks = Kanamycin-sensitive, Vancomycin-sensitive, Colistin-resistant Which organism is most likely the agent of his infection?
Antibodies made by a single B cell are all the same
A different B cell makes antibodies with the same "constant region", different variable region.
B (all groups of trematodes/flukes use a snail as a host, most often as an intermediate host)
A freshwater snail is always a part of the life cycle in which of the following groups? A. Cestodes B. Trematodes C. Filarial worms D. Intestinal nematodes
What is genotyping?
A genotypic test where an individual would look for genetic traces of a pathogen, then extract DNA, amplify DNA (PCR), probe DNA which an be done with PCR or can be done with colored probes
What is a Rapid Strep Test?
A immological test, The rapid strep test (RST), or rapid antigen detection test (RADT), is a rapid diagnostic test that is used to assist in the diagnosis of bacterial pharyngitis caused by group A streptococci (GAS), sometimes termed strep throat. The antibodies bind to antigens in the test confirming strep throat. *(Similar to pregnancy test)
What is a hapten?
A molecule with an epitope, but too small to elicit an immune response (antibody formation).
A (the diagnosis is leishmaniasis, vectored by the sandfly - this particular diagnosis is cutaneous leishmaniasis, since the same parasite can cause mucocutaneous, and visceral infections) (B; this is for Cyclospora spp) (C; this is for Plasmodium spp) (D; this is for amoebas and trichomonads) (E; this is for tapeworms and flukes)
A patient enters your clinic with complaint of a large and unsightly sore on her ankle. She has recently returned from a trip to Ethiopia. She recalls being very bothered by many biting flies, particularly about the legs. The lesion is as pictured. Which of the following is the best course of treatment? A. Antimony-containing compounds like sodium stibogluconate B. Folic acid inhibitors like trimethoprim C. Quinine D. Metronidazole E. Praziquantel
Bacteroides fragilis
A patient has developed renal symptoms. After a series of tests a liver abscess was found. She had recently undergone surgery of the small bowel. The abscess was aspirated and sent to lab. The following culture results were observed. Growth on ANA blood plate and LKV plate after 48 hour anaerobic incubation. (no hemolysis) Subsequent culture of those colonies to an aerobic chocolate plate resulted in no growth. Blackening was observed on BBE agar. Gram stain was gram negative rods. Special potency disks: Kanamycin-resistant Vancomycin-resistant Colistin-resistant What is the most likley ID?
C (these are whipworms, Trichuris trichiura, which are geohelminths, meaning environmentally exposed, and the infective stage is the embryonated egg, which has a characteristic bipolar plug appearance) (the prevalence of this worm is very high, and is more common in warm and moist soil) (E; whip it. whip it good)
A patient with intestinal worms has developed an iron-deficiency anemia due to blood loss as well as a deficiency in vitamin A. Microscopic examination of the pathogen is as shown. She was most likely infected by which of the following? A. Rhabditiform larva from contaminated water B. Filariform larva from the soil C. Embryonated eggs from the soil D. Unembryonated eggs from pet-to-human transmission E. The rhythm
C (A, B, and C are all the 'barefoot walking worms' [or geohelminths] but Strongyloides deposits its eggs in the mucosa, so that larvae appear in the stool, while Necator lays eggs directly into the lumen) (Ancylostoma will not lay eggs in people as the definitive host for that species is dog - this guy will cause the cutaneous larva migrans, or creeping eruption)
A patient, who was walking around barefoot on a beach, presents with diarrhea. His fecal smear shows parasitic helminth larvae, but no eggs. Your initial suspicion is infection by A. Ancylostoma B. Necator C. Strongyloides D. Anisakis
What is a phagolysosome?
A phagosome and a lysosome fuse together and function in the digestion of engulfed bacteria.
What is the Lactate Fermentation test? How do you perform this test and examine the results?
A phenotypic test that determines if the bacteria contains lactose. In the test, you use an Eosin Methylene blue plate (selective media because the dye kills gram + bacteria). If the lactose fermentation is rapid, the bacteria will appear as a dark color on the plate. If the bacteria is a non-fermentation, it will appear as colorless. Lastly, E. coli is the only bacteria that will appear green on this plate.
Panel of enzymatic tests
A phenotypic test to see if the bacteria can break down all of the enzymes (ex: Entero Pluri Test)
What is the Catalase test?
A phenotypic test used to determine if a bacteria possesses a catalase enzyme. The presence of catalase enzyme in the test is detected using hydrogen peroxide. If the bacteria possess catalase (i.e., are catalase-positive), when a small amount of bacterial isolate is added to hydrogen peroxide, bubbles of oxygen are observed.
What is a coagulase test?
A phenotypic, biochemical test which can clot plama into gel in tube or agglutinate cocci in slide. This test is useful in differentiating S.aureus from other coagulase-negative staphylococci, because Staphylococcus aureus is known to produce coagulase.
What can an activated B cell differentiate into?
A plasma cell or a memory cell.
What is an epitope?
A portion of an antigen that reacts with an antibody. (one pathogen can have multiple epitopes)
What is a chronic infection?
A prolonged infection.
Haemophilus influenzae
A specimen was isolated from the cerebrospinal fluid of a 4 yr old girl visiting from outside the US. The following results were observed after 48 hr incubation in CO2. Choc = small grey colonies with a mouse-like odor Gram stain= gram negative coccobacilli Blood agar= no growth MacConkey = no growth Satteliting with Staphylococcus = positive Oxidase= pos The most likely organism is: _____________________________________________________
Moraxella catarrhalis
A sputum specimen was obtained from a 79 year old male. He has COPD (chronic obstructive pulmonary disease). The xrays suggest pneumonia. His culture results are as follows: Growth on Chocolate and Blood agar: large opague colonies that when pushed glide across the surface of the media Macconkey= no growth Gram stain = gram negative diplococci Oxidase=positive Carbohydrate fermentation: Glucose=neg, Maltose=neg, Sucrose=neg, Lactose=neg Dnase=positive
What is an antigen?
A substance that can generate an immune response.
What is an adjuvant?
A substance that increases immunogenicity, used in vaccines to boost immune response. (Not antigenic).
What is Immunological Identification?
A type of genotypic test that use antibodies for diagnostic purposes -Antibodies present in blood stream indicate immune response to antigen -Antigens present on surface of pathogens
D (Trichruis trichiura, to be precise - the image is showing Charcot-Leyden crystals) (whipworm is the only parasite we know of that can cause rectal prolapse, but the anemia is also a decent hint to rule out other possibilities - eosinophilia is not terribly specific where parasitic infections are concerned) (Trichuriasis is caused by ingesting the eggs found in the soil - these are NOT the 'piercing and ascending' worms, you must EAT them to get the disease)
A woman presents with a prolapse of her rectum, anemia, and eosinophilia. Her fecal smear reveals the pictured finding. She has an infection of A. Tapeworm B. Hookworm C. Pinworm D. Whipworm E. Ringworm
Why is genotype and phenotype important?
Bacteria
combination therapy
AZT it contains AZT, 3tc, protese inhibitor, and enzyme
agglutination
Ab aggregation; cross-linking cells or particles into large clumps
What are the symptoms for Yellow Fever?
Abdominal pain Hepatomegaly Jaundice Liver malfunction
neutralization
Abs fill the surface receptors on a virus or the active site on a microbial enzyme to prevent it from attaching
Nonliving virus characteristics
Acellular, no self metabolism, RNA or DNA
Why do semi-synthetic work well?
Acid resistant (can take orally) Work against gram - Resistance Central Nervous System
Cell mediated immunity
Activates T lymphocytes, macrophages, NK cells, and cytokines
How can host tissue be indirectly damaged?
Activation of host cell tissue resulting in inflammation.
The degradation of lignin only occurs?
Aerobically.
•
Aerosols importance in disease transmission
Trypanosoma brucei-gambiens
African sleeping sickness Bite of a fly, flagella movement
What is the result of antigen-immunoglobulin binding?
Agglutination (reduces the number of infectious units) Opsonization Neutralization (blocks adhesion of viruses and bacteria to mucosa) Antibody dependent cell-mediated cytotoxicity (antibodies attach to target cell cause destruction by macrophages, eosinophils, and NK cells) Activation of complement
Modified Thayer Martin
All of the following medium are examples of selective anaerobic media, except:
GROWTH AT THE SURFACE OF LIQUID (THIOGLYCOLLATE) MEDIA Foul odor Growth at the surface of liquid (thioglycollate) media Gas Positive gram stain with no aerobic growth
All of these are indicative of the presence of anaerobes, except:
notifying a disease does what
Allows CDC to keep track and intervene.
Multiresistance
Almost all pathogenic microbes have acquired resistance to some chemotherapeutic agents A few pathogens have developed resistance to all known antimicrobial agents *Methicillin-resistant S. aureus (MRSA)* Resistance can be minimized by using antibiotics correctly and only when needed Resistance to a certain antibiotic can be lost if antibiotic is not used for several years
_________ is added to alpha-ketoglutarate to form glutamic acid?
Ammonia. (nitrate can also be used in this step but must be reduced to ammonia first)
What causes Meningioencephalitis ?
Amoeba
capsular characteristics.
Among H. influenzae strains, group designations (e.g., type b) are based on:
B (the diagnosis is most likely Giardia, which is transmitted fecal-orally through contaminated water - the infective stage is the cyst) (A; dogs do have Giardia - they just don't transmit it to people) (C, D, E; this is a whole bunch of other stuff - Giardia isn't one of them) (Giardia is considered a traveler's disease and is more prevalent in children - the diarrhea is also notable in that it's fatty, because the Giardia adheres to the small intestine and prevents fat absorption - this causes the stool to float) (treatments are azoles - usually albendazole for children, metronidazole, or tinidazole)
An 8-year old child presents abdominal distension, and a watery diarrhea that floats in the toilet bowl. Assuming all of the following are true, which is the most likely to have caused this child's infection? A. The family dog licking his face B. Drinking unfiltered water in the family's trip to Mexico one week ago C. A bite from a mosquito D. A bite from a tick E. A bite from a fly
What is hypersensitivity?
An excessive immunological response to an antigen.
What is an Agglutination Test?
An immological test, a blood test used to identify unknown antigens; blood with the unknown antigen is mixed with a known antibody and whether or not agglutination occurs ( clumping of particles) helps to identify the antigen; used in determining blood type
What is the Serology Test?
An immological test, blood tests that look for antibodies in your blood. Diagnosis based on the presence of antibodies in blood stream. This test detects autoimmune disorders, presence of antigen, presence of patient antibodies, allergens, mismatch during blood transfusion
What is autoimmunity?
An immune response against self antigens.
A (Strongyloides stercoralis penetrates the skin, such as the bare soles of feet, in its filariform stage, and ascends via the circulation, typically ending up in the lungs - the host then coughs up and swallows the larvae, which is how they end up in the GI system and become symptomatic)
An immunocompromised patient has diarrhea and abdominal cramps. A blood panel revealed marked eosinophilia and stool samples showed numerous filariform and rhabditiform larvae under microscopy. Which of the following is false regarding this patient's pathogen? A. When the infective stage of this patient penetrated the patient, it went directly to the intestines. B. This pathogen is capable of auto-infecting the patient as part of its life cycle. C. The pathogen is a facultative parasite, and is capable of living freely in soil. D. The pathogen is implicated as the causative agent in some cases of childhood asthma. E. This pathogen can be consistently and safely treated with ivermectin.
What is a nosocomial infection?
An infection originating in a hospital.
Sulfur reduction is linked to what type of respiration? Also what is an effect of sulfur reduction?
Anaerobic respiration Involved in corrosion and souring of oil wells
Denitrification can be a result of?
Anaerobic respiration, present in many facultative anaerobes.
What is a ruminant?
Animals that have stomach divided into four compartments and chew a cud.
Pregnancy test
Another immological test similar to the rapid strep test where the Control strip adheres to molecules not absorbed by the test strip (SEE POWERPOINT PICTURE)
•
Anthrax toxin
Some examples of notifiable disease listed by Arkansas Dept. of Health:
Anthrax, Botulism, Hepatitis A, Pertussis, Plague, Q Fever, SARS, Smallpox, Tularemia, Typhus, Viral Hemorrhagic Fevers
What is the treatment of Cutaneous Anthrax?
Anti bacterial - rod shaped
What is the treatment of Leishmaniasis?
Anti-protist orally and topically
What is the treatment for Chagas disease?
Anti-protozoal
Quinolones
Antibacterial compounds that *interfere with DNA gyrase* (e.g., ciprofloxacin) *Prevent supercoiling of DNA* Broad spectrum Fluoroquinolones Ciprofloxacin used for UTI and against Bacillus anthracis
What is the treatment for Haemophilus influenzae?
Antibiotics
What is the treatment for Neonatal meningitis?
Antibiotics
Challenges with antibodies
Antibiotics are naturally produced antimicrobial agents Challenges with developing new therapeutic drugs: -Toxicity -Pharmokinetics -Can be modified to enhance efficacy (semisynthetic)
What is the treatment for Cellulitis?
Antibiotics to treat Staph Aureus
What is the treatment for Impetigo?
Antibiotics used for Staph aureus
_________ generally can't enter cells (are proteins)
Antibodies
•
Antibody titer in relation to infection/pathogen exposure
•
Antifungal Drugs: drug targets (potential drug toxicity)
Phagocytosis and communication
Antigen is taken up and "processed" Displayed on surface (presenting) attached to MHC molecule (UPC code of host) APCs travel to lymph nodes where majority of B & T cells hang out. Contact, activate T helper cells (CD4 cells)
Innate immunity
Antigen non-specific and immediate
Adaptive immunity
Antigen specific and takes days
28.10 Antimicrobial Drugs
Antimicrobial drugs are classified on the basis of: ----Molecular structure ----Mechanism of action ----Spectrum of antimicrobial activity ----Synthetic or antibiotics
•
Antimicrobial susceptibility tests (e.g. disk diffusion test, Etest etc.)
What is the treatment for Toxoplasmosis?
Antiprotizoal
What is the treatment for viral meningitis?
Antiviral
•
Applications of fluorescent antibody methods
What percentage of the adult population is a carrier of Roseola?
Approximately 100% , many never have symptoms
Obligate Anaerobes
Are poisoned by oxygen, so they gather at the bottom of the tube where the oxygen concentration is lowest.
composting
Arranging organic waste to promote microbial degradation
Yeast reproduction
Asexual budding Sexual spores called acospores undergo meiosis for genetic recombination Asexual spores like blastoconidia
Sporangiospore
Asexual mold spore Born on an external sac aerial hyphae on sporangiophore
Arthrospore
Asexual mold spore Born on fragmentation of vegetative hyphae
Conidiospore
Asexual mold spore Grown externally on aerial hyphae conidiophore
Blastoconidia
Asexual spores
F (any of the listed organisms can be host to a life cycle stage of a trematode species) (snails are the first intermediate hosts, and crustaceans, often crayfish and crab, are the second intermediate hosts - underwater vegetation is where the metacercariae encyst to be consumed by the definitive hosts)
At a seafood restaurant, Tamzid suddenly has crippling flashbacks to Dr. Jensen's intestinal fluke lectures. Which of the foods available are not a part of the trematode life cycle? A. Water chestnuts B. Crayfish C. Freshwater snail D. Watercress E. Crab F. None of the above
What effects viral host range?
Attachment sites on host cell, and host cell genetics/physiology are a basis for host specificity.
Nature of influenza
Attack on respiratory tract *Kills ciliated epithelial cells*, allows bacterial infections. Release of interferon from cells causes symptoms
How do pathogens avoid the innate immune system?
Avoid recognition/killing by competent proteins or phagocytes Capsules Intracellular pathogens Biofilms
Why has resistance of vancomycin sprung up?
Avoparcin (in same family) used in animal feeds
What type of cells are involved in antibody production?
B cells
Humoral immunity
B lymphocytes and antibody production Tips of antibodies block viruses from attaching to the host
What is the agent for Impetigo?
BACTERIA -Staphylococcus aureus (exfoliative toxins: break down ECM/ dermis breaks down) -Stretopcoccus pyogenes (more common in older children
What is the agent in Cutaneous Anthrax?
Bacillis Anthracis (Bacterial endospores)
•
Bacillus anthracis (soil bacteria)
Bacteria can be invasive
Bacteria spread through tissues, usually using digestive enzymes which damage tissues, kill cells.
What type of infection is Endocarditis ?
Bacterial
examples of antigens
Bacterial membrane proteins Bacterial capsules Viral capsid proteins Proteins on plant pollen, bee venom, mold spores
What type of infection is Listeria monocytogenes?
Bacterial meningitis
What type of infection is Neisseria meningitidis?
Bacterial meningitis
What type is Neonatal meningitis?
Bacterial meninigtis
What type of infection is Streptococcus pneumoniae?
Bacterial meninigtis
How can host tissue be directly damaged?
Bacterial toxins and viral multiplication can cause host cell lysis.
What is the treatment for Conjunctivits?
Bacterial: Antibacterial eye drops Viral: Clears on own
•
Bacteriophage encoded exotoxins
What is a bacterial virus called?
Bacteriophage or phage.
Periodontal disease (disease of digestive)
Bacteroides gingivalis
Balantidium coli
Balantidiasis (Diarrhea) Cilia movement
Viral classification
Based upon nucleic acid and how that acid is transcribed Type of genome they contain, the type of capsid they possess, and whether they are enveloped or naked.
What are the symptoms of Rubella in fetuses?
Birth defects: deafness, cardiac abnormalities
B (the eggs of both Ascaris lumbricoides and Trichuris trichiura must mature in soil before they are infectious, and infection occurs when the human ingests these eggs from the soil - since both nematodes are endemic in the same areas of the world, they are often seen as a mixed infection)
Because mature infectious eggs are ingested from the soil, which of the following parasites might be seen in the same patient at the same time? A. Hookworm and Ascaris lumbricoides B. Ascaris lumbricoides and Trichuris trichiura C. Trichuris trichiura and Schistosoma japonicum D. Schistosoma japonicum and hookworm
Antibodies attach to antigens. Period. But...
Because there are at least 2 binding sites, cross-bridges form, linking antigens together in clumps. Attaching covers up critical sites on the antigens
HIV Assembly
Begins in the plasma membrane Maturation occurs at budding of virion Env polyprotein is cleaved by a protease in the Golgi to make gp41 and gp120
B (even if you don't know the typical history and presentation of Clonorchis sinensis, it should be reasonable to deduce that the passing of abnormal gallstones indicates something residing in the gallbladder, with cholangiocarcinoma therefore being the most likely cancer consequence)
Bernard presents with liver dysfunctions, the most notable of which is the passing of pigmented gallstones. He admits to eating raw fish recently. The parasite currently infecting him predisposes to what kind of cancer? A. Hepatic angiosarcoma B. Cholangiocarcinoma C. Pancreatic adenoma D. Duodenal adenocarcinoma
Amphotericin B, pimaricin, and nystatin
Bind to ergosterol in cell membrane and cause leakage Treats systemic Candida infections
Protease Inhibitors
Bind to the active site of *HIV proteases* Inhibit the processing of large viral proteins into individual components Used in treatment against *HIV* and *hepatitis*
Viral Attachment
Binding of attachment sites on viral surface to receptor cells on cytoplasmic membrane of the host
what is the HIV life cycle
Binding to host cell Copying RNA into DNA by Rev Trans Integration into host DNA Transcription Translation Assembly and exit by budding
What kind of test is a Biochemical Test? What is it? Give examples.
Biochemical tests are a phenotypic test that tests the enzymes produced by a given bacteria, or presence of a bacterial chemical. Examples of this would be a Coagulase Test, Gram staining, and selective vs differential media.
How can bacteria in biofilms evade the immune system?
Biofilms are highly resistant to immune response. They can occur on teeth, catheters, and other implants.
when was the creating and stockpiling of biological weapons banned (at which convention)
Biological Weapons Convention-1972
Naftifine (Naftin) and terbinafine (Lamisil)
Block ergosterol synthesis Treats dermatophyte infections
What is the diagnosis for Streptococcus pneumoniae?
Blood agar (sphere) and gram staining (+)
What is the Nervous System's defense network?
Blood brain barrier and Microglia *Pros and cons Immune privileged Weakened defense
What is the diagnosis of Chagas disease?
Blood smear
•
Bordetella pertussis (Pertussis - whooping cough)
Pertussis (whooping cough) (disease of respiratory)
Bordetella pertussis; very contagious, toxins interfere w resp. tract normal ability to eliminate germs. destroys ciliated cells in trachea. transmission by droplets. flu-like symptoms. dry cough becomes wet cough in couple weeks. coughing spells that last over a min.
What is the treatment for Botulism ?
Botex
What is the treatment for Tetanus?
Botex (muscle relaxers)
-
Botulinum toxin
What is the portal of entry for Cellulitis?
Breaks in the skin
What is the portal of entry for Ringworm?
Breaks in the skin
D (the patient does have Giardia lamblia, as the cysts are present in his stool, but he is not symptomatic nor immunocompromised, so there is no reason to treat him - only symptomatic cases should get treatment) (B; cryptosporidium looks different from this - look for acid fast staining of spherical [not ovoid] cysts)
Bryce recently returned home from a camping trip. He is planning on heading back to college but is worried that he may have a parasitic infection after drinking unfiltered stream water on his trip. He sets up an appointment with his primary care physician. He denies any diarrhea, constipation, bloating, or flatulence. He is adamant about having a fecal exam, so the primary care physician obliges. The fecal exam reveals the image. What is the most appropriate next step in Bryce's care? A. Treatment with metronidazole to prevent future symptoms, as he is infected with Giardia lamblia B. Treatment with azithromycin because he is infected with cryptosporidium C. No treatment, as he is not currently infected D. No treatment because is not symptomatic E. No treatment because there is no effective treatment
What are the symptoms of Lyme Disease?
Bull's-eye rash Rheumatoid conditions Fever Fatigue Soreness *chronic
What is a viral protein coat called?
Capsid.
Translation for eukaryote has a _____ which tells ribosome where to bind, and a _____ which protects from enzymes
CAP, Poly A tail
AIDS diagnosis
CD4 cell count below 200/µl (from 500-1200), or if you get an opportunistic infection once you've been diagnosed with HIV
What is the nervous system comprised of?
CNS, PNS
How do you diagnose Encephalitis?
CSF/ blood plasma example sample and look for protist
What do monocytes do?
Called macrophages when they mature. Involved in phagocytosis.
Campylobacteriosis (disease of digestive)
Campylobacter jejuni; infection of colon epithelial lining with exotoxin production; frequent watery stools; nausea; headaches; transmission by ingesting contaminated meat
Plus (+) RNA viruses can perform similar actions to what?
Can act as a message i.e. like mRNA.
Facultative anaerobes
Can grow with or without oxygen because they can metabolise energy aerobically or anaerobically. They gather mostly at the top because aerobic respiration generates more ATP than either fermentation or anaerobic respiration, but some reside at bottom.
D (the diagnosis is neurocysticercosis, which is caused by ingestion of tapeworm eggs from human feces [humans are the definitive host], NOT from ingesting the infected pork that contains the cysticerci [pigs are the intermediate host]) (the pathogen is Taenia solium)
Carey is admitted to the hospital with recurring seizures and neurologic episodes. During his seizures, he is unable to talk, suffers visual disturbances, and loses consciousness. He has lingering neurological symptoms, including hemiparesis. The MRI revealed several cystic masses in the brain as shown. Carey was most likely infected by A. Ingesting undercooked beef B. Ingesting undercooked pork C. Ingesting raw fish D. Ingesting human fecal material E. Watching the pilot episode of House, M.D.
Entamoeba histolytica
Causes amoebic dysentery Cysts pass through intestines Pseudopodia
What is the Varicella virus? What type of virus is it?
Causes chicken pox and shingles. A DNA virus.
Coccidioides immitis
Causes coccidioidomycosis by inhaling arthrospores in southwestern USA
*Opportunistic pathogen*
Causes disease only in the absence of normal host resistance (e.g cancer or AIDS)
•
Causes for wide spread drug resistance
Histoplasma capsulatum
Causes histoplasmosis around the Great Lakes from bird or bat droppings Budding encapsulated yeast
Plasmodium species
Causes malaria Non motile
What is the Influenza virus? What type of virus is it?
Causes the flu. A RNA virus.
Malassezia globosa
Causes tinea versicolor which is a skin infection and is hypopigmentation of the skin. Also causes dandruff and is found naturally on the skin. Dimorphic.
Digestive organ with bacterial digestion located near the small intestine?
Cecum.
How does viral release work for naked viruses?
Cell lysis occurs.
Where do enveloped animal viruses obtain their envelope from?
Cell membrane of the host cell.
Where do DNA viruses go during uncoating?
Cell nucleus.
HIV Transmission
Cell to cell contact
Viruses do not have what?
Cells.
DNA -> RNA -> Protein
Central dogma
Oral Cavity: Common microbes/pathogens in upper respiratory tracts
Certain microbes colonizes the *upper respiratory tract* ---->E.g.: staphylococci, streptococci, diphtheroid bacilli, gram-negative cocci ---->*Potential pathogens* (e.g. staphylococcus aureus and streptococcus pneumonia) -*Innate and adaptive immune systems are typically extra active in these mucosal surfaces to protect from pathogenic infections*
H. ducreyi
Chancroid is the sexually transmitted disease caused by:
What is the treatment for Dengue fever ?
Clears on own usually after 2-7 days
What is the diagnosis for Chicken Pox and Shingles ?
Clinical
What is the diagnosis for Smallpox?
Clinical
What is the agent for Botulism ?
Clostridium botulinum (Endospores) * anaerobic
Botulism (disease of nervous system)
Clostridium botulinum; Gram + bacillus; food borne infections especially honey; botulinum toxin blocks Ach messages betweenn muscles and nerves and causes paralysis. muscle weakness and no fever
Gas Gangrene (skin disease)
Clostridium perfringens; an anaerobe, often mixed infection; really distinct smell; brown frothy pus
What is the agent that causes Tetanus ?
Clostridium tetani
Clostridium AB exotoxins
Clostridium tetani and Clostridium botulinum produce potent AB exotoxins that affect nervous tissue Botulinum toxin - consists of several related AB toxins that are the most potent biological toxins known Tetanus toxin - is also an AB protein neurotoxin Both *neurotoxins block release of neurotransmitters involved in muscle control, but they have different mode of action (with different disease symptoms)*
Tetanus (disease of nervous system)
Clostridium tetani, infection begins when spores are introduced into an injury or wound. tetanospasmin selectivley blocks inhibitory nerve transmission (GABA) release from spinal cord to muscles allowing muscle spasms. lockjaw, seizures, spasms
What is Spongiform Encephalopathy?
Clumps of misfolded prions causing a "spongy" appearance in the brain
Dimorphic fungi examples
Coccidioides immitis Histoplasma capsulatum
New Antimicrobial Drugs
Computers can now be used to design molecules to interact with specific microbial structures ----Most successful example is saquinavir ---------Binds to active site of HIV protease New methods of screening natural products are being used ---Led to the discovery of platensimycin Combinations of drugs can be used (e.g., ampicillin and sulbactam) Bacteriophage therapy
Microconidia
Conidium of the smaller of two types produced by same fungal species but differing in shape
viruses cause damage directly
Consume host resources, cell dies Blocks host transcription or translation
Oncogenes
Converted from proto-oncogenes and promote tumor growth
RNA mutates more readily than DNA explain why..
Copying mistakes by the enzymes are not corrected Not double stranded, so no mismatches noticed that can be fixed. Many variants result Less chance of lasting immunity Harder to create vaccines
SARS: Severe Acute Respiratory Distress syndrome What kind of virus what is its reservoir
Coronavirus that caused brief pandemic Reservoir is animals; currently under control.
•
Corynebacterium diphtheria (Diphtheria)
-Diptheria toxin
Corynebacterium diptheriae *Encapsulated strains are especially pathogenic (highly invasive) *Forms irregular rods during growth *Diphtheria toxin inhibits protein synthesis in eukaryotes Spreads by airborne droplets via the respiratory route Pathogenic strains lysogenized by bacteriophage produce a powerful exotoxin (diphtheria toxin) that causes: Tissue death The appearance of the pseudomembrane in the patient's throat Previous infection or immunization provides resistance Vaccine (DTaP - diptheria toxoid, tetanus toxoid, and pertussis) Diphtheria antitoxin available for acute cases
exit from mouth by
Coughing sneezing speaking
Give an example of Spongiform Encephalopathy.
Creutzfeldt-Jakob /Bovine Spongiform Encephalopathy
What are Koch's postulates used for?
Criteria used to determine the cause for a disease.
Cryptococcus neoformans
Cryptococcosis infection from bird feces that can inflame the meninges Dimorphic
V factor
Cultivation of H. ducreyi will require all of the following, except:
Drug treatment for HIV
Current drugs block 6 different steps in HIV ART cocktail includes a mixture of several drugs
prevalence in the world (don't need to know the numbers - just have a general idea how bad it is);
Currently an estimated 34 million people are living with HIV/AIDS (0.6% of world's population)
Pathology
Cutaneous lesions Scalded Skin Syndrome Impetigo of the Newborn
HIV Attachment
Cyclophilin protein binds to heparin receptor on the host cell
Candida albicans virulence factors
Cytokine GM-CSF suppresses complement production, acquires iron, switch to virulent form
What happens after a phagocyte binds a pathogen?
Cytokines are released, more phagocytes are recruited to the area, inflammation is triggered. Then the pathogen is engulfed into a phagosome which then fuses with a lysosome to form a phagolysosome.
Type I interferons
Cytokines blocking viral replication
Where do RNA viruses go during uncoating?
Cytoplasm.
What are Tc?
Cytotoxic T cells.
Replication
DNA -> DNA
Transcription
DNA -> RNA
Central Dogma
DNA -> RNA -> Protein
Important enzyme in DNA replication: makes covalent bonds to join DNA strands; Okazaki fragments, and new segments in excision repair
DNA Ligase
Important enzyme in DNA replication: Synthesizes DNA; proofreads and repairs DNA
DNA Polymerases
Neutrophils release *what two things*
DNA and anti-microbial proteins
Important enzyme in DNA replication: reduces supercoiling ahead of the replication fork
DNA gyrase
Definition of a virus criteria
DNA or RNA not both (single or double) Dependent on host cell for replication Viral components must assemble
What is the vaccine for Tetanus?
DTaP- killed virus (only good for 10 years)
What enzyme do bacteria use in order to perform ammonification?
Deaminase.
Mortality and morbidity rate:
Death and sickness per total population over a given period of time.
What is humoral immunity?
Defends against extracellular pathogens only. Can be active or passive. Produces and secretes B cells, antibody production.
What is the portal of entry for Hemorrhagic Fever ?
Dengue fever (mosquitos) Yellow fever (mosquitos) Ebola (bats)
Tropical and other arboviruses tropics:
Dengue fever, Yellow fever, other hemorrhagic fevers.
What is the treatment for Septicemia?
Depends on agent for treatment
Obtaining a viral envelope
Derived from host cell membrane through budding
Where is the reservoir for Chicken Pox and Shingles ?
Dermatome (spinal nerve area)
What is the agent for Ringworm?
Dermatophytes (fungal)
4)
Describe how orally taken antibiotics can result in the harmful alteration in digestive functions and disease. How can you minimize these alterations of the natural microflora during/after an antibiotics therapy?
1)
Describe mechanisms of transfer of healthcare-associated infections (HAIs). Why are the incidences of HAIs so high?
5)
Describe the use of combination drugs and why they are effective.
What do natural killer (NK) cells do?
Destroy target cells (host cells that have a pathogenic infection or tumor).
What cells does HIV infect?
Destroys helper T cells.
What happens when someone gets Naegleria fowleri?
Destruction of brain and spine, hemmorage
•
Detection of viral infections
What are diagnosis of Nonhemorrhagic Fever ?
Diagnosis culture based/patient history based
•
Difference between direct agglutination and passive agglutination
Blastomyces dermatitidis
Dimorphic fungi that causes blastomycosis in Great Lakes through inhalation or skin leading to pulmonary infection
What shape is Neisseria meningitidis?
Diplococcal
•
Direct and indirect fluorescent antibody methods
What is the portal of entry for Endocarditis ?
Direct entry to blood stream
What is the portal of entry for Septicemia?
Direct entry to blood stream, can arise as secondary infection UTI Some maternal death post-childbirth
2)
Discuss antimicrobial susceptibility testing and how the use of minimal inhibitory concentration (MIC) values.
7)
Discuss different types of virulence factors a pathogen can use to infect a host.
4)
Discuss how recent advances in molecular biology of nucleic acids have revolutionized methods used for identification of infectious disease agents and how they are favored over traditional biochemical tests.
What are prions?
Disease caused by misfolded proteins
BACTERIAL FOOD INFECTION
Disease results from microbial growth in host tissues. Slower onset of symptoms.
Bacterial Diseases of the Digestive System
Diseases of the digestive system rank 2nd only to respiratory diseases. 1. Dental Carries/Tooth decay 2. Acute Necrotizing Ulcerative Gingivitis
What is the agent for Infectious Mononucleosis?
EBV
What is the diagnosis for Rubella?
ELISA
What is the diagnosis for measles?
ELISA
What is the diagnosis for Lyme Disease?
ELISA Genetic methods
How do you diagnose Dengue fever ?
ELISA Genotyping (RNA)
What is the diagnosis for Botulism ?
ELISA (look for toxin, agent) Gram staining (+) Presence of microbe in stool sample
What is the diagnosis of HIV?
ELISA-based test to look for virus particles Viral load tests - PCR-based test to determine amount of virus circulating. Monitors progression of the disease
•
Effect on immune system
What type of disease are prions?
Encephalitis
What type of disease is Spongiform Encephalopathy?
Encephalitis
What type of disease is Toxoplasmosis?
Encephalitis
What type of disease is rabies?
Encephalitis **ZOONOTIC DISEASE*
released upon G- cell death
Endotoxin
what are considered super antigens
Endotoxin, Toxic Shock Syndrome toxin
Steps in an infectious disease-Overview
Entry and attachment Deal with host defense Damage Escape
What is essential to a virus binding to a host cell and infecting it?
Envelope if present or capsid if not.
What is a virus called that has a membrane?
Enveloped.
What is Penicillinase? What is it produced by?
Enzyme -an enzyme that can inactivate penicillin -produced by bacteria
28.7
Enzyme Immunoassays, Rapid Tests, and Immunoblots
HIV detection - ELISA and Western Blot;
Enzyme Linked Immunosorbant Assay (too many false positive) Test for HIV antibodies If ELISA is positive, the same sample is tested again If comes back positive 2 times, the sample is subjected to Western Blot test - test for viral antigen
•
Enzyme immunoassay (EIA)
F+ x F- = ?
F+
Can only conjugate with _____ cell
F-
Hfr x F- = ?
F-
Cutaneous lesions
FOLLICULITIS ➡️ FRUNCLES ➡️ CARBUNCLES ALL = staphylococcus
HIV- classification, origin, discovery;
Family: Retrovirus (VI) Genus: Lentivirus Species: HIV 1 (high virulence) and HIV 2 (low virulence) discovered 1983-Luc Montagnier
Transmission of cholera
Fecal-oral - water (ice) - raw foods - undercooked foods - washing foods in contaminated water Found in copepods and algae
What are the symptoms of Septicemia?
Fever, chills, shaking, altered mental state
What are the symptoms of Endocarditis ?
Fever, fatigue, joint pain, edema, anemia, abnormal heartbeat, heart murmurs, fever, blue nail beds Bacterial Vegetation in valve
What are the symptoms for Malaria?
Fever, nausea, weakness
Mold
Filamentous, multinucleated fungi composed of hyphae Reproduce with sexual or asexual spores
Virulence of the pathogen
Fimbriae, capsules, toxins, antibiotic resistance Pathogen concentration (for fecal-oral spread)
Protozoa Asexual Reproduction
Fission, budding, schizogeny Can reproduce sexually with gametes
Macule
Flat red lesion
What are the symptoms in Toxoplasmosis?
Flu-like symptoms, although many in the population are asymptomatic, chronic, behavioral changes
What is the treatment for Cryptococus neoformans?
Flucanazole, amphotericin B (Antifungal orally) *Opportunistic Pathogen*
Vesicle
Fluid filled < 1 cm
Cells specializing in Antigen presenting
Follicular dendritic cells Macrophages, and B cells
IgE
Found on mast cells and basophils, binds to antigens on parasites, also involved in response to allergens.
IgD
Found on the surface of some B cells instead of IgM
What is the agent for Tularemia?
Francisella tularensis (intracellular)
Tularemia (rabbit fever) (disease of circulatory)
Francisella tularensis; spread through direct contact and usually spread through mammals; red spots on skin, enlarged lymph nodes where site is, weight loss, joint stiffness, chills
What type of infection is Coccidioides?
Fungal Meningitis
Pseudohyphae
Fungal filament that grew and remain attached made of budding yeast cells
Mycoses
Fungal infection
What type of infection is Cryptococus neoformans?
Fungal meningitis
•
Fusion inhibitors (entry inhibitors)
Food Poisoning by Clostridum perfringens (disease of digestive)
G+ bacillus, endospore, food borne intoxication; diarrhea; bacteria usually in meat and animals
Diphtheria (disease of respiratory)
G+, pleomorphic, rods; fever, headaches, sore throat, tiredness, eventual formation of a pseudomembrane that covers throat and nasal passages
50% of infectious diseases of
GI tract
where do lots of bacteria live at on human. and do they colonize?
GI tract, Skin yes
exit in excretory system by
GI tract, in feces; in urine
Cryptosporidium
Gastrointestinal infection called cryptosporidiosis Non motile Intracellular parasite
What is Meningitis?
General inflammation of the meninges
People have lots of entry points name same:
Glands, follicles that open to outside Larger openings: respiratory system, digestive system, genito-urinary tract Cuts, bites, burns, surgical incisions Crossing the placenta, infecting the fetus
How can you prevent Staph Aureus?
Good hygiene is the only prevention; Clean scratches and scrapes when they occur with a topical antibiotic
LTA is what LPS is what
Gram + Gram -
What is the treatment for Listeria monocytogenes?
Gram + Antibiotic (ex: penicillin)
Is the Bubonic Plague gram + or - ? Shape?
Gram-negative rods
What is the agent for Acanthamoeba?
Granulomatous AM
Dermatophyte
Group of molds causing superficial mycoses of the skin, hair, and nails by utilizing keratin EX: Ringworm, tinea
Dimorphic yeast
Grows as an oval, budding yeast, but can elongate and remain attached using pseudohyphae EX: Candida
Why is Listeria monocytogenes so hard to fight off?
Grows within cells, contributing to virulence
Viruses Causing Cancer
HBV, HCV, HPV, EBV, KSHV, HIV, HTLV-1
What is the vaccine for Haemophilus influenzae?
HIB
what causes AIDS
HIV
examples of RNA virus
HIV and Common Cold (Rhinovirus)
Modes of HIV transmission;
HIV is transmitted by exposure to infected body fluids, blood, some, semen and vaginal secretion, needle sharing -tears, urine, not potent enough -mother to child: breast milk, blood, etc, 100% to antibodies 25% HIV positive
*Treatment for HIV - nucleoside inhibitors, why don't we have vaccine;
HIV mutates too rapidly -several vaccines have been developed, none of the m successful -inhibit some viral structures, entry and exit from cell
•
HIV pathogenesis
•
HIV treatment
What is the Cardiovascular system?
Heart, lungs, and blood vessels
Virus shapes
Helical, polyhedral, enveloped, binal 5 to 300 nm
Important enzyme in DNA replication: Unwinds double stranded DNA
Helicase
Gastritis/Peptic Ulcers (disease of digestive)
Heliobacter pylori; bacteria thrives in environment of stomach by producing enzyme that converts urea to ammonia, raising ph in the cells vicinity. G- bacillus; sharp burning pain, bloody stools, vomiting, may lead to stomach cancer.
What are Th?
Helper T cells.
•
Hemagglutination (human blood typing)
Transmitted by sex, blood, mother to child Lasts for weeks; can become chronic, cause cancer
Hep B
Transmitted through blood contact *Quietly* destroys liver; chronic, cancer possible
Hep C
Enteric virus, transmitted fecal oral route Lasts for weeks; seldom chronic, no cancer related to rhinovirus
Hep. A
can cause chronic inflammation Liver cells replaced by connective tissue, organ fails can also lead to liver cancer
Hepatits B, C
HSV-1
Herpes virus that remains latent Causes fever blisters or oral herpes Replicates in epithelial cells
HSV-2
Herpes virus that remains latent Genital herpes Replicates in epithelial cells
EBV
Herpes virus that remains latent Mononucleosis
Viral Replication
Host cell ribosomes will translated viral mRNA into structures and enzymes
Host processes
Host defenses, like inflammation, may over-respond, cause significant tissue damage. Bacteria can aid this: Gram neg. endotoxin (LPS)
describe more of a opportunistic pathogen
Host is compromised (cuts, underlying conditions, etc.) Organism gets where it doesn't belong -E. coli and urinary tract infections. Lack of microbial antagonism, e.g. superinfection -competition for space, nutrients; bacteriocins.
HHV
Human Herpes Virus Member of large Herpes virus family
What is the agent for Roseola?
Human Herpesvirus 6
Cause of genital warts and cervical cancer
Human Papilloma virus HPV
What is an example of an oncogenic virus?
Human Papilloma virus (HPV) which causes cervical cancer.
reservoirs
Humans -May be sick, or may be carriers Animals -Many diseases are zoonotic but can also be caught by humans; animal may be healthy or not. Non-living -Soil, water are typical homes to microbes, some of which can cause disease.
What type of morphology does Coccidioides have?
Hyphae (filamentous) morphology
What is another name for rabies?
Hyrdophobia
Following antibiotic therapy, patients are often administered ________ to facilitate recolonizaton of normal flora. A. Fluoride. B. Probiotics. C. Iron. D. Antivirals.
I think it is B
What is clonal proliferation?
If an antigen/antibody interaction occurs, the body needs to make more of these specific B cells, these B cells will secrete antibodies that specifically target the antigen present in the body.
What type of rash is Cellulitis?
Large Pustular
28.5
Immunoassays and Disease
•
Immunoblots
What is congenital immunodeficiency?
Immunodeficiency from birth.
Naturally resistant of microbes to antibiotics
Impermeable to antibiotic Inactivate the antibiotic Modify the target of the antibiotic Resistant biochemical pathway Ability to pump out the antibiotic (efflux)
C (ingestion of Taenia solium cysticerci in undercooked or raw pork will cause the adult tapeworm to mature in the intestine, making the human the definitive host - if the eggs from the adult tapeworm are ingested, then the cysticerci will develop in the human causing cysticercosis and making the human the accidental intermediate host)
In infections with Taenia solium, humans can serve as the A. Definitive host B. Intermediate host C. Both of the above D. None of the above
What is a plus single-stranded (+ss) nucleic acid?
It is analogous to the sense strand.
Where does RNA replication of RNA viruses occur?
In the cytoplasm, nucleus is not involved.
Yes- this is a suitable source
Indicate whether the following clinical specimens are suitable or unsuitable for anaerobic culture: Aspirated bladder urine
T-tropic
Infect mature dendritic cells and T4 lymphocytes, faster replicating, more harmful Bind to CD4 and CXCR4
What is a generalized/systemic infection?
Infection affects many or all parts of the body.
Acanthamoeba
Infection of eye, skin, and CNS Enters eye through contact lens/wounds Pseudopodia
CYSTITIS
Infection of the bladder
What is Keratitis?
Infection of the cornea
PYLONEPHRITIS
Infection of the kidney
PROSTATITIS
Infection of the prostate
URETHRITIS
Infection of the urethra
What is a localized infection?
Infection restricted to one area.
Haemophilus influenzae
Infection with which of the following organisms is not associated with the ingestion of food products
Prions
Infectious protein particles causing transmissible neurodegenerative disorders EX: Mad cow disease, Parkinsons, Alzheimers
Acute HIV infection (Early)
Infects T4 lymphocytes expressing CCR5 Binds to dendritic cells and dendritic cells detach and carry virus to lymph nodes
Trichomonas vaginalis
Infects male urinary tract and female vagina Flagella movement
ENCEPHALITIS
Inflammation of the Brain
What is Encephalitis?
Inflammation of the brain itself
What is Conjunctivits?
Inflammation of the conjunctiva
What is Endocarditis ?
Inflammation of the endocardium, heart valves
MENINGITIS
Inflammation of the meninges
•
Influenza outbreaks (antigenic drift/shift)
•
Influenza viruses (Orthomyxovirus)
a serious respiratory disease Virus has a segmented genome
Influenza:
What is the portal of entry for prions?
Ingested
Examples of portals of entry?
Inhalation, ingestion, wounds, sexual contact, etc.
What is the portal of entry for Spongiform Encephalopathy?
Injestion
What is a minus single-stranded (-ss) nucleic acid?
It is analogous to the antisense strand.
What is the immune status of the host?
Innate immune system must be breached to allow pathogen to get established.
What basically occurs in nitrogen assimilation?
Inorganic N is converted to organic N.
CO2
Inorganic carbon Carbon dioxide
CH4
Inorganic carbon Methane
C
Inorganic carbon. Graphite, Diamonds, Charcoal.
What encompasses the first line of defense?
Intact skin/ mucous membranes Normal microbiota
Antimetabolites (trimethoprim + sulfomethoxazole , trimetrexate, atovaquone, and flucytosin)
Interfere with nucleic acid synthesis Treats Candida, Pneumocystis
Clotrimazole, miconazole, itraconazole, fluconazole, and ketoconazole
Interferes with ergosterol synthesis and alters membrane structure
Griseofulvin
Interferes with nuclear division by preventing microtubule aggregation Only for dermatophytes
Antibiotic(s) that target mycolic acid
Isoniazid Ethambutol
Isoniazid
Isoniazid is a *growth analog effective specific against Mycobacterium tuberculosis* Interferes with synthesis of mycolic acid required for mycobacterial cell walls Combination with other drugs (e.g. moxifloxacin - fluoroquinolones)
How does gastric juice aid in reducing pathogens?
It acts as a chemical barrier that inhibits almost all pathogens.
How does vomiting aid in reducing pathogens?
It acts as a physical barrier. Forcefully expels microbes.
What is hemolysis? What media would you use for this test? What are the results?
It is the breaking down red blood cells (phenotypic test). This test is performed on a sheep blood agar. If the bacteria is a greenish/brown color, it is considered "Alpha" meaning the oxidation of iron in blood cells. If the bacteria is a yellow color with a halo, it is considered "Beta" meaning iron was not oxidized, but broke down the blood cells. If the bacteria is clear, it is considered "Gamma", meaning no hemolysis took place.
Why is it important to follow prescription?
It lowers all the microbes long enough, so immune system can wipe out resistant ones
Why is fever an important non-specific defense?
It prevents pathogens from replicating, decreases free iron, and increases interferon production (this protects mainly from viral infections)
A (the diagnosis is amebiasis from Entamoeba histolytica - a key feature is bloody diarrhea, as opposed to watery and fatty diarrhea as with giardiasis or cryptosporidiosis) (B; this is a trait of tapeworm infection, not amoeba) (C; a stool sample is a good idea, but the E. disbar cysts are not pathogenic nor diagnostic) (D; too many organisms are spread by contaminated water in underdeveloped countries for this to be a useful avenue of investigation - asking about symptoms is usually better than asking about etiology) (E; immune compromise is a good question to ask if suspecting cryptosporidiosis, but the MRI gives you no reason to suspect that, as the liver abscess is more characteristic of Entamoeba)
Jen, a medical student recently on mission to Colombia, comes in with complaints of right upper quadrant abdominal pain, but the history didn't match a true appendicitis. Suspicious, you popped her into the MRI, and found a large liver abscess in the right lobe of the liver. Which of the following should you do to determine the presence of a key symptom/sign? A. Ask about bloody diarrhea B. Check for a low level of vitamin B12 C. Acquire a stool sample and look for Entamoeba disbar cysts D. Ask if the patient drank the water in Colombia E. Whether the patient is immune-compromised
•
Kaposi's sarcoma
What are the symptoms of HIV?
Kaposi's sarcoma (picture) Muscle atrophy, wasting AIDS associated dementia
functions of non specific host defenses
Keep microbes "out" Minimize microbial load Internal, non-specific line of defense
C (this guy has a cestode/tapeworm, most likely Diphyllobothrium latum based on the diet - tapeworms are long and flat, and in fact D. latum is one of the longest; adults can reach up to 10 meters long) (A; this describes ascarids/roundworms) (B; this describes trichuris/whipworm) (D; this describes trematodes/flukes) (E; Giardia or entamoeba, but those would cause diarrhea)
Kelsey comes in for tests due to a concern over his stool: he says that his stool is riddled with semitransparent white spheres, and he's worried he has a parasite. He has no diarrhea or symptoms of any kind that he knows of, nor has he felt ill. When asked about recent travel, he replies that he was in Japan a couple months ago, where he ate sushi as fresh (meaning raw) as he has ever had. Blood tests revealed pernicious anemia. Which of the following is the most likely morphology of Kelsey's parasite? A. Pencil-thick and about 30 cm long B. A thread-like anterior end and a tightly coiled posterior end C. Flat and several meters long D. Flat and leaf-shaped with two suckers E. Amoeboid
What do Tc cells do?
Kill cells infected with intracellular pathogens and tumor cells (target cells) Infected cells display an antigen from the infecting organism on their surface, marks them as "non-self" and Tc cells find these cells with the non-self antigens and destroy them.
Lactic acid in Vagina
Lactic acid maintains a local *weak acidic environment*
What are the symptoms of Impetigo?
Lesions of 'crusty skin' around mouth or face
Which cells play a role in the second line of defense?
Leukocytes (white blood cells)
•
Library screening methods
What are the symptoms of Dengue fever ?
Life threatening complications: anemia, low blood pressure
What happens to lignin under anaerobic conditions?
Lignin accumulates to form peat bogs and muck soils.
Tumor suppressor genes
Limits the growth of tumors
Yeast PAMPs
Lipoteichoic acid, zymosan, mannose rich glycans
What is the vaccine for Smallpox?
Live Vaccine- bio-terrorism agent
What is the vaccine for Chicken Pox and Shingles ?
Live attenuated vaccine
Hyphae
Long filament of cells divided by cell like crosswalls called septa
What is a memory cell?
Long-lived cell for quick future response to same antigen.
Development of new antimicrobial agents
Long-term solution to antimicrobial resistance relies on the development of new antimicrobial compounds Average new drug = 10-25 years; >$0.5 billion Modification of current antimicrobial compounds is often productive Automated chemistry methods (combinatorial chemistry) have sped up drug discovery 7 million compounds must be screened to find a single useful clinical drug
what means for some viruses, patients show no symptoms,mild symptoms, or occasional symptoms.
Low activity or latency
What are the 2 chemical barriers of mucous membranes and their secretions? How do they work?
Lysozyme - breaks down bonds between NAG and NAM Lactoferrin - binds iron in the body so there's no iron for pathogens
Oral Cavity: Defense against pathogens
Lyzosymes lactoperoxidases) Mucus ciliated epithelial cells
30.6
MMR and Varicella-Zoster Infections
What is the vaccine for Rubella?
MMR vaccine the mothers can get to protect fetus
What is the vaccine for the measles?
MMR vaccine- Live attenuated
•
MRSA and other multi-resistant microbes
What types of leukocytes are involved in phagocytosis?
Macrophages (wandering and fixed) Dendritic cells Neutrophils
What kind of rash is Fifths Disease?
Maculopapular
What type of rash are measles?
Maculopapular
What type of rash is Roseola?
Maculopapular
What type of rash is Rubella?
Maculopapular
What types of rashes are there?
Maculopapular, Vesicular, Large Pustular
HIV envelope
Made of gp120 and gp41
HIV matrix
Made of p17
cells infected with HIV
Main types of cells infected: T helper cells and dendritic cells (including macrophages, microglia)
•
Major antibacterial targets
•
Major classes/groups of toxins (e.g. how can you catalog toxins based on mechanism or target)
•
Major groups of antibiotics and their targets
How does ORT work?
Make a Gatorade type solution (salt, water, packet of stuff) using boiled water (think of drink made for xc)
Campylobacter jejuni
Match the organism with the appropriate term, disease or selective media. Choose the best answer: Curved "seagull wings" gram negative bacilli
Clostridium difficile
Match the organism with the appropriate term, disease or selective media. Choose the best answer: Antibiotic associated diarrhea
Bacteroides fragilis
Match the organism with the appropriate term, disease or selective media. Choose the best answer: BBE agar
Legionella pneumophilia
Match the organism with the appropriate term, disease or selective media. Choose the best answer: BCYE agar
Clostridium botulinum
Match the organism with the appropriate term, disease or selective media. Choose the best answer: Botulinal neurotoxin
Moraxella catarrhalis
Match the organism with the appropriate term, disease or selective media. Choose the best answer: Friable "hockey puck" colony
Clostridium perfringens
Match the organism with the appropriate term, disease or selective media. Choose the best answer: Gas gangrene
Haemophilus influenzae
Match the organism with the appropriate term, disease or selective media. Choose the best answer: Satellite test
Is there a vaccine for Neisseria meningitidis?
Menactra (live attenuated)
What type of disease is Naegleria fowleri?
Meningioencephalitis
•
Meningitis
30.5
Meningitis and Meningococcemia
What is the diagnosis of Leishmaniasis?
Microscopic analysis of host tissue
Dermatophyte examples
Microsporum, Trichophyton, and Epidermophyton
B (Crypto can cause watery diarrhea in immunocompromised or immunocompetent patients, but is obviously more severe in the former - don't let immune status or age or food poisoning possibilities confuse you, acid-fast cysts are almost definitely crypto!) (the treatment is nitazoxanide)
Mike returns from vacation in Eastern Europe. He reports he had been eating new, strange foods, and had been swimming daily in a local river for exercise. He reports that since returning home, he has had daily watery diarrhea. He denies having any blood in his stools. You order a particular stool test, and the results are shown above. What is the cause of Mike's diarrhea? A. Giardia B. Cryptosporidium C. E. coli D. S. aureus E. B. cereus
What is the portal of entry for Dengue fever ?
Mosquitos (Vector)
What is a viral host range?
Most animal viruses only infect and replicate in one host and even one host cell type.
IgG
Most common in serum, appears after IgM, neutralization of toxins and viruses, protects fetus.
Resistance Mechanisms and Spread
Most drug-resistant bacteria isolated from patients contain drug-resistance genes located on R plasmids Evidence indicates that R plasmids predate the antibiotic era The use of antibiotics in medicine, veterinary medicine, and agriculture selects for the spread of R plasmids ----Many examples of overuse of antibiotics ----Antibiotics are used far more often than necessary
Where does most nitrogen occur and how is it found?
Most nitrogen occurs in the atmosphere as N2.
•Enrichment culture and different media types (e.g. general-purpose media, enriched media, selective media, differential media)
Most pathogens can be grown, isolated, and identified with specialized growth media *General-purpose media* Support growth of most aerobic and facultatively aerobic organisms *Enriched media* Contain specific growth factors that enhance growth of selected pathogens *Selective media* Allow some organisms to grow while inhibiting others *Differential media* Allow identification of organisms based on their growth and appearance on the medium
IgM
Most prominent in the circulatory system, first to appear after antigen exposure, found on the surface of B cells.
B (tapeworms/cestodes often do not produce any symptoms, with the exception of a potential B12 deficiency - the other statements are false) (A; this is true for trematodes/flukes - tapeworms are acquired by eating raw or undercooked meat infested with the procercoid) (C; praziquantel or albendazole for everybody!) (D; tapeworms are a global disease, but they are more common in some places than others) (E; humans, and many other mammals, are definitive hosts for tapeworms)
Most tapeworm infections A. Can be caused by swimming in contaminated water B. Are asymptomatic C. Have unique treatments for each species D. Occur only in underdeveloped nations E. Use the human as an accidental host
What is the treatment for Staphylococcal Scalded Skin Syndrome?
Nafcillin, vancomycin (MRSA)
What is a virus called that does not have a membrane?
Naked.
Viral Assembly
Naked: Capsid assembled around the genome Enveloped: Released by budding then glycoproteins added Can bud into ER or other organelles to gain a membrane
Cell Release
Naked: Host cell lysis Enveloped: Released by budding if obtained from membrane, released by exocytosis if envelope came from organelles Some can be transmitted through cell to cell contact
What is the portal of entry for Naegleria fowleri?
Nasal cavity or respiratory tract -Enters body during swimming activities in flagellated or trophozoite stage
•
Natural Vs. synthetic/semisynthetic drugs
•
Natural antibiotics resistance
Obligate Aerobes
Need oxygen because they cannot ferment or respire anaerobically. They gather at the top of the tube where the oxygen concentration is highest.
Gonorrhea (disease of urinary)
Neisseria gonorrheae; G- diplococci; possess pili that allows them to attach to epithelial cells and sperm; endotoxin damages reproductive tract mucosa and produce proteases that destroy IgA antibodies. areas involved can be cervix, urethra, rectum, pharynx, and conjunctiva
What is the most severe form of Meningitis?
Neisseria meningitidis
What are some of the agents that could responsible for Meningitis?
Neisseria meningitidis Streptococcus pneuomoniae Haemophilius influenza Listeria monocytogenes Cryptococcus neoformans (fungus) Coccidioides immitis (fungus) Viral meningitis (Nancy screamed hairy lollipops cause crying)
•
Neisseria meningitidis (meningococcus)
Meningococcal Meningitis (disease of nervous system)
Neissseria meningitidis; gram- diplococci; meningitis symptoms with red pinpoint rash (petechiae)
What is the PNS comprised of?
Nerves Ganglia Schwann cells
•
Neuraminidase inhibitors
•
New drug discovery approaches (e.g. combinatorial chemistry)
What makes nitrogen hard to use?
Nitrogen gas is inert, and it takes a lot of energy to change it. The reaction from N2 → NH3 takes a lot of energy.
What enzyme do bacteria use for nitrogen fixation?
Nitrogenase.
Why would accuracy be important in bacteria?
No backup on another chromosome; only have one copy
How does AIDS effect the host immune system?
No immunity. Results in susceptibility to secondary infections. (ex: pneumonia, tuberculosis, Kaposi's sarcoma) These secondary infections can cause death.
What is the treatment for Naegleria fowleri?
No treatment
What is the treatment for Spongiform Encephalopathy?
No treatment
How can you treat the measles?
No treatment, Clears on its own
How can you treat Rubella?
No treatment, clears on its own
What is the treatment for Roseola?
No treatment, clears on own
What is the treatment for fifths disease?
No treatment, clears on own
What is the treatment for Smallpox?
No treatment, clears own
What is the vaccine for Botulism ?
No vaccine
What is the vaccine for Cellulitis?
No vaccine
What is the vaccine for Impetigo?
No vaccine
What is the vaccine for Listeria monocytogenes?
No vaccine
What is the vaccine for Naegleria fowleri?
No vaccine
What is the vaccine for Ringworm?
No vaccine
What is the vaccine for Roseola?
No vaccine
What is the vaccine for Spongiform Encephalopathy?
No vaccine
What is the vaccine for Staphylococcal Scalded Skin Syndrome?
No vaccine
What is the vaccine for Toxoplasmosis?
No vaccine
What is the vaccine of Leishmaniasis?
No vaccine
What is the recommended treatment for Fifth Disease
No vaccine, no treatment; clears on own
What are features of the innate immune system?
Non-specific response No memory Short-term response
•
Nonnucleoside reverse transcriptase inhibitors (NNRTI)
Normal microbiome (Human Microbiome)
Normal microbial flora ~10^14 microbial cells Microorganisms usually *found associated with human body tissue* Many different habitats Colonized by microorganisms at birth Most microorganisms are *benign* *Few contribute to health, and fewer pose direct threats to health*
water borne pathogen from drinking water
Norovirus
What is Sulfonamides and when would you use it?
Not antibiotics -Early therapy -Bacteriostatic -Blocks folic acid synthesis
How can you tell the difference between MRSA and Staph Aureus?
PCR (genetic test to see if genes are present in bacteria (regular strand vs MRSA)
How can you diagnose Bubonic Plague?
PCR, Serology, blood sample Gram stain (-)
What is the diagnosis for Yellow Fever?
PCR, serology
What is the vaccine for Streptococcus pneumoniae?
PCV13 and PPSV24
What are the symptoms of Poliomyelitis?
Paralysis, weakness in legs -Bulbar polio can cause paralysis of lungs
What is the agent in Fifth Disease?
Parvovirus B19
What do phagocytes use in order to recognize bacteria?
Pathogen associated molecular patterns (PAMPs)
What are exceptions to Koch's postulates?
Pathogen specificity, pathogens can't be grown in lab, some pathogens can cause multiple diseases, etc.
Infectious Disease is caused by *what*
Pathogens
Siderophores
Pathogens can produce iron-chelating compounds
Siderophores
Pathogens can produce iron-chelating compounds (i.e. siderophores)
How do pathogens avoid the adaptive immune system?
Pathogens like HIV can avoid the adaptive immune system altogether.
Types of Microbial infections
Pathogens may grow locally at the site of invasion or may spread throughout the body ---->*Bacteremia:* the presence of bacteria in the bloodstream ---->*Septicemia:* bloodborne systemic infection May lead to massive inflammation, septic shock, and death
Found on a variety of immune cells Causes release of multiple inflammatory molecules
Pattern Recognition Receptor TLR4
What are phagocyte receptors called?
Pattern recognition receptors. (PRRs)
What are the cell wall antibiotic(s)?
Penicillin Monobactams Cephalosporins Vancomycin Bacitracin
Capsomere
Protein subunits forming a capsid
non specific host defenses
Physical barriers Cellular defense Chemical defenses
What is the agent for Malaria?
Plasmodium falciparum (Vector) Mosquito
What is the agent for meningitis Streptococcus pneumoniae?
Pneumococcal meningitis
Examples of opportunistic "infections" *on test*
Pneumocystis jiroveci pneumonia (PCP pneumonia) Kaposi's sarcoma; Tuberculosis; several others
What are some AIDS-defining illnesses?
Pneumonia, HIV-related Encephalitis, Tuberculosis
What is the vaccine for Poliomyelitis?
Polio vaccine, stay away from infected
Which antibiotic(s) target cell membrane?
Polymyxin
What are some common symptoms in Cardiovascular infections?
Poor oxygenation Irregular heartbeat or blood pressure Weakness/fatigue
Vegetative mycelium
Portion of the mycelium that anchors the mold and brings in nutrients
Aerial mycelium
Portion of the mycelium that produces asexual reproductive spores
Is Botulism gram + or - ?
Positive
Is Listeria monocytogenes gram + or - ?
Positive
Is Pneumococcal meningitis gram + or - ?
Positive
prevention of HIV
Practice monogamous sex, avoid shared needles HIV cannot be spread by casual contact, skeeters
•Assays to determine presence of endotoxins
Presence of endotoxin can be detected by the *Limulus amoebocyte (Horseshoe crab) lysate assay*
Actinomyces
The following gram stain is characteristic of this branching anaerobe:
The common cold
Primarily Rhinoviruses, but also Adenovirus, Coronavirus, Parainfluenza virus.
What is the agent in Naegleria fowleri?
Primary amoebic meningioencephalitis (PAM)
Important enzyme in DNA replication An RNA polymerase that makes RNA primers from a DNA template
Primase
processes of non specific host defenses
Processes: Phagocytosis Inflammation
What do basophils do?
Produce histamine which is an allergic response.
What do eosinophils do?
Produce toxic proteins, kill parasites. (helminths)
Where are B cells produced?
Produced and mature in the bone marrow.
What is active humoral immunity?
Produced by a host in response to an antigen. Can be artificial (ex: a vaccine) or natural (ex: catching a cold)
_________ responsible for all natural nitrogen fixation.
Prokaryotes (some archaea and bacteria)
What type of cells do viruses infect?
Prokaryotic and Eukaryotic cells.
72 hours
Proper incubation time for specimens being screened for Campylobacter jejuni is
What is bacteriophage DNA called that incorporates into a bacterial host chromosome?
Prophage.
What is the treatment for Malaria?
Prophylactic antiprotozoal Quinine (tonic water)
•
Pros and cons
•
Protease inhibitors
What are the symptoms of Cellulitis?
Red Inflamed skin around large pus filled bump (infection has spread to the surrounding tissue and the immune system is yet to respond)
Blood typing using agglutination
Red blood cells contain antigens on their surface (A = A antigen, B = B antigen) Your body contains antibodies to the opposite blood type, can cause agglutination in your blood stream if given an incompatible blood type during a transfusion (SEE PICTURE IN POWERPOINT)
What are the symptoms of Fifth Disease
Red cheeks
How does inflammation recruit phagocytes to the infection site?
Redness & heat causes increased permeability and vasodilation (increase in diameter of blood vessels) Allows phagocytes to leave the blood and enter injured tissue. (also brings fluid into the injured tissue)
Epitope
Region of the antigen that reacts with the antibody
•Specificity and sensitivity of a diagnostic test
Reliability of a diagnostic test depends on:
Latent life cycle
Remain within the host cells without replicating or causing harm due to established miRNA
RNA Genome
Replicate in cytoplasm
DNA Genome Release
Replicate in the nucleus Uncoated: Goes through nuclear pores Coated: Capsid binds to the pores and dumps in genomes, entire capsid fits through pore then removed in nucleus
Living virus characteristics
Reproduce and mutate
always found on human tissues.
Resident microbiota
Resilience of host
Resistant normal microbiota Healthy immune response Age (very young, old, more susceptible) Genetics factors
What is the portal of entry for Chicken Pox and Shingles ?
Respiratory Tract
What is the portal of entry for Haemophilus influenzae?
Respiratory Tract
What is the portal of entry for Smallpox?
Respiratory Tract
What is the portal of entry for Toxoplasmosis?
Respiratory Tract
What is the portal of entry in Poliomyelitis?
Respiratory Tract
What is the portal of entry for Coccidioides?
Respiratory Tract (Spores released into the lungs, move to bloodstream )
•
Respiratory infections (different types of respiratory pathogens)
What is the portal of entry for Rubella?
Respiratory system
What is the portal of entry for the measles?
Respiratory system
What is the portal of entry for Infectious Mononucleosis?
Respiratory tract
Rocky Mountain Spotted Fever (RMSF) (disease of skin)
Ricksettsia rickettsii; transmitted by bite of ticks; fever,chills, headache, muscular pain. distinctive spotted rash (petechiae) rash present on hands and soles. disease affects cells lining blood vessels.
Antibiotic(s) that target nucleic acid
Rifamycin Quinolones Fluoroquinolones
(diarrhea, children and infants)
Rotavirus
cause of German measles Major cause of viral birth defects; vaccination campaign has made it 2nd to CMV.
Rubella
•
Rubella (togavirus)
What are the symptoms of Chagas disease?
Rupture of blood cells Chagoma (swelling at the site of insect bite)
Sulfur reduction equation.
SO4 → S2
STAPHYLOCOCCAL skin infection
STAPHYLOCOCCUS PYOGENIC- Pus producing TOXIC SHOCK SYNDROME - TSS TOXIN
portals of entry - transplacental
STORCH (syphillis, toxoplasmosis, other, rubella, cytomegalovirus, and herpes simplex virus
Salmonellosis (disease of digestive)
Salmonella enterica; diarrhea, cramps, fever, vomiting, dehydration or systemic bloodborne infection
Typhoid fever (disease of digestive)
Salmonella typhi; high fever; enlarged lymph nodes, liver and spleen and ulcerated intestine; transmission-fecal oral route
What are the symptoms of Rubella in adults?
Same as regular measles (Bumps, Laryngitis, etc)
Describe some ways specimens can be collected
Samples are usually collected from bodily fluids and must be cold and moist. Samples can be stored immediately (preferred), in a culturette (must be kept moist), or dry and in an aerobic/anaerobic environment.
What is a plasma cell?
Secrete IgM and then IgG into bloodstream (Isotype/class switching - a B cells production of antibody changes from one class to another)
IgA
Secretory immunoglobulin, found in secretions (tears, mucus, saliva, breast milk) Newborn protection against GI pathogens.
C (this is the classic 'flask shaped' lesion in the colon that indicates infection by Entamoeba histolytica, which can progress to the liver if left untreated - under microscopy, these amoeba will have ingested red blood cells in their cytoplasm, so they are distinct there as well)
Shamim traveled to Latin America presents with abdominal pain and bloody diarrhea. A biopsy of the colon revealed the histological finding shown. If treatment is withheld, Shamim is in danger of developing which of the following conditions? A. Aplastic anemia B. Neurocysticercosis C. Hepatic amoebiasis D. Bladder cancer E. Cholangiocarcinoma
Shigellosis (disease of digestive)
Shigella dysenteriae; bacteria cause lysis of infected colon cells=patchy areas of destruction and inflammation; shiga toxin=damage to blood vessels in intestinal wall and intense inflammation; transmission-fecal oral route; bacteria invades large intestine
In translation, the specific sequence that tells ribosome where to bind; looks for UG to start
Shine Dalgarno Sequence
What are cationic antimicrobial peptides? How do they protect us from pathogens?
Small cationic peptides made by the host. Inhibit cell wall synthesis Form pores in pathogens plasma membrane and destroy pathogen nucleic acids.
What are interferons? How do they protect us from pathogens?
Small proteins, stable at low pH and high heat. (they work well in fever conditions) Cell infected with a virus make interferons (sacrifice itself for the greater good). Bind to healthy cells and cause them to make antiviral proteins, protects them from potential infection.
Viroid
Small, circular, single stranded molecules of infectious RNA lacking a protein coat EX: Potato spindle tuber disease
True
T/F: Haemophilus influenzae is spread person to person.
Why are viral proteins potentially useful?
Some viral proteins are unique to viruses and can be targets for antiviral drugs.
What encompasses the third line of defense?
Specialized lymphocytes (T and B cells) Antibodies
Hemagglutinin and Neuraminidase are what
Spikes
How do you diagnose Meningitis?
Spinal tap
Disc diffusion test
Standard procedure for assessing antimicrobial activity Agar media is spread evenly with culture of bacteria Inhibition zones Used to determine an organism's susceptibility to an antimicrobial agent
What is the agent for Cellulitis?
Staph Aureus
What is the agent in Staphylococcal Scalded Skin Syndrome?
Staph aureus
Rheumatic fever (disease of the respiratory)
Streptococccus pyogenes; autoimmune reaction results in antibodies directed @ strep to cross react w heart antigens. affects joints, skin, brain, and children.
•
Streptococcus pneumoniae (pneumonia)
Pneumococcal Meningitis (disease of nervous system)
Streptococcus pneumoniae; gram + diplococci (sometimes chains)
•
Streptococcus pyogenes diseases (strep throat, erysipelas, scarlet fever, rheumatic fever, necrotizing fasciitis etc)
Scarlet fever (disease of respiratory)
Streptococcus pyogenes strains that produce the erythrogenic (pyrogenic) exotoxin. pinkish-red rash on whole body except palms and soles , feels like sandpaper, tongue spotted like strawberry
•Growth factor analogs
Structurally similar to growth factors but do not function in the cell --Analogs similar to vitamins, amino acids, and other compounds
•
Structure-activity relationship
deal with host defense
Successful parasite must infect, persist long enough to reproduce, then escape. Host defense seeks to kill it.
Sulfur oxidation equation?
Sulfide and sulfur can be oxidized to H2SO4 → corrosion Also used for recovery of Cu from sulfide ores
________ binds tightly to a lot of metals?
Sulfide.
Sulfa Drugs
Sulfonamides Discovered in the 1930s First widely used growth factor analogs Inhibit growth of bacteria Sulfanilamide is the simplest Blocks the synthesis of folic acid inhibit nucleic acid synthesis Sulfa allergies are common
What is the treatment for Acanthamoeba?
Symptom management
How do you diagnose Tetanus?
Symptoms
How do you diagnose rabies?
Symptoms
What are the symptoms of Hemorrhagic Fever ?
Symptoms range in severity based on viral agent
Clonal expansion
T & B cells replicate themselves
Where are T cells made? Where do they mature?
T cells are made in the bone marrow. They mature in the thymus (thymic selection)
Activation
T cells mature to T-cytotoxic (CD8) cells, kill cells B cells become Plasma cells, make Aby
*what* cells help B & T cells thru direct contact
T helper
*what* either kill cells directly or recruit macrophages to kill cells directly
T lymphocytes
T and B cells receive signals from *what*
T-Helper cells
False
T/F: All specimens received for anaerobic culture should be plated and incubated for 18-24 hours.
True
T/F: Campylobacter jejuni, Moraxella catarrhalis and Neisseria meningitidis are all oxidase positive.
-
Tetanus toxin
D (Cryptosporidium is fecal-oral and transmitted by contaminated water - it will cause mild watery diarrhea in the immunocompetent, and is markedly more severe in the immunocompromised)
The CDC investigates a series of outbreaks of cryptosporidiosis. The cluster of cases occurs in separate upscale suburban communities. Which of the following is the most plausible source of transmission for these outbreaks? A. Fecal-oral person-to-person route B. Parasite-infected meat C. Bird-mosquito-human route D. Manure-recontaminated drinking water E. Inhalation of bird droppings
30.7
The Common Cold
when was the use of biological warfare banned (by what protocol)
The Geneva Protocol-1925
•
The R plasmid (HGT spread, enzymes that give antibiotics resistance)
Antimicrobial Drug Resistance
The acquired ability of a microorganism to resist the effects of a chemotherapeutic agent to which it is normally sensitive
C (the hydatid cysts are found in sheep or humans, which are accidental intermediate hosts, but the adults are found in the intestines of dogs)
The adult tapeworm of Echinococcus granulosus is found in the intestine of A. Humans B. Sheep C. Dogs D. Cattle
What reaction occurs in nitrification?
The aerobic 2-step process of ammonia oxidation NH3 → NO2 nitrite oxidation NO2 → NO3
Carbohydrate fermentation
The boxed in area of this test kit represents the following reaction
A (cutaneous larva migrans, or "creeping eruption" is indicative of a pathogen that has infected its non-definitive host - Ancylostoma is a dog hookworm, and is unable to do anything to a human host other than aimlessly wander through the skin - aww, poor baby is lost...)
The causative agent of this particular lesion is most likely of the genus A. Ancylostoma B. Ascarida C. Dermacentor D. Strongyloides E. Diphyllobothrium
What is cell mediated immunity?
The cellular response to tumors. Fights intracellular pathogens, responsible for transplant rejection.
•
The common cold (rhinoviruses)
What is ammonification?
The conversion of organic nitrogen into inorganic ammonia.
6)
The emergence of multi-drug-resistant pathogens has become a major health care problem. What are some ways we can deal with these antibiotics resistant strains?
NAD
The factor present in several bacteria and yeast and is responsible for satellite growth of Haemophilus sp. around colonies of Staphylococcus species on sheep blood agar is
C (which is insane)
The fever generated by Plasmodium vivax malaria consumes ____ calories a day. A. 1,000 B. 3,000 C. 5,000 D. 10,000 E. 2
What is the first step of DNA replication?
The first step in DNA replication is to 'unzip' the double helix structure of the DNA molecule.
M. catarrhalis
The following biochemical profile would most likely belong to: Glucose-negative, maltose-negative, lactose-negative , sucrose negative, and DNase positive
D (the giant intestinal fluke is commonly found in Southern and Eastern Asia but considerably rarer in the US)
The following organisms are linked with specific relevant information. The incorrect combination is A. Strongyloides stercoralis ::: internal autoinfection B. Echinococcus granulosus ::: hydatid disease C. Taenia solium ::: humans can serve as definitive and intermediate hosts D. Fasciolopsis buski ::: common within the United States
Fusobacterium
The gram stain shown below is consistent with:
A (this is a tapeworm, and the anterior structure is a scolex - the scolex is NOT used to feed, but to attach to the intestine - nutrients are absorbed directly through the tapeworm's surface, as it does not have or need a digestive system)
The image shows the anterior end of a cestode. The structure at the tip of the organism is used for A. Attachment B. Reproduction C. Feeding D. Looking scary
What is phagocytosis?
The ingestion of a microorganism or other substances by a cell.
Clostridium botulinum
The ingestion of honey is not recommended for young children because it may contain the spores of this anaerobic organism.
What is a primary infection?
The initial infection.
contamination of food, milk, or water with animal feces.
The majority of human infections with Campylobacter species are caused by:
D (normally, the filariform larva penetrates the body and the rhabditiform larva leaves it - but Strongyloides stercoralis is able to make the switch from one to the other within the same host, and reinfect and multiply to absurd numbers) (A; hyperparasitism is when a parasite is a host to another parasite, like a tapeworm inside a flea) (E; man, I wish)
The nematode Strongyloides stercoralis is one of the only parasites known to be capable of A. Hyperparasitism B. Hyperinfection C. Autoparasitism D. Autoinfection E. Megahypersuperultrarareparasitinfectionism
D (Ascarid worm eggs are remarkably resilient, lasting viable for up to 10 years, and a lipid layer surrounding the egg protects it from destruction)
The only way to destroy Ascaris lumbricoides eggs is to A. Freeze them B. Dessicate them C. Bathe them in formaldehyde D. You can't
42 degrees C
The optimum temperature for the primary cultivation of Campylobacter jejuni is:
Campylobacter jejuni
The organism in this gram stain is most likely to be identified as:
Clostridium perfringens
The organism in this gram stain is most likely to be identified as:
How do pathogens actively penetrate host tissue?
The pathogen has its own mechanism for penetration.
B (notable features - large kidney-shaped nucleus [macronucleus] and cilia) (symptoms are similar to entamoebiasis, but the buzzword is pigs - look out for farmers and such) (treatment is tetracycline or metronidazole)
The pictured organism is from the genus A. Ascarida B. Balantidia C. Cyclospora D. Diphyllobothria E. Entamoeba
C (this is Entamoeba histolytica, as evidenced by the endocytosed red blood cells, which colonizes the large intestine, often the cecum) (D; E. histolytica does eventually get to the liver via the portal system to create abscesses, but that is not the initial site)
The pictured pathogen initially infects the A. Duodenum B. Ilium C. Cecum D. Liver
What is the polymerase chain reaction?
The polymerase chain reaction (PCR) is a technique used in genotyping, to amplify a single copy or a few copies of a piece of DNA across several orders of magnitude, generating thousands to millions of copies of a particular DNA sequence.
Gastric Biopsy
The proper specimen to collect when Helicobactor pylori is suspected is:
What happens if an enveloped virus loses its envelope?
The virus becomes non-infectious.
What is an immunoglobulin?
These bind and destroy antigens but alone are not harmful. Mark pathogens for destruction by complements, phagocytes, other effects.
How do peristalsis and defecation aid in reducing pathogens?
They act as a physical barrier. Propel food and microbes out of the GI tract.
How do urine flow and vaginal secretions aid in reducing pathogens?
They act as a physical barrier. They physically flush microbes from the body.
Are nucleic acids DNA or RNA?
They can be either.
How many strands are nucleic acids?
They can be single-stranded (ss) or double stranded (ds).
How can Mycobacteria, Legionella, and Listeria evade the immune system?
They can multiply within phagocytes.
What do Th cells do?
They help to activate B cells (in the place of an antibody in some cases) They release cytokines that activate B cells and enhance immune response. They also produce memory cells.
How do viruses reproduce?
They must infect living cells.
How do T cells bind to antigens?
They use T cell receptors (and not antibodies)
fungi
This list describes the functions of _____________ in nature: 1. decompose organic matter 2. provide food for humans (mushrooms) 3. provide food for ants (lepiota fungi) used in brewing, wine making, and bread making 4. source of industrial chemicals and enzymes 5. source of drugs (penicillin)
Haemophilus influenzae
This organism is known to be the major cause of bacterial meningitis worldwide among non-vaccinated children.
Neisseria meningitidis
This organism may cause lethal meningitis but is commonly carried in the respiratory tract of humans and may be asymptomatic.
Clostridium botulinum
This organism produces a powerful preformed toxin which may be ingested with food and lead to respiratory paralysis:
Neisseria gonorrhoeae
This oxidase positive, gram-negative diplococci is not part of normal human flora and is spread person to person by sexual contact.
What are molecular postulates?
This postulates relies on the presence of a gene found only in pathogenic strains. (This gene must be absent in non-pathogenic members of the same genus/species)
Gram negative coccobacilli
This typical gram stain of Haemophilus influenzae is best described as:
How do pathogens attach to host tissue?
Through adhesion, pathogens have adhesins or ligands that specifically bind to host receptors.
How do you identify an infectious agent?
Through culturing bacteria, isolating pure cultures, and diagnostic tests (5 "I's" of Culturing Bacteria-Inoculation, Incubation, Isolation, Inspection, Identification)
What are the 3 physical barriers of the skin?
Tightly packed Waterproof Sheds
Dermatophyte infections
Tinea capitis, tinea barbae, tinea corporis
Mycelium
Total mass of hyphae
What is the agent in Toxoplasmosis?
Toxoplasma gondii (Apicomplex)
Toxoplasma gondii
Toxoplasmosis (nervous system) Non motile
What are virulence factors?
Traits or features that act as pathogen weapons.
come and go, can include potential pathogens.
Transient microbiota
C (rule of thumb; protozoans are transmitted either fecal-orally, or by arthropod vectors - and the tsetse fly vectors something much scarier than cyclosporiasis)
Transmission of cyclosporiasis occurs by what method? A. Person-to-person B. Through the bite of the tsetse fly C. Fecal-oral route D. Respiratory route E. Sexual transmission
simple life cycle
Transmission to host and entry into host Attachment to host cell, entry into cell Transcription and translation, replication, assembly Lysis of cell or budding to release virus from host cell Exit from host organism, spread
Pneumocystis jiroveci
Transmitted by respiratory route and is often asymptomatic Can cause severe PCP (pneumonia)
What is Neonatal meningitis?
Transmitted during birth, or in utero *Opportunistic Pathogen*
What is the portal of entry for Listeria monocytogenes?
Transmitted through food - Ingestion
•
Treatment
•
Treatment and vaccination
Syphilis (disease of urinary)
Treponema pallidum; 3 phases; 1 phase- chancres 2 phase- rash phase 3- damage to all organs and death. treatable with penicillin
true or false Some diseases are found only in some geographic areas
True
true or false Some bacteria specialize in disease, some are opportunists
True
30.4
Tuberculosis and Leprosy
A (the fish tapeworm egg resembles the liver fluke egg - both are operculated [meaning they have little caps] - you need a specialist to distinguish them - D latum is the ONLY cestode that infects humans and lays operculated eggs)
Two helminth eggs that may resemble one another are A. Diphyllobothrium latum and Fasciola hepatica B. Opisthorchis sinensis and Taenia solium C. Taenia saginata and Hymenolepis nana D. Ascaris lumbricoides and Trichostrongylus
Which hypersensitivity type involves the IgE on mast cells binding to antigens and releasing histamine? (ex: allergic response)
Type I hypersensitivity.
What are the 4 types of hypersensitivity?
Type I: Immediate or anaphylactic Type II: Antibody-dependent cytotoxic Type III: Immune-complex Type IV: Delayed or cell-mediated
Which hypersensitivity type involves the combination of IgM and IgG on cell surfaces that leads to cell death by complement activation or phagocytosis? (ex: blood group incompatibility)
Type II hypersensitivity.
Which hypersensitivity type involves a small antigen-antibody combination (called an immune complex) that is deposited in the organs which results in surrounding host cell/tissue destruction?
Type III hypersensitivity.
Which hypersensitivity type involves T cells bound to antigens that release cytokines to cause inflammatory response and phagocytosis that damages tissue? (ex: poison ivy or latex allergy)
Type IV hypersensitivity.
Disease: TULAREMIA Causative organism: FANCISCELLA
Type of Organism: BACTERIAL Part of Body Affected: Cardiovascular & lymphatic system Mode of Transmission: 1. TICK BITES- DIRECT CONTACT with INFECTED ANIMALS: ULCEROGLANDULAR TULAREMIA 2. INHALATION: PNEUMONIC TULAREMIA 3. EATING INFECTED/ contaminated MEAT: TYPHOID TULAREMIA 4. EYE: OCULOGLANDULAR TULAREMIA Characteristic Sign or Symptom: Skin ulcer, septicemia, abscesses
Disease: NEISSERIA MENINGITIS/ MENINGOCOCCAL MENINGITIS Causative organism: NEISSERIA MENINGITIDIS
Type of Organism: Bacteria Part of Body Affected: Nervous System Characteristic Sign or Symptom: rash and throat infection Notes: primarily in young adults
Disease: LISTERIOSIS Causative organism: LISTERIA monocytogenes
Type of Organism: Bacteria Part of Body Affected: Nervous System Mode of Transmission: Dairy, eating animal products Characteristic Sign or Symptom: headache, stiff neck, delirium Complications: coma Notes: NEONATAL and FETAL INFECTIONS* *crosses the PLACENTA
Disease: BOTULISM Causative organism: CLOSTRIDIUM
Type of Organism: Bacteria Part of Body Affected: Nervous System Mode of Transmission: INGESTION of EXOTOXINS in FOOD* Mechanism: BOTULIN- BLOCKS the RELEASE of ACETYLCHOLINE* Complications: lethal Notes: Infant botulism - cases from HONEY HONEY should NOT be FED to INFANTS <1 y.o.
Disease: LEPROSY -HANSONS DISEASE Causative organism: MYCOBACTERIUM
Type of Organism: Bacteria Part of Body Affected: Nervous System Mode of Transmission: respiratory, skin contact Characteristic Sign or Symptom: 1. NEURAL Form: TUBERCULOID 2. PROGRESSIVE Form: LEPROMATOUS/LEPROMAS- disfiguring nodules
Disease: TETANUS Causative organism: CLOSTRIDIUM
Type of Organism: Bacteria Part of Body Affected: Nervous System Mode of Transmission: soil Characteristic Sign: TETANOSPASMIN* (exotoxin) sustained muscle contraction Symptom: LOCKJAW /RISUS SARDONICUS* Prevention: TDap* (older children and adults) or DTap* (children)
Disease: Bacterial Meningitis Causative organism: HAEMOPHILUS INFLUENZAE MENINGITIS
Type of Organism: Bacteria Part of Body Affected: Nervous System Notes: Occurs in children (6 mos - 4 yrs)
Disease: STREPTOCOCCUS PNEUMONIA MENINGITIS
Type of Organism: Bacteria Part of Body Affected: Nervous System Notes: most common in children 1 mo-4 yrs Prevention: VACCINATION*
Disease: GROUP A STREPTOCOCCAL PNEUMONIA Causative organism: STREPTOCOCCUS PYOGENS
Type of Organism: Bacterial Part of Body Affected: Lower Respiratory
Disease: PERTUSSIS - WHOOPING COUGH Causative organism: BORDETELLA
Type of Organism: Bacterial Part of Body Affected: Lower Respiratory Mode of Transmission: INHALATION Characteristic Sign or Symptom: 1. CATARRHAL stage: HIGHLY CONTAGIOUS 2. PAROXYSMAL stage: lMPEDES CILIARY ACTION ➡️ SEVERE, SPASMODIC and RECURRENT/EPISODIC COUGH ➡️ ANOXIC-SHORT of OXYGEN 3. CONVALESCENT stage: lasts for MONTHS Complications: CONJUNCTIVAL HEMORRHAGE, DETACHED RETINA - may result in BLINDNESS, RECTAL PROLAPSE, HERNIA, CRACKED RIBS* Prevention: DTaP (a= ACELLULAR)*
Disease: PNEUMOCOCCAL PNEUMONIA Causative organism: STREPTOCOCCUS
Type of Organism: Bacterial Part of Body Affected: Lower Respiratory Mode of Transmission: INHALATION Characteristic Sign or Symptom: OTITIS MEDIA Prevention: PNEUMOCOCCAL POLYSACCHARIDE VACCINE
Disease: HAEMOPHILUS PNEUMONIA Causative organism: HAEMOPHILUS
Type of Organism: Bacterial Part of Body Affected: Lower Respiratory Complications: meningitis, OBSTRUCTIVE EPIGLOTTIS (OBSTRUCTION of the TRACHEA ➡️ TRACHEOTOMY required)
Disease: LEGIONNAIRES DISEASE/LEGIONELLOSIS Causative organism: LEGIONELLA
Type of Organism: Bacterial Part of Body Affected: Lower Respiratory Mode of Transmission: INHALATION, WATER*- LIVE & MULTIPLY in WATERBORNE PROTOZOA Notes: PONTIAC FEVOR: MILDER form of LEGIONELLOSIS
Disease: SERRATIA PNEUMONIA Causative organism: SERRATIA
Type of Organism: Bacterial Part of Body Affected: Lower Respiratory Mode of Transmission: OPERATION SEASPRAY*-biological weapon
flagellin are what *what* are made of
flagella
Disease: KLEBSIELLA PNEUMONIA Causative organism: KLEBSIELLA
Type of Organism: Bacterial Part of Body Affected: Lower Respiratory Mode of Transmission: RESPIRATORY and NOSOCOMIAL infections Characteristic Sign: MALNUTRITION Notes: MALE ALCOHOLICS over 40 y.o. MOST SUSCEPTIBLE
Disease: TUBERCULOSIS Causative organism: MYCOBACTERIUM
Type of Organism: Bacterial Part of Body Affected: TUBERCLE*-body walls of the lung Mode of Transmission: droplet route Characteristic Sign or Symptom: CASEOUS LESION* ➡️ GHON COMPLEXES- SCAR FORMATIONS and CALCIFICATION seen on CHEST X-RAY Complications: MILIARY (spread) TUBERCULOSIS/CONSUMPTION* Prevention: VACCINE- BCG* (bacillus of Calmette and Guerin)
Disease: STREP THROAT Causative organism: STREPTOCOCCUS -Group A strep
Type of Organism: Bacterial Part of Body Affected: Upper Respiratory Mode of Transmission: Characteristic Sign or Symptom: PHARYNGITIS, INFLAMMATION of MUCUS MEMBRANES of the THROAT, TONSILS* Complications: SCARLET FEVER*
Disease: DIPHTHERIA Causative organism: CORYNEBACTERIUM
Type of Organism: Bacterial Part of Body Affected: Upper Respiratory Mode of Transmission: INHALATION of Droplets Characteristic Sign or Symptom: PSEUDOMEMBRANE LEATHER/MEMBRANE Prevention: DPT, DtaP D Vaccine
Disease: COCCIDIOMYCOSIS Common name: SAN JOAQUIN VALLEY FEVER, VALLEY FEVER Causative organism: COCCIDIOIDES
Type of Organism: FUNGAL Part of Body Affected: Lower Respiratory Mode of Transmission: INHALATION of SPORES Complications: Notes: DIMORPHIC FUNGUS
Disease: Candidiasis Causative organism: CANDIDA
Type of Organism: FUNGAL Part of Body Affected: skin Mode of Transmission: sugary diet -Opportunistic Characteristic Sign or Symptom: WIDESPREAD YEAST*- oral cavity, genitalia & large intestine Complications: Fatal
Disease: RESPIRATORY SYNCYTIAL VIRUS (RSV)
Type of Organism: Viral Part of Body Affected: Lower Respiratory - to blame for 1:13 PEDIATRICIAN VISITS - STRIKES HEALTHY KIDS, and it affects MORE youngsters than the FLU
Disease: Q FEVER - QUERY Causative organism: COXIELLA
Type of Organism: Viral Part of Body Affected: Lower Respiratory Mode of Transmission: ANIMAL to ANIMAL by TICKS, INHALING CONTAMINATED DUST, TICK or CATTLE FECES, RESPIRATORY DROPLETS, UNPASTEURIZED MILK, TICK BITE to HUMANS-RARE Notes:
Disease: PSITTACOSIS/ORNITHOSIS Causative organism: CHLAMYDIA
Type of Organism: Viral Part of Body Affected: Lower Respiratory Mode of Transmission: INHAILING organism in airborne DUST/BIRD DROPPINGS, BITE of INFECTED BIRD or DROPLETS from an INFECTED HUMAN Complications: EXTENSIVE PULMONARY INFECTION ➡️ CYANOSIS
Disease: INFLUENZA
Type of Organism: Viral Part of Body Affected: Lower Respiratory Mode of Transmission: INHALATION, RESPIRATORY DROPLETS, INDIRECT CONTACT Complications: REYES SYNDROME ANTIGENIC VARIATION: H SPIKE: HEMMAGGLUTININ - BINDS to HOST CELLS N SPIKES: NEURAMINIDASE- ASSISTS in VIRAL BUDDING and RELEASE TYPE A: MOST PANDEMICS TYPE B: EVERY 4-6 YRS, GEOGRAPHICALLY LIMITED TYPE C: ONLY SPORADIC CASES DRIFT: minor mutations in the genes that code for the H & N spikes SHIFT: reassortment of RNA from different strains
Disease: MYCOPLASMA PNEUMONIA Common name: WALKING PNEUMONIA Causative organism: MYCOPLASMA
Type of Organism: Viral Part of Body Affected: Lower Respiratory SMALLEST FREE LIVING CELLULAR ORGANISM, LACK CELL WALLS - MOST COMMON ATYPICAL PNEUMONIA - PAP
Causative organism: RHABDOVIRUS
Type of Organism: Viral Part of Body Affected: Nervous System Mode of Transmission: ENTERS through WOUND CONTAMINATED with SALIVA - ANIMAL BITE Characteristic Sign or Symptom: Complications: HYDROPHOBIA -can't swallow
Disease: POLIOMYELITIS Causative organism: PICORNAVIRUS
Type of Organism: Viral Part of Body Affected: Nervous System Mode of Transmission: FOOD or WATER* Characteristic Sign or Symptom: fever, headache, sore throat, vomiting Complications: paralysis Pathology: MOST cases DON'T PROGRESS - NEUROTROPHIC* Prevention: 1. SALK - CONTAINS KILLED POLIOVIRUS 2. SABIN - ORAL VACCINE: WEAKEND/ATTENUATED LIVE VIRUS * TYPE III may REVERT to VIRULENCE
Disease: Arboviral encephalitis Causative organism: ENCEPHALITIS
Type of Organism: Viral Part of Body Affected: Nervous System Mode of Transmission: MOSQUITO Characteristic Sign or Symptom: chills, headache, fever, mental confusion, coma Complications: Brain damage and deafness Notes: ST. LEWIS (SLE) & WEST NILE (WNV)
INPETIGO of the NEWBORN Causative organism: staphylococcal
Type of Organism: bacteria Part of Body Affected: skin Characteristic Sign/Symptom: superficial pustule Complications: OSTEOMYELITIS* Notes: PEMPHIGUS NEONATORUM*
Otitis Externa -SWIMMERS EAR Causative organism: PSEUDOMONAS
Type of Organism: bacteria Part of Body Affected: skin Mode of Transmission: Direct contact/contaminated water Characteristic Sign or Symptom: fatal infections due to exotoxin A Complications: major cause of nosocomial infections
Inflammatory acne Causative organism: PROPIONIBACTERIUM
Type of Organism: bacteria Part of Body Affected: skin Mode of Transmission: opportunist Characteristic Sign or Symptom: ACNE LESIONS
ERYSIPELAS- ST ANTHONY's FIRE Causative organism: STREPTOCOCCAL
Type of Organism: bacteria Part of Body Affected: skin, lymphatic vessels Characteristic Sign or Symptom: PAINFUL REDDISH, SWELLING PATCHES (rea)
SCARLET FEVER Causative organism: streptococcal
Type of Organism: bacteria (lymphatic vessels) Part of Body Affected: skin Characteristic Sign or Symptom: STRAWBERRY TONGUE Complications: NECROTIZING FASCIITIS Notes: EXOTOXIN A - SUPERANTIGEN
Disease: ENDOCARDITIS Causative organism: ALL STAPHS and STREPS
Type of Organism: bacterial Part of Body Affected: Cardiovascular & lymphatic system 1. Subacute bacterial endocarditis 2. ACUTE BACTERIAL ENDOCARDITIS - STAPH* PERICARDITIS - STREP*
Disease: BUBONIC PLAGUE/BLACK DEATH Causative organism: YERSINA
Type of Organism: bacterial Part of Body Affected: Cardiovascular & lymphatic system Mode of Transmission: FLEA Characteristic Sign or Symptom: BUBO= SWELLING Complications: fatal Notes: 1. SYLVATIC PLAGUE- ZOONOSIS 2. SEPTICEMIA PLAGUE-FLEA bite 3. PNEUMONIC PLAGUE- HUMAN to HUMAN by DROPLET ROUTE
Disease: EPIDEMIC TYPHUS Causative organism: RICKETTSIA
Type of Organism: bacterial Part of Body Affected: Cardiovascular & lymphatic system Mode of Transmission: LICE* or feces Characteristic Sign or Symptom: rash Complications: fatal, FRIGHTFUL DREAMS, HALLUCINATIONS and DELIRIUM
Disease: EPIDEMIC MURINE TYPHUS Causative organism: RICKETTSIA
Type of Organism: bacterial Part of Body Affected: Cardiovascular & lymphatic system Mode of Transmission: RODENTS & FLEAS Characteristic Sign or Symptom: rash Notes: less severe than epidemic typhus
Disease: ANTHRAX Causative organism: BACILLUS
Type of Organism: bacterial Part of Body Affected: Cardiovascular & lymphatic system Mode of Transmission: Spores Complications: mortality Notes: 1. Cutaneous Anthrax- 20% mortality 2. Gastrointestinal Anthrax- 50% mortality 3. Inhalation Anthrax - 100% mortality
Disease: LYME DISEASE Causative organism: BORRELIA
Type of Organism: bacterial Part of Body Affected: Cardiovascular & lymphatic system Mode of Transmission: TICK Characteristic Sign or Symptom: ERYTHEMA CHRONICUM MIGRAINES Complications: DESTRUCTION of MYELIN SHEATH - CROSS the PLACENTA
Disease: ROCKY MOUNTAIN SPOTTED FEVER Causative organism: RICKETTSIA
Type of Organism: bacterial Part of Body Affected: Cardiovascular & lymphatic system Mode of Transmission: TICKS Characteristic Sign or Symptom: MACROPAPULAR RASH Complications: fatal Notes: vaccine available. Protective clothing and insect repellent
Disease: BRUCELLOSIS Causative organism: BRUCELLA
Type of Organism: bacterial Part of Body Affected: Cardiovascular & lymphatic system Mode of Transmission: mouth; CONTAMINATED DAIRY products, UNDERCOOKED MEAT, CATTLE SECRETIONS or EXCRETIONS in CUTS or eyes Characteristic Sign or Symptom: UNDULANT FEVER*- INCREASES in the EVENING Notes: Hepatosplenomegaly
Disease: HEPATITIS D- DELTA VIRUS
Type of Organism: bacterial Part of Body Affected: Digestive System Mode of Transmission: CONFECTION HBV Signs and symptoms: JAUNDICE, CHRONIC HEPATITIS, CIRRHOSIS and DEATH
Disease: MUMPS or PAROTITIS Causative organism: RUBULAVIRUS
Type of Organism: bacterial Part of Body Affected: Digestive System Mode of Transmission: DROPLETS, FOMITE's Signs and symptoms: PAINFUL SWELLING of the PAROTID GLANDS Prevention: MMR, MEASLES- MUMPS - RUBELLA
Disease: CHOLERA Causative organism: VIBRIO
Type of Organism: bacterial Part of Body Affected: Digestive System Mode of Transmission: CONTAMINATED FOOD and WATER Signs and symptoms: 1. ENTEROTOXIN ➡️ "RICE WATER STOOLS" ➡️ SUDDEN LOSS OF FLUID ➡️ BLOOD THICKENS 2. VISCOUS BLOOD: ORGAN DYSFUNCTION, VOMITING, DECREASED BLOOD to BRAIN ➡️ SHOCK ➡️ COLLAPSE ➡️ COMA Complications: ➡️ DEATH Notes: food infection
Disease: TYPHOID FEVER Causative organism: SALMONELLA
Type of Organism: bacterial Part of Body Affected: Digestive System Mode of Transmission: CONTAMINATED FOOD and WATER. FLIES, FOOD HANDLERS- "TYPHOID MARY" Signs and symptoms: Complications: can LEAVE the G.I. TRACKED - DISSEMINATE and INVADE OTHER ORGANS Prevention: VACCINE, CHOLECYSTECTOMY of CARRIERS Notes: food infection
Disease: HEPATITIS B- SERUM HEPATITIS
Type of Organism: bacterial Part of Body Affected: Digestive System Mode of Transmission: IV DRUG USE, NEED STICK, TATTOOS or BODY PIERCINGS, SEXUAL CONTACT, DIALYSIS, BLOOD TRANSFUSIONS (before 1972), FOMITES (survives several days) Signs and symptoms: JAUNDICE Prevention: VACCINE Notes: OFTEN PROGRESSES to CIRRHOSIS, CHRONIC HEPATITIS, LIVER CANCER and DEATH
Disease: BACILLIARY DYSENTERY/ SHIGELLOSIS Causative organism: SHIGELLA- SHIGA TOXIN
Type of Organism: bacterial Part of Body Affected: Digestive System Mode of Transmission: ORAL, BY MOUTH. CONTAMINATED FOOD and WATER. Signs and Symptoms: DYSENTERY, ULCERATIONS, BLOODY-MUCOID STOOLS Complications: Notes: FOOD INTOXICATION
Disease: SALMONELLOSIS Causative organism: SALMONELLA
Type of Organism: bacterial Part of Body Affected: Digestive System Mode of Transmission: ORAL, BY MOUTH. INGESTING, PET TURTLES, REPTILES in general* Signs and symptoms: ENDOTOXIN RELEASED* Complications: Notes: FOOD INFECTION
Disease: PSEUDOMEMBRANOUS COLITIS (GASTROENTERITIS) Causative organism: CLOSTRIDIUM
Type of Organism: bacterial Part of Body Affected: Digestive System Mode of Transmission: ORAL. BY MOUTH
Disease: HEPATITIS A- INFECTIOUS HEPATITIS
Type of Organism: bacterial Part of Body Affected: Digestive System Mode of Transmission: ORAL. BY MOUTH Signs and symptoms: JAUNDICE Prevention: VACCINE Acute DOESN'T CAUSE CHRONIC HEPATITIS or CIRRHOSIS Similar to Hep E
Disease: GASTROENTERITIS Causative organism: VIBRIO PARAHAEMOLYTICUS
Type of Organism: bacterial Part of Body Affected: Digestive System Mode of Transmission: ORAL. BY MOUTH. INGESTING UNCOOKED/UNDERCOOKED SEAFOOD MOST COMMON
Disease: STAPHYLOCOCCUS FOOD POISONING/INTOXICATION Causative organism: STAPHYLOCOCCUS
Type of Organism: bacterial Part of Body Affected: Digestive System Mode of Transmission: ORAL. BY MOUTH. Ingesting CONTAMINATED FOOD Produces: EXOTOXINS. VERY HEAT STABLE
Disease: HEPATITIS E- ENTERIC NON-A NON-B HEPATITIS
Type of Organism: bacterial Part of Body Affected: Digestive System Mode of Transmission: ORAL. CONTAMINATED water and food Signs and symptoms: JAUNDICE Similar to Hep A Acute
Disease: HEPATITIS C-POSTTRANSFUSION NON A NON B HEPATITIS
Type of Organism: bacterial Part of Body Affected: Digestive System Mode of Transmission: as for HBV, TRANSFUSIONS before 1992 Signs and symptoms: JAUNDICE
Disease: GASTROENTERITIS Causative organism: ESHERICHIA & CAMPYLOBACTER
Type of Organism: bacterial Part of Body Affected: Digestive System Signs and symptoms: INFANTILE/EPIDEMIC DIARRHEA, TRAVELLERS DIARRHEA, HEMOLYTIC-UREMIC SYNDROME
Disease: LEPTOSPIROSIS
Type of Organism: bacterial Part of Body Affected: Genital and Urinary System Complications: WEIL'S DISEASE- KIDNEYS and LIVER BECOME INFECTED
Disease: GONORRHEA Causative organism: NEISSERIA
Type of Organism: bacterial Part of Body Affected: Genital and Urinary System Mode of Transmission: STD, Progress to PID Complications: NO IMMUNITY - May cause STERILITY in males OPTHALMIA NEONATORUM -blindness
Urinary tract infection
Type of Organism: bacterial Part of Body Affected: Genital and Urinary System Signs and symptoms: FREQUENCY and URGENCY of URINATION Complications: DYSURIA - DIFFICULTY or PAIN in URINATING, BURNING feeling NOCTURIA - INCONTINENCE, INVOLUNTARY VOIDING of URINE at NIGHT BLATTER CRAMPS and SPASMS.
Disease: CHLAMYDIAL URETHRITIS, NGU, NONGONOCOCCAL URETHRITIS Causative organism: CHLAMYDIA
Type of Organism: bacterial Part of Body Affected: Genital and Urinary System Mode of Transmission: Signs and symptoms: Complications: FALLOPIAN TUBES, BLOCKAGE/SALPINGITIS Females: PID- PELVIC INFLAMMATORY DISEASE - ANY EXTENSIVE BACTERIAL INFECTION of PELVIC ORGANS Males: May CAUSE STERILITY CHLAMYDIAL OPTHALMIA/INCLUSION CONJUNCTIVITIS: ACQUIRED by NEWBORNS, BURNING PASSAGE THROUGH an INFECTED BIRTH CANAL
Disease: SYPHILIS Causative organism: TREPONEMA
Type of Organism: bacterial Part of Body Affected: Genital and Urinary System Mode of Transmission: STD Signs and symptoms: PRIMARY: CHANCRE SECONDARY: SKIN RASH TERTIARY: GUMMA Complications: NEUROSYPHILIS - INSANITY Notes: CONGENITAL SYPHILIS HUTCHINSON'S TRIAD
Disease: ACUTE NECROTIZING ULCERATIVE GINGIVITIS, ANUG, TRENCH MOUTH Causative organism: PROPHYROMONAS BACTEROIDES* PREVOTELLA
Type of Organism: bacterial Part of Body Affected: Mouth/Digestive System Mode of Transmission: Signs: ULCERS & NECROSIS of PERIODONTAL TISSUE ➡️ Tooth loss PREDISPOSING FACTORS: Poor oral hygiene, stress, heavy smoking, malnutrition.
Disease: DENTAL CARIES Causative organism: STREPTOCOCCUS
Type of Organism: bacterial Part of Body Affected: PERIODONTAL DISEASE Ferments sugars* ➡️ Lactic and other acids* ➡️ Dissolves tooth enamel and dentin.*
Scalded Skin Syndrome Causative organism: staphylococcal
Type of Organism: bacterial Part of Body Affected: skin Mode of Transmission: Direct contact Characteristic Sign/Symptom: DESQUAMATION- epidermis becomes necrotic and peels off Complications: TOXIC SHOCK SYNDROME - TSS TOXIN Notes:EXFOLIATIVE TOXIN
Disease: BLASTOMYCOSIS Causative organism: BLASTOMYCES
Type of Organism: fungal Part of Body Affected: Lower Respiratory
Disease: ASPERGILLOSIS
Type of Organism: fungal Part of Body Affected: Lower Respiratory Mode of Transmission: INHALATION of SPORES Signs: CONTINUES to GROW in a MOLD form WITHIN the BODY - ASPERGILLOMA's, BALL of MYCELIUM in the LUNGS (not dimorphic)
Disease: PNEUMOCYSTIS PNEUMONIA (PCP) Causative organism: PNEUMOCYSTIS
Type of Organism: fungal Part of Body Affected: Lower Respiratory Mode of Transmission: INHALATION, DROPLET route Signs and Symptoms: Complications: often FATAL in AIDS patients
Disease: HISTOPLASMOSIS - DARLINGS DISEASE Causative organism: HISTOPLASMA
Type of Organism: fungal Part of Body Affected: Lower Respiratory Mode of Transmission: INHALATION of SPORES Notes: DIMORPHIC FUNGUS
Disease: Thrush/Oral Candidiasis Causative organism: CANDIDA*
Type of Organism: fungal Part of Body Affected: MUCUS MEMBRANE of the ORAL CAVITY* Mode of Transmission: Vaginal passage- Opportunistic Characteristic Sign or Symptom: WHITE MILK LIKE GROWTH COVERING the MUCUS MEMBRANE
Causative organism: CRYPTOCOCCUS
Type of Organism: fungal Part of Body Affected: Nervous system Mode of Transmission: INHALATION of SPORES Characteristic Sign or Symptom: CHRONIC MENINGITIS Complications: cutaneous lesions Notes: YEAST FORM ONLY
Disease: INCLUSION CONJUNCTIVITIS Causative organism: CHLAMYDIA
Type of Organism: fungal Part of Body Affected: eyes Mode of Transmission: Fingers/Fomites; Flies (birth canal) Characteristic Sign: scar tissue on cornea Symptom: dry eyes Complications: BLINDNESS-LEADING CAUSE*
Disease: MADURA FOOT Causative organism: Mycetoma, MADURELLA
Type of Organism: fungal Part of Body Affected: skin
Disease: FONSECAEA Causative organism: CHROMOBLASTOMYCOSIS
Type of Organism: fungal Part of Body Affected: skin Characteristic Sign or Symptom: VARICOSE LESIONS- tough, Warty
Disease: PIEDRA
Type of Organism: fungal Part of Body Affected: skin Characteristic Sign or Symptom: colored nodules on hair shaft Complications: WHITE PIEDRA: TRICHOSPORON BLACK PIEDRA: PIEDRAIA
Disease: TINEA VERSICOLOR or PITYRIASIS VERSICOLOR Causative organism: MALASSEZIA fur fur
Type of Organism: fungal Part of Body Affected: skin Characteristic Sign or Symptom: spotty appearance
Disease: SPOROTHRIX Causative organism: Sporotrichosis (Rose Gardener's Disease)
Type of Organism: fungal Part of Body Affected: skin Mode of Transmission: thorns and splinters Characteristic Sign or Symptom: hard nodule and lesions
Disease: VAGINITIS* Causative organism: CANDIDA*
Type of Organism: fungal Part of Body Affected: vaginal skin Characteristic Sign or Symptom: itching, burning sensation, white "cheesy" discharge Complications: systemic; meninges & heart tissues
Disease: MYCOTOXINS- Poisonous Mushrooms Causative organism: AMANITA
Type of Organism: fungi- MYCOTOXINS Part of Body Affected: Digestive System Potent toxins: PHALLOIDIN & AMANITIN
Disease: AFLATOXIN Causative organism: ASPERGILLUS
Type of Organism: fungi- MYCOTOXINS Signs and symptoms: CARCINOGEN. Known to CAUSE CANCER, LIVER and COLON
Disease: ERGOT POISONING Causative organism: CLAVICEPS
Type of Organism: fungi- MYCOTOXINS Signs and symptoms: HALLUCINATIONS, CONSTRUCTION of CAPILLARIES ➡️ GANGRENE of the EXTREMITIES
Disease: CHICKENPOX Causative organism: Herpes VZ/VERACELLA/ZOSTER VIRUS -VZV*
Type of Organism: viral Mode of Transmission: RESPIRATORY DROPLETS and SKIN LESIONS Notes: VIRUS can CROSS the PLACENTA ➡️ CONGENITAL ABNORMALITIES*
Disease: SHINGLES Causative organism: Herpes zoster
Type of Organism: viral Mode of Transmission: RESPIRATORY DROPLETS and SKIN LESIONS Prevention: VACCINE Note: more localized
Disease: EBOLA Causative organism: EBOLAVIRUS
Type of Organism: viral Part of Body Affected: Cardiovascular & lymphatic system Mode of Transmission: DIRECT CONTACT with BODY FLUIDS* Characteristic Sign or Symptom: BLEED BOTH INTERNALLY and EXTERNALLY - hemorrhaging
Disease: PULMONARY SYNDROME/ SIN NUMBRE VIRUS Causative organism: HANTAVIRUS
Type of Organism: viral Part of Body Affected: Cardiovascular & lymphatic system Mode of Transmission: INHALATION of MOUSE droppings SALVIA, FECES and URINE Characteristic Sign or Symptom: LUNGS FILL with FLUID Complications: DEATH from RESPIRATORY FAILURE Causes : HANTAVIRUS PULMONARY SYNDROME- 4 CORNERS DISEASE
Disease: DENGUE HEMORRHAGIC FEVER Causative organism: YF VIRUS
Type of Organism: viral Part of Body Affected: Cardiovascular & lymphatic system Mode of Transmission: MOSQUITO Characteristic Sign or Symptom: BREAK BONE FEVER* high fever, sharp pain in muscles and joints Complications: HEMORRHAGES in SKIN Notes: SHOCK
Disease: YELLOW HEMORRHAGIC FEVER Causative organism: FLAVIVIRUS YF
Type of Organism: viral Part of Body Affected: Cardiovascular & lymphatic system Mode of Transmission: MOSQUITO* Characteristic Sign or Symptom: JAUNDICE - BILE PIGMENTS in BLOOD, BLEEDING GUMS, BLOODY STOOLS, VOMITING BLOOD, DELIRIUM Complications: fatal hemorrhaging
Disease: ZIKA VIRUS Causative organism: FLAVIVIRUS
Type of Organism: viral Part of Body Affected: Cardiovascular & lymphatic system Mode of Transmission: MOSQUITOS and SEXUAL CONTACT* CROSSES the PLACENTA causing MICROCEPHALY*
Disease: CYTOMEGALIC INCLUSION DISEASE Causative organism: CYTOMEGALOVIRUS (CMV)
Type of Organism: viral Part of Body Affected: Cardiovascular & lymphatic system Mode of Transmission: TRANSPLACENTAL Characteristic Sign or Symptom: Complications: 1. CONGENITAL CMV -BLUEBERRY RASH- ENLARGED LIVER and SPLEEN- HEPATOSPLENOMEGALY - JAUNDICE and CAPILLARY BLEEDING 2. MICROCEPHALY Disseminated Cytomegalovirus OPPORTUNIST in AIDS PATIENTS HIGH MORTALITY
Disease: INFECTIOUS MONONUCLEOSIS*/KISSING DISEASE Causative organism: EPSTEIN-BARR VIRUS (EBV)
Type of Organism: viral Part of Body Affected: Cardiovascular & lymphatic system Mode of Transmission: saliva Characteristic Sign or Symptom: ENLARGED cervical LYMPH NODES- LYMPHADENOPATHY, B CELLS proliferation Complications: ONCOGENETIC Notes: 1. BURKITT'S LYMPHOMA-jaw 2. NASOPHARYNGEAL CARCINOMA 3. LYMPHATIC TUMORS- kidney transplant or aids
Disease: NORWALK AGENTS/ROTAVIRUS Causative organism: NOROVIRUSES
Type of Organism: viral Part of Body Affected: Genital and Urinary System Signs and symptoms: GASTROENTERITIS Notes: MOST COMMON cause of VIRAL ENTERIC DISEASE WORLDWIDE
Disease: COMMON COLD
Type of Organism: viral Part of Body Affected: Upper Respiratory Mode of Transmission: INHALATION, FOMITES Characteristic Sign or Symptom: sneezing, nasal secretions, congestion Notes: 1. CORONAVIRUS 2. RHINOVIRUS (most common) 3. ADENOVIRUS (most severe)
GENITAL WARTS* Causative organism: HUMAN PAPILLOMA VIRUS - HPV*
Type of Organism: viral Part of Body Affected: genital and urinary Mode of Transmission: Direct contact Characteristic Sign or Symptom: CONDYLOMATA ACUMINATA Associated with CANCER OF THE CERVIX & PENIS
Disease: ROSEOLA/6th Disease Causative organism: HUMAN HERPES VIRUS- HHV6
Type of Organism: viral Part of Body Affected: skin Mode of Transmission: Direct contact Characteristic Sign or Symptom: Rash
Disease: 5TH DISEASE/Erythema infectiosum /Slap Face/Slap Cheek Causative organism: HUMAN PARVOVIRUS B19
Type of Organism: viral Part of Body Affected: skin Mode of Transmission: Direct contact Characteristic Sign or Symptom: facial rash & flu symptoms
WARTS* Causative organism: HUMAN PAPILLOMA VIRUS - HPV*
Type of Organism: viral Part of Body Affected: skin Mode of Transmission: Direct contact Characteristic Sign or Symptom: wart Complications: may be associated with cancer
Disease: COLD SORE/FEVER BLISTER Causative organism: Herpes Simplex Type 1 (HSV-1*)
Type of Organism: viral Part of Body Affected: skin Mode of Transmission: Direct contact/Oral Characteristic Sign or Symptom: lesions on oral mucosa Notes: HERPETIC WHITLOW - infection of fingers
Disease: GENITAL HERPES Causative organism: HERPES SIMPLEX TYPE 2 (HSV-2*)
Type of Organism: viral Part of Body Affected: urinary and genital system Mode of Transmission: STD Characteristic Sign or Symptom: HSV2 REMAINS LATENT in DORSAL NERVE ROOT GANGLIA Notes: 1. NEONATAL HERPES - CESAREAN SECTION RECOMMENDED 2. TRANS PLACENTAL - MICROCEPHALY and MICROPTHALMIA
Disease: SMALLPOX Causative organism: VARIOLA
Type of Organism: virus Notes: ERADICATED*
Disease: Rubeola/ Measles Causative organism: MORBILLIVIRUS
Type of Organism: virus Part of Body Affected: skin Mode of Transmission: RESPIRATORY DROPLETS/CONJUNCTIVA Characteristic Sign or Symptom: Koplik spots (bulls eye along gums) Complications: Subacute sclerosing panencephalitis - SSPE Notes: MMR VACCINE
Disease: RUBELLA/ GERMAN MEASLES Causative organism: RUBIVIRUS
Type of Organism: virus Part of Body Affected: skin Mode of Transmission: respiratory droplets Characteristic Sign or Symptom: rash and fever Complications: CONGENITAL RUBELLA
vectors
Typically arthropods (insects, ticks)
removing DNA primer and fixing errors
proofreading
Eclipse period
Uncoating because no intact viruses in the cell
Protozoa Characteristics
Unicellular eukaryotes that lack a cell wall
Yeast
Unicellular fungi that is 1-5 um wide and 5-30 um long Eukaryotic, facultative anaerobes, polysaccharide cell wall
Viral PAMP
Unmethylated C-G repeats high frequency
A (this reaction to medication and the morphology of the worm suggests ascariasis, for which the ascaricide albendazole is the best drug of choice - pyrantel pamoate is also good, but may cause intestinal obstruction if the worm load is high - it's also contraindicated in pregnant women and young children) (B; use this for Giardia and Entamoeba) (C; use this for cryptosporidiosis) (D; this could treat the symptoms of ascariasis, but it does not kill or expel the worms) (E; who wants chowdah?)
Upon giving a patient some medication for a suspected parasitic worm infection, the patient vomited up a large bolus of dozens of thick live worms. An even larger amount were found in his stool. The drug most likely administered was A. Albendazole B. Metronidazole C. Nitazoxanide D. Corticosteroids E. Ipecac
What is a rumen?
Upper part of the ruminant stomach that contains large, diverse population of microbes.
How does bacteria attach to a host cell?
Using a capsid or cell envelope.
What are common portals of entry for Cardiovascular infections?
Usually occur if something enters the bloodstream directly -Into bloodstream through lesions in skin -Kidney infections can transmit pathogen to bloodstream through glomeruli (tonsils) -Childbirth
Resistance to antibiotics sometimes considered a *what*
V factor
Causes chickenpox and shingles Member of Herpes family, produces distinctive rash
VZV: Varicella/Zoster virus
Haemophilus influenzae
Vaccination is responsible for the reduced incidence of severe invasive infections caused by
What is the vaccine for Tularemia?
Vaccine (live attenuated)
What is the agent in Chicken Pox and Shingles ?
Varicella-zoster, herpes virus
What is the agent in Smallpox?
Variola
Trophozoite
Vegetative form of protozoa
What type of rash is Impetigo?
Vesicular
What kind of rash is a staph infection?
Vesticular
What bacteria causes cholera?
Vibrio cholerae
Cholera (disease of the digestive)
Vibrio cholerae, G- vibrio in salty waters; vomiting ,rice water stools, sunken eyes, cramps, coma, hypotension , death if left untreated within 48 hrs; grows in small intestine and produces an exotoxin
What are the symptoms for rabies?
Violent behavior, fear of water , Confusion, loss of consciousness
What are the symptoms of Tetanus?
Violent muscle contractions Lock-jaw Motor neurons firing uncontrollably (Patient can survive symptoms, patient will live)
What type of disease is Dengue fever ?
Viral
How does viral release work for enveloped viruses?
Viral budding, the viruses pick up membranes from host cell.
What is Poliomyelitis?
Viral infection of Motor neurons in the spinal cord
What is Hemorrhagic Fever ?
Viral infection which causes hemorrhaging (capillaries rupturing on skin)
What type of infection is Conjunctivits?
Viral, Bacterial, can also be Allergens
Individual infectious virus particle is called what?
Virion.
•
Virulence factors (diphtheria toxin)
What is the invasive ability of a pathogen?
Virulence factors that help to establish a pathogen
What kind of pathogen causes Rabies
Virus
5)
What are some of the microenvironment occurring on skin of the human body? Do microbial communities differ between these different microbial habitats?
8)
What are the different categories of exotoxins, and how do they function?
Dependent part of the name
What type of nucleic acid is being copied
When does someone have AIDS and not HIV anymore?
When the t cell count drops below a certain level leaving the body open for opportunists diseases
Provirus
When viral DNA has been incorporated into the host cell's DNA
D (the shedding of cysts is intermittent and inconsistent, making wet mounts and samples equally unreliable - the lateral flow immunoassay has a 100% sensitivity and specificity, can be read in 15 minutes, eliminates the need for a trained miscroscopist, and doesn't rely on the presence of cysts in the stool)
Which laboratory test is superior for diagnosing giardiasis? A. Wet mount examination of stool B. Trichrome stained stool sample C. DNA analysis by PCR D. Lateral flow immunoassay E. Immunofluorescence assay
Polymerase part of the name
Which nucleic acid is being synthesized
C, E (the sequence is egg, miracidium, sporocyst, redia, cercaria, metacercaria, adult) (egg passed in feces into water, miracidium swims to infect first intermediate host, the snail, and becomes sporocyst, which asexually divides to become the redia, which produces the cercaria that swims to the 2nd intermediate host, where it becomes the metacercaria and encysts - the adult forms in the definitive host after the metacercaria is eaten)
Which of the following fluke life stages is adapted for free-swimming? (Select all that apply) A. Egg B. Metacercaria C. Cercaria D. Redia E. Miracidium F. Adult G. Sporocyst
E (but this is still a global disease that can and does affect any age group or nation)
Which of the following groups has the highest rate of helminth infections? A. Adults in industrialized countries B. Children in rural America C. Teenage males D. Young girls in cold climates E. Children in rural tropical or subtropical areas
B (these are the flukes, several species of which can predispose to cancers of the liver, gallbladder, and bladder)
Which of the following helminth families is best known for promoting malignancies? A. Nematoda B. Trematoda C. Cestoda D. Annelida
C (E. granulosus causes hydatid disease, and to diagnose it, you use imaging to find the cyst and an aspirate to find the proscolices inside)
Which of the following is an incorrect pairing between the parasitic pathogen and the procedure used to diagnose it? A. Cryptosporidium ::: acid-fast stain of stool sample B. Schistosoma haematobium ::: microscopic examination of urine sample C. Echinococcus granulosus ::: routine ova screen D. Taenia solium ::: microscopic examination of stool sample
C (instead, they actually resemble the eggs of tapeworm species, namely fish tapeworms - you'll need a specialist to tell them apart, but the buzzword for the eggs is 'operculated', meaning they look like they have little caps on ONE end [as opposed to having caps on BOTH ends, which is a trait of whipworm eggs])
Which of the following is false about the family represented by the pictured organism? A. Infection can cause symptoms ranging from biliary fibrosis and diarrhea to nasopharyngeal obstruction B. Its standard life cycle involves the infection of at least three hosts, one definitive and two intermediate C. The eggs produced by the species of this family greatly resemble those of the threadworm species D. The drug of choice for treatment of infection is praziquantel
E (the dwarf tapeworm, Hymenolepis nana, has a direct life cycle - it only infects one host, which serves as both the definitive and intermediate host - typically rats and humans)
Which of the following is false regarding cestode reproduction? A. Cestodes can reproduce sexually or asexually B. Cestodes require a definitive host for their sexually mature stage to survive C. Cestodes are hermaphrodites D. The proglottids contain both male and female gonads, and each individual proglottid is capable of making numerous eggs E. All species of tapeworm require an intermediate host separate from their definitive host to complete their life cycle
E (this is a Giardia intestinalis cyst, which colonizes the duodenum, is not-invasive and therefore does not cause bloody diarrhea) (the symptoms are caused by the trophozoite stage of the organism, NOT the cyst)
Which of the following is false regarding the pictured organism and life cycle stage? A. It is in the infective stage B. It is in the sexually mature stage C. It is in the environmentally resistant stage D. It is transmitted fecal-orally E. It causes bloody diarrhea as a chief symptom
A (E. granulosus forms a cyst with a capsule that settles in the liver and is treatable - E. multilocularis forms cysts with no capsules in multiple locations, and is usually fatal)
Which of the following is most characteristic of Echinococcus worms? A. Hydatid cysts B. Cutaneous larva migrans C. Predisposition to hepatic and cystic cancer D. Charcot-Layden crystals E. Eggs in the urine
D (flukes are members of the phylum Platyhelminthes, all of whom are unsegmented flatworms)
Which of the following is not a morphological feature of an intestinal fluke? A. Leaf-shaped B. Dorsoventrally flattened C. Oral and ventral suckers D. Segmented E. Unbranched digestive system
A (diarrhea is a pathologic effect of protozoans, bacteria, and viruses - things that alter the biochemistry of the GI tract with toxins and virulence factors to cause nutrient or water loss - helminths just attach and suck, or migrate, until they can reproduce and pass out eggs)
Which of the following is not a pathological effect of helminth infections? A. Diarrhea B. Hemorrhage C. Enlargement and swelling of organs D. Anemia E. Weight loss
D (you see ascariasis, you should think viscera larva migrans - Ascaris doesn't hang out in the skin, ever - the egg is ingested, hatches, migrates to the liver, migrates to the lung, gets coughed up and reswallowed) (this is a big worm, and one of the most prevalent global parasites - can be easily fatal by causing intestinal obstruction by sheer volume of worm bolus - the female lays 200,000 eggs a day, and the eggs can remain viable in the soil for 10 years)
Which of the following is the correct path of travel for the parasite Ascaris lumbricoides upon infecting a host? A. Skin B. Skin -> Muscle C. Skin -> Intestine -> Liver -> Lung D. Intestine -> Liver -> Lung -> Intestine E. Intestine -> Liver
Chocolate agar
Which of the following media provides the factors necessary for the growth of Haemophilus spp.?
A (the mnemonic is EAT - for enterobus, ascaris, and trichuris) (Strongyloides penetrates the skin, ascends into the lungs, and is swallowed, to reach the intestines)
Which of the following nematodes does not cause intestinal infection by ingestion of its eggs? A. Strongyloides B. Enterobus C. Ascaris D. Trichuris
C (Ascaris eggs require a period of development in soil before they are infective - ingesting the larva or unembryonated eggs will not get you sick)
Which of the following organisms and infections can occur under normal conditions but not in a hospital setting? A. Hymenolepis nana - dwarf tapeworm disease B. Taenia solium - cysticercosis C. Ascaris lumbricoides - ascariasis D. Enterobius vermicularis - pinworm infection
E (Giardia interferes with intestinal absorption - Plasmodium causes tissue absorption by lysing red blood cells)
Which of the following parasites is paired with its LEAST correct mechanism of pathogenesis? A. Diphyllobothrium latum ::: pernicious anemia B. Ascaris lubricoides ::: intestinal obstruction C. Hookworms ::: microcytic anemia D. Echinococcus granulosus ::: increased cranial pressure E. Giardia ::: tissue destruction
D (Chlorine does NOT kill Giardia - iodine does, as does boiling) (filters are somewhat effective, depending on the filter - giardia cysts are flexible and can pass through, but this is still more effective than chlorination)
Which of the following prevention and control methods on water against Giardia is LEAST effective? A. Filters B. Boiling C. Iodination D. Chlorination
Proprionibacterium sp.
Which of these anaerobes is part of the normal flora of the skin and is often the cause of acne:
•
Why is antimicrobial susceptibility testing important?
•Cytotoxins (phospholipases and non-phospholipases)
Work by *degrading cytoplasmic membrane integrity, causing cell lysis and death* --->*Hemolysins* - toxins that lyse red blood cells (i.e. erythrocytes) --->*Production of hemolysins* can be *tested on blood agar plates* --->Zone of hemolysis
Candida albicans
Yeast causing candidiasis leading to thrush, vaginitis, balantitis, dermatitis, onychomycosis, septicemia, meninges infections Dimorphic & opportunistic
Fungi Groups
Yeast, mold, fleshy fungi
What is Yellow Fever?
Yellow fever virus, attacks RBC
What agent is the Bubonic Plague?
Yersinia pestis
do microbes outnumber us?
Yes
E (hookworms have a skin-lung-intestine path, and form their eggs in the lungs - contrast this with the Strongyloides, which use the rhabditiform larva instead and do not lay eggs)
You suspect your patient is infected with hookworms. Which of the following is the diagnostic stage for this pathogen? A. Cyst B. Procercoid C. Trophozoite D. Rhabditiform larva E. Egg
C (Anisakis worms, also called herring worms, use fish as paratenic hosts and marine mammals like whales and porpoises as definitive hosts - the human is an accidental host)
Your patient, Miles, has complaints of severe acute abdominal pain after eating ceviche, a Latin American dish that utilizes raw fish. A gastroscopic examination revealed the presence of parasitic helminth larva. Miles is which of the following for this parasite? A. Definitive host B. Paratenic host C. Accidental host D. Vector host E. Faculty awards host
Name one emerging infectious disease, the agent causing the disease, how is it transmitted, what are the symptoms, is there a treatment?
Zika -mosquito borne disease -when present, symptoms are mild and last less than a week. They include fever, rash, joint pain, and red eyes. -no vaccine or specific treatment, relieving symptoms
Tropical and other arboviruses South America:
Zika virus
you can treat shingles with what
anti herpes medication
clostridium botulinum
a Gram-positive, rod-shaped, anaerobic, spore-forming, motile bacterium with the ability to produce the neurotoxin botulinum. Can cause paralysis or death.
clostridium difficile
a bacterium that can cause symptoms ranging from diarrhea to life-threatening inflammation of the colon
HIV is no longer a death sentence like it once was. with current medications it is now considered what
a chronic disease
Giardia muris
a flagellate protozoan, is an intestinal parasite found in rodents, birds and reptiles
Blepharisma Japonicum
a free-living ciliated protozoan
Pathogen must be able to leave host to be transmitted to
a fresh host
vector
a live animal (other than human) that transmits an infectious agent from one host to another
proteobacteria
a major phylum of Gram-negative bacteria
pathogen
a microbe that can cause disease
firmicutes
a phylum of bacteria, most of which have Gram-positive cell wall structure.
opsonization
a process that enhances phagocytosis by which bacteria are coated by opsonins so as to become more readily and more effeciently engulfed by phagocytes
Lactobacillus acidophilus
a resident organism in the vagina, ferments the glycogen, producing lactic acid
Ergosterol
stabilizes the fungal membrane
natural immunity
acquired as part of normal life experiences
artificial immunity
acquired thru a medical procedure ex: vaccine
Protein molecules produced by *what*
activated B cells
naive cytotoxic t cells
activated by APCs or cytokines released from dendritic cells, macrophages, and t helper cells -divide into: effector cytotoxic cells, or memory cytotoxic cells
complement fixation
activation of the classical complement pathway can result in the specific rupturing of cells and some virues
eosinophils
active in worm and fungal infections, allergy, and inflammation
immature t helper cell
actively differentiating t helper cells from original naive t cell ultimately mature into: t helper 1 cells, t helper 2 cells, memory cells
biological vector
actively participate in pathogens life cycle
interferon
acts against viruses involved in immune regulations inhibits viral replication
The "illness" phase is also called the "_______" phase
acute
What are the nitrogenous bases in DNA? What are the pairs?
adenine - thymine guanine - cytosine
-stasis/static
stand still
attachment to specific tissues (adhesin binding)
adhesins: specific structures or macromolcues on microbes that aid in attachment receptors: complimentary macromolecule binding site on eukaryotic surface that binds specific adhesins binding is highly specific and typically dictates type of host to which bacteria can attach
movement of lymph
adjacent skeletal muscle one way valves, arterial pulsations
How is TB transmitted?
aerosols (prolonged contact)
primary response
after first exposure to an Ag, and after a lag period, plasma cells gradually produce igM followed by prodcution of igG (IgM produced first, IgG second)
secondary (anamnestic) response
after second contact with same Ag, immunse system produces a more rapid, stronger response due to memory cellls with high IgG titer, lower igM
Aby links cells, viruses together to make clumps that attract macrophages
agglutination
precipitation
aggregation of particulate antigen
vehicles microbes attached to dust, skin flakes, dried mucus become aerosols, travel thru air.
airborne
reticuloendothelial system (RES)
aka mononuclear phagocyte system network of reticular fibers that interconnect other cells and meshes with the connective tissue network surrounding organs inhabited by phagocytic cells ready to attack and ingest microbes that passed the first line of defense
tissues lining surfaces (that connect to the outside)
alimentary canal, respiratory system, genito-urinary tract
acids and alkalis
alkaline solution-ammonium -organic acids prevent spore germination and bacterial and fungal growth
antimicrobials
all inclusive term for any antimicrobial drug
What is the treatment for Poliomyelitis?
alleviating symptoms
serum created by
allowing blood to clot
endogenous agents
already exist on or in the body (noraml flora)
infant botulism
an illness that can happen when a baby ingests bacteria that produce a toxin inside the body.
opportunistic pathogen
an infection caused by pathogens (bacteria, viruses, fungi, or protozoa) that take advantage of an opportunity not normally available, such as a host with a weakened immune system, an altered microbiota (such as a disrupted gut flora), or breached integumentary barriers
zoonosis
an infection indigenous to animals but naturally transmissible to humans
commensal
an integral part of a complex of natural mechanisms on mucosal surfaces and skin that safeguard the resistance of the organism against pathogenic micro-organisms.
first line of defense
anatomical and physiological barriers that keep microbes from penetrating sterile body compartments (physical, chemical, and genetic barriers)
secondary infection
another infection by a different microbe
What is a Disk-diffusion or Kirby-Bauer Method. Why do you use it?
antibiotic testing -uses antibiotic-containing wafers or disks to test whether particular bacteria are susceptible to specific antibiotics.
what does not work on viruses and why
antibiotics they aren't cellular
B lymphocytes are the source of
antibodies
T and B cells independently contact
antigen
antigens vs epitopes
antigen (Ag): is a substance that provokes an immune response in specific lymphocytes epitopes: smaller specific molecules that are recognized on the antigen by lymphocytes
your immune system recognizes, responds to, and remembers molecules that are *what*
antigens
synthetic drugs
antimicrobial compounds synthesized in the laboratory through chemical reactions
Leaf-cutting ants
ants that have a symbiotic relationship with lepiota (fungus)
chemotherapeutic drugs
any chemical used in the treatment, relief, or prophylaxis of a disease
Prereduced, anaerobically sterilized (PRAS) media:
are produced, packaged, shipped, and stored under anaerobic conditions.
convalescent period
as person begins to respond to the infection, symptoms decline
What shape is Listeria monocytogenes?
bacillus (rod)
genetic drift (small)
creates influenza virus with slightly modified antigens
Humoral refers to
body fluids
Furuncles (skin disease)
boil; reddening,swelling,pus; staphylococcus sp; staphylococcus aureus
boiling water
boiling at 100*C for 30 minutes destroys non-spore forming pathogens
Carbuncles (skin disease)
boils combined; staphylococcus sp; staphylococcus aureus
What is the portal of entry for Botulism ?
breaks in skin
What is the portal of entry for Rabies?
breaks in skin from infected saliva Virus travels from peripheral nerves -> brain
What is the portal of entry in a staph infection?
breaks in the epidermis
What is the portal of entry for Tetanus?
breaks in the skin
how is virus host range determined
by viruses ability to interact with the host cell -binding sites must match receptor sites
What do antibiotics target?
cell wall protein synthesis (ribosomes) cell membrane nucleic acid Other- mycolic acid
if disease is involved, *what* occurs
damage
Infectious disease can be Communicable:
can be spread from one person to another by some means Contagious: not rigorously defined. can easily be spread from one person to another, usually by contact. -----Genital herpes, measles.
Why is it important that a bacteria is selectively toxic?
can take it internally
Dysregulation of hostcell division
cancer (some viruses are associated with cancer)
true or primary pathogen
capable of causing disease in healthy person w normal immune defenses
Amoeba proteus
captures a ciliate by surrounding it with is pseudopodia
oppurtunistic pathogens
cause disease when the hosts defenses are compromised or when they grow in part of the body that is not natural to them
Mumps (disease of digestive)
caused by infection of salivary glands by mumps virus (ss enveloped RNA virus) spread by droplets; fever, headache, sore throat, face pain, swelling of parotid glands and temples or jaw. testicle pain or lumps. can lead to complications to other organs like the brain
Disease: STOMACH ULCERS and CANCER Causative organism: HELICOBACTER
causes stomach ulcers
Direct effects of viruses: Viruses alter ____ _______ ________ to promote their own replication; may lead to cancer.
cell cycle regulation
Direct effects of viruses: Viruses multiply inside host cells, using _______ __________, often killing cells.
cell resources
acute infection
comes on rapidl, with severe but short lived effects
one organism benefits, the other is not particularly benefited or harmed; to eat at the same table
commensalism
what is a successful virus (name an example)
common cold
The microbial relationship may be better described in terms of microbial *what*, not individual species
communities
Why would a microbe produce an antibiotic?
competition; kill other microbes
antibiotic that mainly works on resistance factor
competitive inhibitors
Blood and Fluid Proteins collection of >20 blood proteins that work in cascade fashion
complement
Results in lysis of infected cells, stimulation of inflammation, and increased phagocytosis
complement
Streptococcus pneumoniae (disease of respiratory)
constant inflammation bc capsule makes it hard to phagocytose. sputum is rust colored from blood coughed up from lungs, diagnosis=culture of G+ diplococci from sputum
thymus
contains t cells a specialized organ of the immune system t cells migrate here from the red bone marrow to mature function: secretion of thymosin
genetic shift how do they happen and what do they cause (big)
creates a virus with completely new and highly pathogenic antigens -cause pandemics
Put the following steps of animal virus replication in order. a) Viral replication b) Penetration c) Viral release d) Attachment to host cell e) Expression of viral genes f) Uncoating
d, b, f, e, a, c (attachment, penetration, uncoating, expression of viral genes, replication, release)
phospholipases
damage the phospholipids Unspecific *α-toxin (lecithinase)* produced by Clostridium perfringens produce dissolve membrane lipids (lecithin) in plants and animals Causes *myonecrosis* and *hemolysis* C. perfringens common infection in
Flu vaccines made from
deactivated viruses
ionizing radiation
deep penetrating power that has sufficient energy to cause electrons to leave their orbit, breaks DNA
what temporarily decreases the microbial population
defecation
what can a pathogen do
defend, attack, or hide
hemolysin
deissolve host blood cells -> release iron for bacterial metabolism (staph sp., steptococcus sp., clostridium)
artificial active immunity
deliberately exposing a person to material that is antigenic but not pathogenic
disinfection
destruction of vegetative pathogens on inanimate objects
MHC (major histocompatibility complex)
different types of T cells bind to MCH I and MCH II
hyaluronidase
digests hyaluronic acid and allows tissue penetration (staph sp., steptococcus sp., clostridium)
interleukine (IL-1, IL-2)
direct the migration of phagocytes and lymphocytes direct the differentiation of lymphocytes
malfunction in or damage to the host.
disease
pathogen
disease causing bacteria
Viruses may be latent;
disease pops up again later
what is zoonosis;
disease that can be transmitted from animals to humans
endemic
disease that exhibits a relatively stready frequency over a long period of time in a particular geographic locale
the study of disease
epidemiology
nosocomial infections
diseases that are acquired or developd during a hospital stay from surgical procedures, equiptment, personnel, ad exposure to drug resistant microorganisms
modes of action- proteins
disrupt or denature proteins
exoenzymes
dissolve extracellular barriers and penetrate through or between cells
streptokinase
dissolves fibrin clots in host allows organism to spead to other sties
small mutations, making host susceptible Requires new vaccine each year *on the test*
drift
competitive inhibition
drug competes with normal substrate for enzymes active site
selectively toxic
drugs should kill or inhibit microbial cells without damaging the host
semisynthetic drugs
drugs that are chemically modified in the laboratory after being isolated from natural sources
signs and symptoms of inflammtion
earliest symptoms of disease as a result of the activation fo the body defense: fever, pain, soreness, swelling signs of inflammation: edema, granulomas and abcesses (walled off collections of inflammatory cells and microbes)
atopic dermatitis
eczema
narrow spectrum
effective on a small range of microbes
low- level germicides
eliminate only vegetative bacteria, vegetative fungal cells, and some viruses
continually present in low numbers
endemic
family of resp. and GI viruses Includes rhinovirus, Enterovirus 68 (respiratory), Polio, hep A
eneteroviruses
Microbe needs to approach tissue, then attach to it.
entry and attachment
Transformation picks up DNA from _____
environment
Damage occurs due to
enzymes and toxins
1-5% stain red, *attack parasites*
eosinophils
higher than normal occurrence
epidemic
pandemic
epidemic across continents
microbial control methods- mechanical removal methods
filtration- disinfection and sterilization
Terbinafine
ergosterol inhibitor antifungal agent. derived from streptomyces
parasite must escape and spread to others.
escape
It is *what* that reliable information be collected so that worrisome trends can be spotted and specific measures taken.
essential
normal flora
essential to the heath of humans inhibit growth of pathoens - prevents infections enhance host defenses enhance digestion produce useful biochemicals: coenzymes, antioxidants, enzymes, vits
The study of the causes of disease
etiology
exotoxins vs endotoxin - typical sources (gram neg or pos)
ex: a few gram pos and gram neg end: all gram neg
exotoxins vs endotoxin - toxoid formation
ex: can be converted to toxoid end: cannot be converted to toxoid
exotoxins vs endotoxin - manner of release
ex: secreted from live cell end: released from cell wall during lysis
exotoxins vs endotoxin - chemical composition
ex: small proteins end: lipopolysaccharide of cell wall
exotoxins vs endotoxin - effects on the bdoy
ex: specific to a cell type (blood, liver, nerve) end: systematic: fever, shock, inflammation, weakness
exotoxins vs endotoxin - immune response
ex: stimulate antitoxins end: do not stimulate antitoxins
exotoxins vs endotoxin - toxicity
ex: toxic in tiny amounts end: toxic in higher doses
exotoxins vs endotoxin - heat denaturation at 60* C
ex: unstable end: stable
exotoxins vs endotoxin - fever stimulation
ex: usually not end: yes
How is Coccidioides diagnosed?
examining colony morphology under microscope on blood agar
Phenotype
expressed properties of the genotype
Refers to non-specific defense:
external barriers, internal blood cells, complement, interferon, etc
Bacterial Conjuctivitis (pink eye) (disease of the eye)
extremely contagious; S. aureus, S. pyogenes, Haemophilis influenzae, pseudomonas sp. , Neisseria gonnorhoeae; swollen eyelids, dilated blood vessels, grey or yellowish discharge. commonly caused by viruses. if its just bloodshot red, its pink eye but if it has yellow junk then its conjunctivitis.
Virus in saliva:
facilitates its own spread
virulence factors examples
factors used for motility factors used for attachment factors that hekp spread between tissue cells factors that help penetrate inside host tissue and kill cells factors that help overcome host defenses
Cellulitis (disease of skin)
fast spreading infection in dermis & in subcutaneous tissues below, deep cut with bacteria. pain lymphangitis (red streaks leading away from the area), fever,swelling
What are the symptoms of Chicken Pox and Shingles ?
fatigue, fever
Chemical Defenses Secretions:
fatty acids in sebum on skin, low pH and are toxic Lysozymes in tears, saliva, other fluids
Conjugation has to have what?
fertility plasmid (F+)
What are the symptoms of Roseola?
fever
What are the symptoms of Meningitis?
fever, *stiff neck*, nausea, vomiting, headache, bulging fontanelles
What are the symptoms for Tularemia?
fever, inflammation
Many diseases have unique patterns, such as
flu occurring during the winter months.
halogens
fluoride, bromine, chloride, iodine
Indirect contact, via
fomites (inanimate objects)
vehicles unpasteurized or contaminated food, either improperly grown, processed, or prepared.
food
portals of entry - GI
food, drink, and other ingested materials
What is the vaccine for Cutaneous Anthrax?
for bio-terrorism agents (and small pox)
coagulase
forms a fibrin clot in host protects organism from phagocytic cells
virulence factors that help an organism survive attack from phagocytes *coagulase*
forms fibrin clots (microbes hiding inside)
coccidioidomycosis
found in the southwest united states. called the "valley fever"
lyophilization
freeze drying; preservation of bacteria (American Stock Culture Collection)
How can you get Toxoplasmosis?
from changing your cat's litter box
Congenital syphilis (disease of urinary)
from pregnant womans circulation into the placenta and fetal tissues; inhibits fetal growth; notched incisors, deformed shin bone and aged face with saddle shaped nose
basophils
function in inflammatory events and allergies
growing viruses- animal
harder to grow 1. living animals: expensive and time consuming, need space, cleaning, food 2. chicken embryos (eggs): used to be common method for some viruses, still used to produce influenza vaccine 3. cell cultures: most common method, easier to maintain than living animals
Allergitis Rhinitis
hay fever, seasonal allergies
microbial control methods- physical agents
heat and radiation
pasteurization
heat is applied to kill potential agents of infection and spoilage without destroying the food flavor or value; milk, fruit juices, wine, beer
What enzymes make the DNA 'unzip'?
helicase which breaks the hydrogen bonds holding the complementary bases of DNA together (A with T, C with G)
HIV destroys what cells
helper T cells
Differ by type, disease progression, and transmission
hepatitis viruses
indirect protection from disease due to there being a high number of immune individuals in the population.
herd immunity
Microbial traits are controlled or influenced by _____
heredity
H5N1 "bird flu": strain, why is it a concern; why has it not yet become an epidemic among the people
high mortality rate when humans are infected (60%) -current bird->human transmission is very inefficient, no human ->human transmission has been noted
Severe or easily-caused disease is caused by pathogens with *what*
high virulence
Impetigo (skin disease)
highly contagious; isolated pustules that become crusted that are honey colored. staphylococcus aureus, streptococcus pyogenes. easily treated with pennicillins; common in children
viral encephalitis, incl West Nile. Encephalitis, reservoir includes _____ West Nile, reservoir = _______
horses birds
Disease is a condition of the *what*, not an infectious microbe.
host
T cells directly kill *what* invaded by bacteria or viruses; combat intracellular infections
host cells
Damage occurs from battling the *what*
host defenses
3rd line of defense
host defenses that must be developed uniquely for each microbe through the action of specialized WBC specific (B and T lymphocytes, antibodies, cytotoxicity)
what are the reservoirs for most human viruses
humans
influenza-difference in transmission and severity of the disease by the infection location
if it has specificity to respiratory tract-can be spread easily attaches to lower respiratory tract, more serious, but transmission is lower
antibodies Belong to class of proteins called ______________
immunoglobulins -(Ig), a subclass of globulins.
What is the diagnosis for Malaria?
immunological, or based on blood smear
where in the body do microbes usually benefit and why? What is it classified as?
in the gut. Warm, wet place to live/Plenty to eat. Mutualism
Mycobacterium tuberculosis (disease of respiratory)
inactive- organism is growing in lungs but shows no symptoms, necrotic tissue healed by calcifying, when tubercules are ruptured ,infection is reactivated. active- macrophages spread through bloodstream. diagnosed by presence of acid fast cells
what is the most effective route for biological agents
inahalation
# of new cases during a time period
incidence
flora of resp tract
included in nose/nostrils, nasal cavity, mouth, pharynx and larynx
advatanges of moderate fever
increased phagocytosis, tissue repair decreased blood iron
WBCs
innate capacity to recognize and differentiate any foreign material
When B cells multiply, some become ______ ______, able to respond later.
memory cells
stages of disease
incubation period prodromal period illness decline convalescence
source
individual or object from which an infection is acquired
Sty (skin disease)
infected eyelash, folliculitis of eyelash; staphylococcus sp; staphylococcus aureus
WBC fight what
infection
infection vs disease
infection (to corrupt): invasion/multiplication of pathogenic microorganisms intot he body; may proceed to damage tissues and result in disease disease (living apart): a loss of homeostasis in body from various cause; causes: infecton, deficiency, hereditary, physiological; characterized by certain clinical manifestions
systemic infection
infection spreads to several sites and tissue fluids usually in the bloodstream
disease caused by an infectious agent such as a bacterium, virus, fungus
infectious disease
non-communicable
infectious disease that does not arise through transmission from host to host
Gingivitis/Periodontitis (disease of digestive)
inflamed and receding gums; patches of redness and increased bleeding of the gingival
Messy death and pattern recognition lead to *what*
inflammation
Normally protective, too much results in damage
inflammation
Pattern recognition receptors activated Cytokines released, stimulating
inflammation
Endocarditis (disease of circulatory)
inflammation of endocardium; organism latches on to heart; fever, anemia, similar symptoms to heart attack, tachycardia, petechiae (rash) (broken blood vessels) under fingernails and upper half of body. can actually hear heart murmur
Pneumonia (disease of the respiratory)
inflammation of lung w accompanying fluid buildup in alveoli, high fever, difficulty breathing, lungs have reddish appearance due to dilated blood vessels
Pharyngitis (disease of respiratory)
inflammation of throat or pharynx. "sore throat"
Ureteritis (disease of urinary)
inflammation of ureter
Urethritis (disease of urinary)
inflammation of urethra
Cytolysis of host cells releases *what kind of* compounds
inflammatory
Strep throat (disease of respiratory)
inflammed mucous membranes of the throat, Streptococcus pyogenes. throat may be red w white patches, swollen lymph nodes, trouble swallowing, loss of appetite
BACTERIAL FOOD INTOXICATION
ingesting contaminated food or water containing preformed bacterial toxins. Symptoms: occur rapidly
Folliculitis (skin disease)
ingrown hair, infection of hair follicle; reddening, swelling, puss; staphylococcus sp.; staphylococcus aureus
pathogenicity
inherent or genetic ability of a pathogen to inflict damage to host may or may not occur or be expressed
primary infection
initial infection
you have it from birth
innate
What is the vaccine for HIV?
no vaccine
exit from blood by
insect bites, shared needles
Shuts down host translation and Destroys viral mRNA Makes protein synthesis for virus very difficult, but usually kills cells. Makes you feel bad.
interferon
several types, warn neighboring cells of local viral infection, induce anti viral state. -function also as interleukins
interferon
shut down mRNA and block translation which leads to cell death
interferon
immune active proteins involved in communication
interleukins
tyndallization
intermittent sterilization for substances that cannot withstand autoclaving -exposed to free-flowing steam for 30-60 mins then incubated for 23-24 hours -repeat cycle for 3 days -disinfected -used for some canned food and laboratory media
lymph circulation
interstitial areas -> lymphatic capillaries -> lymphatic ducts -> lymph nodes -> lymphatic ducts -> right subclavian vein
Like cold viruses, many, and common
intestinal viruses
probiotics
introducing known microbes back into the body
Molecules called _________ help bacteria enter host cells
invasins
Cell mediated refers to the direct *what*
involvement of cells to attack an infection
describe an antigen
is foreign is large (>5000 MW) is molecularly complex
What are the symptoms of Ringworm?
itching, characteristic rash
phenolics (carbolic acid)
joseph lister- 1867 -lysol -triclosan- antibacterial additive to soap
skin is high in *what* and also water repellant
keratin
Pyelonephritis (disease of urinary)
kidney infection
-cide
kill
oncolytic viruses
kill cancer cells -ex: strains of adenovirus
high-level germicides
kill endospores; may be sterilants
intermediate level germicides
kill fungal spores (not endospores), tubercle bacillus and viruses
What does HIV do to the cells in your body?
kills the CD4, t cells in blood stream lose adaptive immunity, body only has b cells now
vagina typical resident
lactobacillus
Genito-urinary tract: Female reproductive tract colonized, especially with ______________ which is helpful and _____ which is sometimes harmful
lactobacillus yeast
What are the symptoms of Staphylococcal Scalded Skin Syndrome?
large patches of separation of the epidermis from dermis (appears burnt looking)
Large Pustular
large pustules
immune system
large, complex, and diffuse network of tissue, vessels, cells, and flids that penetrate into every organ and tissue
macrophages
largest phagocytes; ingest and kill foreign cells required for certain specific reactions
nonionizing radiation
little penetrating power so it must be directly exposed
What is the vaccine for rabies?
live attenuated virus
hepatits viruses All multiply in *what*, generally cause jaundice (buildup of bilirubin)
liver cells
Central Dogma is the same in every _____ thing
living
lymph nodes
location: head, neck, limbs, thorax anatomy: pea size groups of nodes, containing T and B cells and macrophages function: filter lymph
How can you get VRE?
long term antibiotic use hospitalization weakened immune system surgical procedures medical devices carriers of VRE
modes of action- the cell membrane
loses integrity (holes)
thermal death time (TDP)
lowest temperature required to kill all microbes in a sample in 10 minutes
98% of lymphocytes in body are where
lymph system
lymphatic vessels
lymphatic capillaries permeate all parts of the body except the CNS, bone, placenta, and thymus
25-33% of total T and B cells, deal with specific immunity
lymphocytes
phagolysosome
lysosome fused with phagosome
Which RNA? - product of transcription - message from the genome to the ribosome
mRNA
messenger RNA
mRNA
Upon first exposure to antigen, accumulation of antibody is slow. Memory cells do what
make for a quicker, larger response afterwards.
virulence factors that help an organism survive attack from phagocytes *slime layer or capsule*
makes phagocytosis difficult and imparts an ability to survive intracellular phagocytosis
Measels (Rubeola) (disease of skin)
measels virus-ss enveloped virus; infection of respiratory tract spreads to blood then skin. syncytia formation of infected cells due to fusion protein. kopliks spots (white spots on roof of mouth), fever, rash develops on head tunk, extremities.
MMR
measles(Rubeola), mumps, and rubella
*Virulence*
measure of pathogenicity
simply spread disease, e.g. houseflies walking on feces, spread germs to humans.
mechanical vectors
recognize, bind to different types of pathogens activate cascades leading to up regulation of immune-activating proteins
membrane and endoscope proteins
microbiostatic
merely slows, or stops the growth of microbes
lymph nodes filter out
microbes ---nodes filled with macrophages and lymphocytes
localized infections
microbes enter the body and remains confined to a specific tissue
transient flora
microbes that occupy the body for only short periods of time (hrs to weeks) do not become establsihed
outer layers of skin slough off, reducing *what*
microbial load
septicemia
microorganisms are multiplying in the blood and present in large numbers
infectious dose (ID)
minimum number of microbes required for infection to proceed microbes w small IDs have greater virulence lack of ID wil not reslt in infection, but may lead to immunity of some microbes
pathogen-associated molecular patterns (PAMPs)
molecules only associated wtih microorganisms
2-8% of total grow up to macrophages, big eaters
monocytes
antibodies aka immunoglobins
monospecific (bind 1 specific antigenic determinant or epitope) structure: most are monomers, some can exist as dimers, some are pentamers
Helicobacter pylori and correlation with stomach ulcers and cancer
most common (>50% of human population) human gut bacteria H. pylori colonize the gut walls Ureases (urea CO2 + ammonia)
is there a vaccine possible for common cold?
no. different viruses and mutations in Rhinovirus, an RNA virus.
Consists of protein shell (capsid) plus nucleic acid Envelope, spikes, accessory proteins may be present
non cellular life
What are the symptoms of Keratitis?
pain, photophobia, red eye
lines various cavities in the body and surrounds internal organs.
mucous membranes
mucous membranes are easier for microbes to invade but they are coated with *what* that traps microbes
mucus
what acts as a nutrient source for bacteria
mucus
period of invasion
multiplies at high levels, becomes well-established; more specific signs and symptoms
organisms live together, both benefit
mutualism
APC-MHC II binds to ..
naive CD4 helper T cell
APC-MHC I binds to ..
naive CD8 cytotoxic T cell
Isoniazid and Ethambutal are _____ spectrum
narrow
Respiratory tract:
nasal passages, sinuses, trachea, lungs. Lungs well protected, other areas colonized.
What is the problem of using IV fluids to treat cholera in developing countries?
need sterile needles
Is Haemophilus influenzae gram + or -?
negative
Is Neisseria meningitidis gram + or - ?
negative
cuts sugar on glycoprotein receptor off end so its not sticky anymore
neuraiminidase
toxins, viruses no longer active. Because critical binding site is covered
neutralization
50-70% numerous short lived phagocytes
neutrophils
Dead cells attract *what two things*
neutrophils and macrophages
what are ex. of granulocytes
neutrophils, eosinophils, basophils
Incidence is _____ and Prevalence is _________.
new ongoing
portals of entry - skin
nicks, abrasions, punctures, incisions
What makes up DNA?
nitrogenous bases deoxyribose phosphate
can you get the flu from the flu shot?
no you would have to get it from another strain. The interferons may make you feel bad
What are the symptoms of Naegleria fowleri?
no symptoms til too late
What is the vaccine for Chagas disease?
no vaccine
commensalism
only beneficial to one organism
can cause disease under the right circumstances.
opportunistic pathogen
stimulate inflammation, act as *what*, lyse cells; work together with antibodies
opsonins
portals of entry - respiratory tract
oral andnasal cavvities
symbiotic relationship
organism existing togehter that may or mat not be a beneficial relationship
frank pathogen
organism that causes disease in the host by direct interactions with the host. AKA primary pathogen
Septicemia (disease of circulatory)
organisms are actively multiplying in blood, very flu-like, shaking, chills, gastrointestinal symptoms, increased heart rate and breathing . septic shock. spreads by use of non sterile needles
Rickettsia
origin of mitochondria, genus of gram-negative, nonspore-forming, highly pleomorphic bacteria that present as cocci, rods, or thread-like. Genus of alphaproteobacteria
exogenous agents
originate from source outside of body
RBC carry what
oxygen
Food poisoning by Stapphylococcus aureus (disease of digestive)
pain, diarrhea, vomiting. exotoxin cant be destroyed by refrigeration or by cooking
expressed properties of the genotype
phenotype
bacteriodetes
phylum composed of three large classes of Gram-negative, nonsporeforming, anaerobic or aerobic, and rod-shaped bacteria that are widely distributed in the environment, including in soil, sediments, and sea water, as well as in the guts and on the skin of animals
Actinobacteria
phylum of Gram-positive bacteria. They can be terrestrial or aquatic. They are of great economic importance to humans because agriculture and forests depend on their contributions to soil systems.
direct contact
physical contact with an infected individual/animal or large droplets that travel short distances
filtration
physical removal of microbes by passing a gas, liquid, or through ffilter -used to sterilize heat sensitive liquids and air in hospital isolation units
virulence factors
physical/biochemical characteristics that enhance the ability to inflict damage and cause disease
lymph
plasma like iquid carried by lymphatic circulation formed when blood components move out of blood vvessels into extracellular space made of water, dissolved sals, 2-5% proteins transports WBC, fats, cellular debris, and infectious agents
Can attack CNS, cause paralysis
polio
Injury to plasma membrane
polymyxin B
colonization
population increases to infectious dose or peyond while overcoming host defenses
superantigens
powerful antigens that stimulate massive rates of mitosis of T lymphocytes in an uncontrolled manner resulting in fever, shock, and death
toxin molecules come out of solution, can be cleared out.
precipitation
passive immunity
preformed antibodies are donated to an individual; does not create memory, acts immediately, and is short term
Food Poisoning by Bacillus cereus (disease of digestive)
present in soil, water, tracts of humans and animals
modes of action- protein and nucleic acid synthesis
prevention of replication, transcription, translation, peptide bond formation, protein synthesis
lymphocytes
primary cells involved in specific immune reactions to foreign matter
reservoir
primary habitat of pathogen in the natural world human or animal carrier, soil, water, plants
hydrogen peroxide
produce highly reactive hydroxyl-free radicals
toxins
produce toxins at the site of multiplication
What are antibiotics?
produced by other microorganisms
chronic infection
progress and persist over long period of time
In transcription _____ sequence tells ribosome where to bind and start
promoter
what are prions, which diseases do they cause
proteinaceous infection particles -proteins that have lost their shape and cause other prions to lose their normal shape -cause scrapie, Kuru, BSE
complicated pathway featuring inactive proteins becoming activated
prothrombin to thrombin; fibrinogen to fibrin; fibrin cross linked by Factor XII
functions of lymphatic system
provides an auxilliary route for return of extracellular fluid to the circulatory system act as a drain off system for the inflammatory response renders surveillance, recogntion, and protection against foreign materials, transports fats during digestion
Vesicular
pus, small vesicles
Inhibition of nucleic acid replication and trsncription
quinolones, rifampin, Fluoroquinolones
Which RNA? important in protein synthesis
rRNA
ribosomal RNA
rRNA
fever - nonspecific defense
raised body temp pyrogens (fever substances) - produced by activated macrophages, bacteria, viruses, and other microbes
What is Therapeutic Index
ratio between the toxic dose and the therapeutic dose -Measure of the relative safety of the treatment
Once transmission occurs, pathogen must do what
reach tissue that it can successfully infect
pathogen recognition receptors (PRRs)
recepor on WBCs for PAMPs
antigen presenting cells
recognize antigens, endocytize them, break them into small pieces, thn displat them on the surface of APC
Maculopapular
red patches, no pus but has inflammation
Bind bacteria, viruses outside of cells, leading to:
removal by phagocytes, lysis by complement, or simply inactivation
Collagenase and other proteases, hyaluronidase, streptokinase assist what
spread
Influenza virus (disease of respiratory)
ss, enveloped RNA virus. fever, sore throat, non productive cough, myalgia (muscle pain), malaise (discomfort), complications depend on age of patient. transmission by droplet. damage to cilia in respiratory tract
Pathogen must evade host defense and do what
reproduce
acquired and specific
requires exposure to microbe/allergern your body responds to, *remembers* specific invader; once exposed
portals of exit
respiratory skin scales fecal exit GU tract removal of blood
70-80% of infectious diseases of
respiratory tract
active immunity
results when a person is challenged with antigen that stimulates production of antibodies; creates memory, takes time and is lasting
hit and run virus
rhinovirus
rabies virus multiplies in
salivary glands
What is the agent in Leishmaniasis?
sandfly (VECTOR) carrying Leishmania sp.
decompose dead stuff.
saprotroph
Genetics
science of heredity
Phagocytic cells protect against *what*
secondary infections
what maintains low Ph
secretions
Genes
segments of DNA that code for functional proteins/products
kills/harms bacteria, but not you
selectively toxic
part of the antibiotic is modified, part of the antibiotic is natural
semi-synthetic
Solutions to limitation of antibiotics
semi-synthetic (look at slide 14) competitive inhibitors
details of parallel circulatory system
series of vessels and nodes drains off excess body fluids from tissues, returns fluid to cardiovascular system
*what is plasma without the clotting factors?*
serum
mixed infection
several microbes grow simultaneously at teh infection site
Influenza: virus classification, family, species
ssRNA(-) group V-virus, family:Orthomyxoviridae A - rapidly mutating, harbored in wild birds, occasionally jumps species causing outbreaks, epidemics and pandemics (esp. in pigs and humans); most severe diseases are associated with A B-almost exclusively human virus (can infect seals and ferrets), low mutation rate, less severe than A, no epidemics because of limited host range C - less common virus of humans and pigs, severe disease
portals of entry - GU tract
sexual, displaced organisms
major mixing of RNAs, whole new virus *on the test*
shift
thermal death time (TDT)
shortest length of time required to kill all test microbes at a specified temperature
things outside of target that may be affected when given an antibiotic
side effect
observable/measurable
signs
signs vs symptoms
signs: what the doctor can detect or measure; objective symptoms: what the pt notices or feels; subjective
largest organ of the body
skin
layered tissue and puncture resistant
skin
physical barriers
skin
sites that harbor normal resident microbes
skin upper resp tract GI tract outer opening of urethra external genitalia vagina external ear and canal externel eye
flora of skin and GI Tract
skin: mostly gram pos bacteria GI: highest amts of microbes in mouth and lg intestine, with smaller numers throughout
bacteremia
small numbers of pacteria present in blood not necessarily multiplying
viremia
small numbers of viruses present not necessarily multiplying
cytokines
small protein molecules secreted by cells for cell-cell communication used extensively in immune response
heavy metals (mercury, silver, gold, copper, arsenic, zinc.)
solutions of silver and mercury kill vegetative cels in low concentrations by inactivating proteins
protein molecules dissolved in blood, body fluids, and secretions
specifically antibodies
other lymphoid tissue
spleen, thymus, tonsils
other lymphatic organs/systems: protective tissues
spleen: monitors circulating blood cells and removes old RBCs and pathogens tonsils MALT/GALT
cases show up only occasionally
sporadic
pressure cooker
steam under pressure- sterilization
pathogensis
stepwise progression of host-pathogen events during disease
Genito-urinary tract: Lower portion of urinary tract contains some bacteria (VNC), but ureters and kidneys normally *what*.
sterile
nose typical resident
streptococcus
mouth typical resident
streptococcus (cheek) Neisseria (teeth)
Necrotizing Fasciitis (disease of skin)
streptococcus pyogenes, msra. extreme pain at infection site , then discolored, hot & rash. fever, malaise, confusion, nausea,. may cause shock. fatalities very high. bacteria attacks subcutaneous ct which becomes necrotic, moves swiftly under skin. limb amputation can be necessary
Define Serology
study of antibodies in blood plasma
antibiotics
substances produced by the natural metabolic processes of some microorganisms that can inhibit or destroy other microorganisms
Inhibition of essential metabolite synthesis
sulfanilamide, trimethoprim
What is the treatment of Chicken Pox and Shingles ?
symptom management , clears on own
are experienced by patient
symptoms
Food Poisoning (disease of digestive)
symptoms are from preformed toxin
collection of signs and symptoms
syndrome
functions of human microbiota
synthesis of vitamins, fermentation of non-digestible substances, digestion of harmful compounds, stimulation of immune system, antimicrobial effect, physical barrier against pathogens
t cell responses vs b cell responses
t cell: naive t cell only bind to antigens displayed on APCs - cell mediated immunity occurs b cell: naive b cells bind directly to free antigens - antibody mediated immunity occurs
transfer RNA
tRNA
additional protection from flushing actions. name some
tears, urine, saliva, other secretions
how does biological warfare differ from bioterrorism
terrorism involving the release of toxic biological agents -purpose is to create social paralysis through mass terror, confusion, and community disruption -target is civilian population (young, elderly, immunocompromised, and healthy) -pre-attack vaccination is either impossible or complex w/unacceptable complications -timing may not be obvious until many casualties have been presented, indicating that a bioterrorism attack has taken place. -by this time, massive secondary spread may have already happened sponsored by groups of people (individual)
*Pathogenicity*
the ability of a parasite to inflict
human microbiota
the aggregate of microorganisms that resides on or within any of a number of human tissues and biofluids, including the skin, mammary glands, placenta, seminal fluid, uterus, ovarian follicles, lung, saliva, oral mucosa, conjunctiva, and gastrointestinal tracts
virulence
the amount of virulence factors being expressed by the microbe during specific host-pathogen interactions a pathogen may be viurlent when infecting one type of tissue and non-virulent wen infecting different tissue a pathogen can be exhibit different levels of virulence dependent on the nature of the infection and hosts immunological condition
Parasitology
the branch of biology or medicine that is concerned with the study of parasitic organisms
obesity
the condition of being grossly fat or overweight
decontamination
the destruction, removal, or reduction in the number of undesirable microbes
synergistic effect
the effects of a combination of the antibiotics are greater than the sum of the effects of the individual antibiotics.
you produce a lot of interferons when you have what
the flu
parallel circulatory system
the lymph system
sterilization
the removal or destruction of all viable pathogens
what is biological warfare
the state sponsored use of biological toxins or microorganisms as weapons of war -target is military (fit healthy soldiers with pre-attack vaccinations -time and place of attack is likely known beforehand, allows for preparation -primary objective is mass destruction/disruption of enemy forces
Epidemiology is the study of disease:
the study of the factors involved in the frequency and spread of disease.
epidemiology
the study of the frequency and distribution of disease and health related factors in human population
mortality
the total number of deaths in a population due to a certain diseas
antimicrobial chemotherepy
the use of chemotherapeutic drugs to control infection
generally no vaccines for some viruses because why
there are not a lot of cases
What is the portal of entry for Lyme Disease?
tick bite
incubation peroid
time from initial contact with the infetious aent to the appearance of first symptoms; agent is multiplying but damge is insuffiecient to cause symptoms; several hours to several years
inflammation response
tissue injury: 1. damaged cells release chemical mediators 2. vasodilation and increased capillary permeability - fluids and phagocytes enter injured area 3. phagocytosis (neutrophils and macrophages) 4. walling off (blood clot) of affected area -> pus forms -> abscess may form -> abscess may rupture -> tissue repair
Anaerobic phenylethyl alcohol agar (PEA)
to isolate anaerobic gram-positive organisms, which of the following media should be used?
What is the treatment for Ringworm?
topical antifungal
Microbes live where it is "__________ ___________"
topologically outside -We are a tube within a tube. We have sacs open to the outside.
(transmission) example of direct contact
touching, kissing, sex, endogenous spread (one part of you to another)
virulence factors that help an organism survive attack from phagocytes *leukocidins*
toxic that destroy white blood cells
Which RNA? - brings amino acids to the ribosome - one for each amino acid
transfer RNA
Which HGT? least common
transformation
Antibiotics that target protein synthesis all target _____
translation
set of rules that determine how the nucleotide sequence is converted into the amino acid sequence
translation
Microbe gets to human tissue or already present
transmission
Microbe needs to get from reservoir to you.
transmission
What is the treatment for Infectious Mononucleosis?
treat symptoms
What is the treatment for HIV?
treatable/ but not curable
true or false Many pathogens have a preferred portal of entry and are much less likely to cause disease if they cannot get in that way.
true
what kills ciliated cells?
viral infections. ex: flu
Things that bacteria have that improve their abilities to cause disease Fimbriae, capsules, enzymes, toxins, all these things.
virulence factors
Ebola (disease of circulatory)
virus infects epithelial cells lining blood vessels; uses phagocytes to spread body to lymph nodes , liver, spleen and lungs. major inflammation to these organs. hemmorhaging, virus makes you bleed and spreads virus this way. blood vessels are breaking bc cells surrounding them are damaged.
In transduction _____ picks up host DNA and moves to another bacterium
viruses
What is the problem with the central dogma?
viruses are not living, so don't always follow the central dogma
ICTV classification of viruses
viruses that are related can be classified similarly to living things: -order, family, subfamily, genus, species
oncogenic viruses
viruses that cause cancer -ex: HPV
By mapping all the known cases of cholera, John Snow guessed cholera was spread by
water
Since cholera is an enterotoxin it makes epithelial cells highly permeable to _____
water
vehicles various viruses, bacteria, protozoa, mostly that cause diarrhea and enter water supply.
water
examples of vehicles
water, food, airborne
attachment to nonspecific tissues
weak attractive bonds
communicable disease
when an infected host can transmit the infectious agent to another host and establish infection in that host
How is Viral meningitis diagnosed?
when another cause is ruled out
sporadic
when occasional cases are reported at irregular intervals
epidemic
when prevalence of a disease is increasing beyond what is expected
are antibodies Y shaped molecules with hinges?
yes
is skin self repairing?
yes
is the GI tract crowded?
yes
plasma still has clotting factors in it. Yes or no?
yes
How are animal viruses classified?
• DNA or RNA, ss or ds • Capsid morphology • Enveloped or naked • If ss nucleic acid is it + or -
What are some antibiotic resistance mechanisms?
•Block entry •Inactivation of resistance enzymes •Alteration of target molecule •Efflux of antibiotic
What is E test?
•Gradient of antibiotics •Minimal Inhibitory Concentration
What are some factors leading to resistance? ()
•Horizontal gene transfer: plasmids -Hospitalization -World travel •Overuse -Patient demand -Immunosuppression •Misuse -Improper prescription -Failure to follow treatment -Long-term, low-dose use -Use in animal feeds
Deoxyribonucleic acid (DNA)
•Nitrogenous bases-adenine-guanine-cytosine-Thymine •Deoxyribose •Phosphate